■ちょっとした疑問はここに書いてね15■

このエントリーをはてなブックマークに追加
1わしがたてていいですか?
   ,.―――')      ______________
 γ∞γ~  \    /
 人w"/ 从从) )   /  いちいちスレッド立てないで
  ヽ | | l  l |〃 <  ちょっとした疑問はここに書いてね。
  `wハ~ ーノ)    \                 Part 15
   / \`「        ̄ ̄ ̄ ̄ ̄ ̄ ̄ ̄ ̄ ̄ ̄ ̄ ̄ ̄
前スレ
Part14:http://science.2ch.net/test/read.cgi/sci/1032629984/
【質問する前に】
教科書をよく読もう。
http://www.yahoo.co.jp/ とか http://www.google.com/ を利用し自分で調べること。
宿題を聞くときは、どこまでやってみてどこが分からないのかを書くこと。
丸投げはダメだからね。
(レスする人も丸投げに答えるのは自粛してねっていうかや・め・ろ)

質問に対する返答には、何かしらの返答ちょうだいね。

★過去スレ、関連スレはここ>>2よ。
★それから、書き込む前に>>3の注意事項を読んでね。
★数式の書き方(参考)はこちら>>4
(予備リンク:>>2-10
荒らし厳禁、煽りは黙殺、忘れないうちに定期age。
単発質問スレを発見したらこのスレへの誘導をよろしくね。>ALL
2ご冗談でしょう?名無しさん:02/10/30 02:48 ID:???
3ご冗談でしょう?名無しさん:02/10/30 02:49 ID:???
書き込む際の注意

1.)
板の性格上、UNIX端末からの閲覧も多いと推察されます。
機種依存文字(ローマ数字、丸数字等)は避けて下さい。

2.)
以下のような質問に物理板住人は飽き飽きしているので、たぶん無視されます。
しないで下さい。
「相対性理論は間違っています」「量子力学は間違っています」
「宇宙論は間違っています」「シュレディンガーの猫は変です」
「永久機関を作りました」「タイムマシンについて教えて」
「どうして〜?」関連(→「どのようにして〜?」と質問すること)
「なぜ〜?」関連(物理学の対象ではないため)
それでも何か書きたければ
相対論は:http://science.2ch.net/test/read.cgi/sci/1006249553/
反相対論は:http://science.2ch.net/test/read.cgi/sci/1023215815/
量子力学は:http://science.2ch.net/test/read.cgi/sci/1010632342/
(↑量子力学スレはありもの有効利用ということで。ご協力ください)
「どうして・なぜ」:http://academy.2ch.net/philo/
(哲学板・雑談板のほうがむいている場合が多いです。)
雑談は雑談スレで:http://science.2ch.net/test/read.cgi/sci/973536997/l50

3.)
宿題を聞くときは、どこまでやってみてどこが分からないのかを書くこと。
丸投げはダメよ。丸投げに答えるのもダメよ。
せめてポインタを示す程度に留めましょう。

4.)
厨房を放置できない人も厨房です。
4ご冗談でしょう?名無しさん:02/10/30 02:49 ID:???
【掲示板での数学記号の書き方例(数学板より拝借)】
●スカラー:a,b,c,...,z, A,B,C,...,Z, α,β,γ,...,ω, Α,Β,Γ,...,Ω, ...
●ベクトル:x=[x[1],x[2],...], |x>, x↑, vector(x) (← 混同しない場合はスカラーと同じ記号でいい.通常は(成分を横で書いても)縦ベクトルと
して扱う.)
●行列(1成分表示):A[i,j], I[i,j]=δ_(ij)
●行列(全成分表示):A=[[A[1,1],A[2,1],...],[A[1,2],A[2,2],...],...]=[a1,a2,a3,...], I=[[1,0,0,...],[0,1,0,...],[0,0,1,...],...] (← ここでは列ごとに表示(縦ベク
トルを横に並べる).行ごとに表示しても構わないが,統一して使わないと混同するので注意.)
●関数:f(x), f[x]
●数列:a(n), a[n], a_n
●足し算:a+b
●引き算:a-b
●掛け算:a*b, ab (← 通常"*"を使い,"x"は使わない.)
●割り算・分数:a/b, a/(b+c), a/(bc) (← 通常"/"を使い,"÷"は使わない.)
●複号:a±b=a士b, a干b (← "±"は「きごう」で変換可.他に漢字の"士""干"なども利用できる.)
●平方根:√(a+b)=(a+b)^(1/2) (← "√"は「るーと」で変換可.)
●指数・指数関数:a^b, x^(n+1), exp(x+y)=e^(x+y) (← "^"を使う."exp"はeの指数.)
●対数・対数関数:log_{a}(b), log(x/2)=log_{10}(x/2), ln(x/2)=log_{e}(x/2) (← 底を省略する場合,"log"は常用対数,"ln"は自然対数.)
5ご冗談でしょう?名無しさん:02/10/30 02:50 ID:???
●三角比・三角関数:sin(a), cos(x+y), tan(x/2)
●内積・外積・スカラー3重積:a・b=(a,b), axb=[a,b], a・(bxc)=(axb)・c=[a,b,c]=det([a,b,c])
●行列式・トレース:|A|=det(A), tr(A)
●絶対値:|x|
●ガウス記号:[x] (← 関数の変数表示などと混同しないように注意.)
●共役複素数:z~
●階乗:n!=n*(n-1)*(n-2)*...*2*1, n!!=n*(n-2)*(n-4)*...
●順列・組合せ:P[n,k]=nPk, C[n.k]=nCk, Π[n,k]=nΠk, H[n,k]=nHk (← "Π"は「ぱい」で変換可.)
●微分・偏微分:y', dy/dx, ∂y/∂x (← "∂"は「きごう」で変換可.)
●ベクトル微分:∇f=grad(f), ∇・A=div(A),∇xA=rot(A), (∇^2)f=Δf (← "∇"は「きごう」,"Δ"は「でるた」で変換可.)
●積分:∫[0,1]f(x)dx=F(x)|_[x=0,1], ∫[y=0,x]f(x,y)dy, ∬_[D]f(x,y)dxdy, 点[C]f(r)dl (← "∫"は「いんてぐらる」,"∬"は「きごう」で変換
可.)
●数列和・数列積:Σ[k=1,n]a(k), Π[k=1,n]a(k) (← "Σ"は「しぐま」,"Π"は「ぱい」で変換可.)
●極限:lim[x→∞]f(x) (← "∞"は「むげんだい」で変換可.)
●図形:"△"は「さんかく」,"∠"は「かく」,"⊥"は「すいちょく」で変換可.

●その他
・関数等の変数表示や式の括弧は,括弧()だけでなく[]{}を適当に組み合わせると見やすい場合がある.
・ギリシャ文字はその読み方で変換可.
・上記のほとんどの数学記号や上記以外の数学記号"⇒∀≠≧≒∈±≡∩∽"などは「きごう」で順次変換できる.
6ご冗談でしょう?名無しさん:02/10/30 04:06 ID:???
7ご冗談でしょう?名無しさん:02/10/30 07:09 ID:t+17qkaY
回転している(球状の)物体に、回転軸を変える力をかけると、かけた
力と垂直の方向に軸が動きますね。この原理を知りたいのですが。
卓球の打法のある技法を理解しようとしているところ(マジ)です。
8ご冗談でしょう?名無しさん:02/10/30 09:28 ID:???
>7
「ジャイロ効果」で検索すれ。
9ご冗談でしょう?名無しさん:02/10/30 10:02 ID:o1iq7/cT
ドップラー効果において、観測者から音源が遠ざかる場合、波長が変化しますよね。
(音の速さ)=(波長)*(振動数)
なので、振動数も伴って変化しますよね。

なのに、観測者が音源から遠ざかる場合、
振動数は変化するのに波長が変化しないのは何故ですか?
10ご冗談でしょう?名無しさん:02/10/30 10:52 ID:???
>9
観測者が音源から遠ざかるときは振動数と音の速さが伴って変化するから
波長は不変。
前者の場合は音の速さが不変なので、混同していると思われ。
走りながら物の長さを測っても変わらないのと一緒。(除:相対論的効果)
11ご冗談でしょう?名無しさん:02/10/30 18:06 ID:k7pqPVLl
パラレルワールドって本当にあるんでしょうか?
12ご冗談でしょう?名無しさん:02/10/30 18:09 ID:???
>>11
あるよ。たしか吉祥寺の近くだったかな。
13ご冗談でしょう?名無しさん:02/10/30 18:31 ID:???
149:02/10/30 18:40 ID:nnuMMO58
>>10
ありがとうございます。大体わかりました。
でも、前者が音の速さは不変なのに対して、後者は音の速さが変化する
理由がよくわかりません。。。
15ご冗談でしょう?名無しさん:02/10/30 19:30 ID:xjomhf5V
>>14
音を伝えるのは空気
音速は空気に対して一定
観測者と空気との相対運動を考えろ
16ご冗談でしょう?名無しさん:02/10/30 19:32 ID:XQggIDdh
黒色光線について誰か知りませんか?
177:02/10/30 20:33 ID:H+HPjVzC
>>8 ありがとです。でも「軸を保とうとする効果」を定性的に説明して
いる情報ばかりでした。私が知りたいのは、回転方向と軸に加える力、
その反力として軸がどのような方向に動くかという定量的な表現なのです。
私、物理・数学は高校までです。ですから回転物体の「軸ねじり抵抗度」
みたいな概念をどう扱って良いかわからんのです。
18ご冗談でしょう?名無しさん:02/10/30 20:41 ID:???
>>17
http://www.phys.ed.sojo-u.ac.jp/~cho/cons3/acons.html
とかを見てみるとヨイかも。
キーワードは角運動量保存則とトルク、モーメントあたりだよ。
19ご冗談でしょう?名無しさん:02/10/30 23:09 ID:lxsR9bV4
V=10sin(〜 [v] というのは交流で
V=10 [v] は直流と考えていいですか?
20ご冗談でしょう?名無しさん:02/10/30 23:12 ID:???
>>19
>V=10sin(〜 [v] というのは交流で
(〜 ってなんだろう?
21ご冗談でしょう?名無しさん:02/10/30 23:19 ID:lxsR9bV4
>>20
ごめんなさい。略しちゃいました。
V=10sin(100t+3.14/3) です。
22ご冗談でしょう?名無しさん:02/10/30 23:24 ID:???
>>21
うん、それは交流だね。波形が波々してるやつは交流。
まっすぐなのが直流。
23ご冗談でしょう?名無しさん:02/10/30 23:26 ID:lxsR9bV4

   r'´ ̄ ̄ ̄ ̄ ̄ヽ
   | .::.  ,,.  、_  .::|
   |:::. "'´・∀・`' ::..{
  }`=':::....::::::::     . `=´{ <ほんとありがとー
   }:::::..    . : :::...{
   |. : ..      : : .;.|
   |  : .: .:::....    :!
   `l r―――i i'´
   "ー'''    'ー-"
24ご冗談でしょう?名無しさん:02/10/31 12:05 ID:Y+tUcD/Q
>>16
ブラック・ライトのことか?
可視光をカットした紫外線ランプのこと
肉眼では光が見えないので、点灯していても黒い
25つばめさん ◆dpLvO.T.cY :02/10/31 12:21 ID:VAmrnb/Q
>>24
秘密道具のことだと。私も、嘗て、黒いセロファンで出来るかどうか試してみました。
また、光線が影によってできるとしても、黒色の部分以外の光の回折によって、
黒色の部分に回り込むのでだめ。
26anndore:02/10/31 13:00 ID:YchEvXfY
英語で死刑ってどうかくの?
27ご冗談でしょう?名無しさん:02/10/31 13:50 ID:???
和英辞典をひけよ。
28ご冗談でしょう?名無しさん:02/10/31 13:53 ID:???
すみません。はじめて投稿させていただきます。
私たちの研究室では、毎日コンピュータを使っているのですが、
研究室の院生の方々のコンピュータのスキルが、目を覆いたくなるほど
乏しくて、まったく使えません。いったい、どうしたらよいでしょうか?
29ご冗談でしょう?名無しさん:02/10/31 14:33 ID:58FfRTNy
電荷と電位の違いをわかりやすく教えてください。
30ご冗談でしょう?名無しさん:02/10/31 14:35 ID:???
>>28
>まったく使えません
じゃあ君が使えばいいじゃないのかな?
31ご冗談でしょう?名無しさん:02/10/31 15:19 ID:???
>>26
death penalty 死刑
execution 死刑執行
executioner 死刑執行人
executioner's sword 中世の死刑執行用の剣
32ご冗談でしょう?名無しさん:02/10/31 15:43 ID:RZuVXCDA
フィルターについての質問です。
http://pc3.2ch.net/test/read.cgi/tech/1036046397/
よろしくです
337:02/10/31 19:30 ID:7owuoU8p
>>18 ありがとうございます。そこの知識をこの問題にどう応用していいのか
分からないので、ここで聞いちゃいます。回転の向きを「ねじの進む方向」
で表して、↑こういう回転している物体に←この方向へ軸をねじろうとする時
矢印の頭は紙面の向こうかこっちかどちらを向こうとするのでしょうか?
34実験屋の精子:02/10/31 20:01 ID:I6pOYuZL
>>33
18ではないが、いまいち説明がよくわからん。
とりあえずどんな打法よ。
いちよ俺も卓球やってた口だからよ。
35ご冗談でしょう?名無しさん:02/10/31 20:17 ID:HzE4xAzU
こんばんは。よろしくお願いします。

量子力学では最初の状態を指定すれば、その後の状態はshorodinger方程式
ih∂|t>/∂t=H|t>で一意的に定まる。(h=ih/2πとする)
そこで時刻t=0の状態ベクトル|0>を時刻tのもの,|t>へ変換する演算子S(t)
を導入すると便利である。Sは次の性質を持つことを証明する
1)ihds/dt=HS、ただし|t>≡S(t)
2)Hがtによらない場合にはS(t)=e^-iHt/h≡Σ(h=0から∞)(-iHt/h)^n/n!
は上の方程式の初期条件S(0)=1を満たす解である。
3)Hは一般にエルミート演算子であるから上のSはユニタリーである

という問題なのですが、1)は
shorodinger eq、ih∂/∂t|t>=H|t>より
|t>=S(t)|0>の関係が成り立つならば、両辺の|t>にこれを代入し
ih∂S(t)/∂t|0>=HS(t)|0>、両辺の|0>を消去し
よってih∂s(t)/∂t=HS(t)、なんて事をして良いんでしょうか?
時間発展演算子の問題である事以外は問題の意図すら分かりません。。。
36ご冗談でしょう?名無しさん:02/10/31 20:30 ID:???
消去はよくない

移項して
(ほにゃらら) |0> = 0  ・・・(1)
の形にしてから
(1)が任意の|0>で成立するために
ほにゃらら = 0
が言える。

先に言っとくけど3)を聞く前にはユニタリーの意味くらいは
自分で調べておかないとクソを投げつけられるよきっと。
37ご冗談でしょう?名無しさん:02/10/31 21:38 ID:HzE4xAzU
>>36
ありがとうございます。「移項して(ほにゃらら) |0> = 0の形にしてから」
というのは{ih∂S(t)/∂t−HS(t)}|0>=0の形にするという事ですよね?
なるほど、この式が成立するには{ }がどういうものでないといけないのか、
と考えるんですね??ありがとうございます〜〜

あ、ユニタリーはい、分かりました、今はあやふやにしか分からないので
ちゃんとやっとこうと思います
38厨房:02/10/31 22:34 ID:xgy0g4QO
夜空は何で星の光で満たされないんですか?
あとニュースで話題になっていたけど
小柴教授のニュートリノって何によって生じたんですか?
39ご冗談でしょう?名無しさん:02/10/31 22:39 ID:81r6HETW
鞭の先端が音速を超えるというのは本当ですか?
それとも単なる俗説ですか?
よろしくおながいします。
40ご冗談でしょう?名無しさん:02/10/31 23:05 ID:???
>>38
>夜空は何で星の光で満たされないんですか?
オルバースのパラドックスのことなら、そのパラドックスを導く前提になっている
条件を調べ、その前提のうちどれが現実の宇宙では満たされていないのか
考えてみよう。

>小柴教授のニュートリノって何によって生じたんですか?
超新星爆発時に陽子と電子がくっついて中性子とニュートリノができる。
41お願いします:02/10/31 23:07 ID:METKgaQs
すいません。一般相対性理論についての質問です。空間の曲がりが重
力の原因となる理由は、アインシュタイン方程式に含まれる計量行列
が、重力ポテンシャルによって変化するから(これが正確な言い方か
どうか分からないけど・・・)だそうですが、どうして他の力(電磁力や
弱い相互作用など)でなく、重力のみが、計量行列に作用すると
分かるのですか?何か、等価原理が関係してるように思うけど、いま
いち詳細な関連が分かりません。どなたかご教授願います。
42>:02/10/31 23:33 ID:???
小柴さんのエピソード
> 東大大学院生だった小柴さんは1950(昭和25)年9月から1年半、
>神奈川県横須賀市田浦町にあった栄光学園(現在は同県鎌倉市)で
>中学生に物理を教えた。

> 「この世に摩擦がなければどうなるか答えよ」。
>これが小柴さんの問題だった。
>摩擦がないと鉛筆の先が滑って、紙に何も書けない。
>だから正解は「白紙答案」。
>正解者は3人だったと教え子たちは記憶している。

白紙答案が正解ってのはへんじゃないか。
”例えば、この答案用紙に何も書けないような世界になる”
と明確に書くことが正解だと思うんだが。。


43ご冗談でしょう?名無しさん:02/10/31 23:42 ID:0RUX4D7x
分からない記号が有るのですが質問しても良いでしょうか?
44ご冗談でしょう?名無しさん:02/10/31 23:51 ID:???
y=x^2
の坂を質点が転がるとき、質点のxの位置を
x=x(t)の式であらわせますか?
45ご冗談でしょう?名無しさん:02/10/31 23:54 ID:???
>>44
できる
4644:02/11/01 00:01 ID:???
>>45
教えてホスィ…
4744:02/11/01 00:34 ID:???
x''=-gx/(1+4x^2)
を解けばいいのですか?
48ご冗談でしょう?名無しさん:02/11/01 00:34 ID:???
>>46
坂の斜面にそった加速度のx成分でx方向の加速度は分かる
それを二回積分
4944:02/11/01 00:49 ID:???
>>48
>>47の式でいいですか?
積分苦手で解けません。
50ご冗談でしょう?名無しさん:02/11/01 01:08 ID:???
>>49
x''=-2gx/(1+4x^2)じゃないか?

積分は適当にcosとか使え
5144:02/11/01 01:37 ID:???
2を書くの忘れてました。
積分は降参します。
52ご冗談でしょう?名無しさん:02/11/01 02:18 ID:???
>>41
いや、他の力も影響するよ?
簡単な例は、ブラックホールのシュバルツシルト解に電荷を入れたやつ。
名前忘れたけど。NS解とか略称されてたかな。
時空の形が変わる。

等価原理は、ソッチ線の方程式で、対象物体の質量がでてこないことで
表現される(ような気がする。違ったかも^^)
53名無しの2年:02/11/01 02:20 ID:bs1P0Xig
エタノール水溶液で粘性度を測ると、
エタノールの重量比50%くらいまでは、粘性度が大きくなるのだが
そこからは逆に粘性度が小さくばるのはなぜ?
5441:02/11/01 02:53 ID:7YKLdJKy
>>52
レスありがとうございました。他の力も影響するということは、他の力は重力
に比べると極めて大きいから、時空をより大きく曲げることになりますよ
ね。それでも他の力には電荷や色荷があって、互いに打ち消しあうから、
重力のようにかさなって大きな力になることができず、重力ほどには時空
の湾曲に貢献しないという考え方でよいのですか?
55ご冗談でしょう?名無しさん:02/11/01 03:18 ID:???
>>54
スケールによるんじゃないかな
時空の歪みを観測しようと思ったら大きいスケールになって
重力ぐらいしか感知できないと直感的には思う
567:02/11/01 05:47 ID:OVBdTCbb
>>34 なんかスレ違いっぽいですが(^^;「ドライブ/カット等、縦回転系の球
を受ける時、回転の赤道が自分に向いていて、回転の軸は概ね左右にある。
回転の軸に近い部分を前向きにこするように打つと元の回転にあまり影響
されずにドライブ回転にして打ち返せる」技法があります。
これって「コマのみそすり運動」の原理で(元の回転を殺さずに)回転軸を
変えているのだと思われるのですが、球打ってる人に聞いても「こうすりゃ
入る」しか言わないし。球の挙動を理解するのに摩擦や運動量保存などの
原理はわかるのですが、上記の技法にはコマの軸ねじりの原理が役立つかなと
思った次第です。卓球スレで聞いても「できりゃいいだろ」で終わりそうだし。
57ご冗談でしょう?名無しさん:02/11/01 07:45 ID:???
>>56
回転軸の付近に力を加えるということは、

dL/dt = N

のNが小さいってことです。
Lはボールの角運動量(の総量)
NはトルクでLもNもベクトル量です。
N = r×F
で、rは回転軸から力の作用点までのベクトル、Fは力のベクトルです。
軸から近い場合、rがゼロに近いのでトルクNもゼロに近くなります。
つまり

dL/dt = 0

よって角運動量Lは時間的に変化ので、
元の回転を保ったまま重心運動のみが変化することになるわけです。
58ご冗談でしょう?名無しさん:02/11/01 07:46 ID:???
×時間的に変化ので
○時間的に変化しないので

書き間違えた死のう
597:02/11/01 08:27 ID:OVBdTCbb
>>57 何度もすみません。回転軸付近に加える力は「中心方向に」ではなく
「接線方向に」です。ライフル回転のまま重心運動が変化するのとは違います。
台の横から見て例えば左回転が+Lなら右回転は-Lですね。これを元の回転を
止めずに回転軸をひっくり返すことを名人はやっているように見えます。
回してから逆立ちするコマがありますね。あれも角運動量を保ったまま軸が
変わります。今まで教えてくれてるのはある(一定の向きの)軸の周りでの
角運動量についての法則なのではありませんか?「回転体の回転軸をねじる
場合の法則」みたいなやつがあるのではないかと思うのですが。
電流と力の所で出てきた、親指・中指・人差し指の向きでどっち向きの
垂直方向か決められる法則みたいな・・・
60ご冗談でしょう?名無しさん:02/11/01 09:13 ID:???
>>59
摩擦がものすごくでかいのかな?
ピン球を回転かけて地面に落とすと
行ったり来たりするのと同じように思える
61ご冗談でしょう?名無しさん:02/11/01 10:45 ID:???
>>54
そんな感じかね。
ちなみに、電子まわりの重力場が
古典的なブラックホール解になりうるかどうか調べてみると面白いと思う。
62ご冗談でしょう?名無しさん:02/11/01 10:53 ID:???
すいません。教えてください。

光はなんで一番速いんですか?
63ご冗談でしょう?名無しさん:02/11/01 11:39 ID:???
他のが遅いから
64ご冗談でしょう?名無しさん:02/11/01 12:09 ID:???
>>62
質量がないからとでも言えばいいかもしれんが、
これは理由と言っていいんだろうか。
個人的には自然がそうなってるからと言いたいが。

あと、真空中では一番速いけど、
媒質中では一番速いわけではない。
チェレンコフ光で検索してみれ。
65ご冗談でしょう?名無しさん:02/11/01 12:21 ID:???
>>62
1. 光より速いものがあると、閉じた世界線(≒タイムマシン)ができてしまう。
2. 光より速いものがあれば発見可能だが、現実には発見されていない。
66ご冗談でしょう?名無しさん:02/11/01 12:23 ID:???
>59
回転方向が入れ替わったら角運動量が保存しているとは言わない。
677:02/11/01 14:05 ID:mlG1clxU
すいません、角運動量が保存している、と回転を保ったままでいる、
とをうまく区別して言えません。(摩擦で減衰する分は無視として)
回転軸の傾きが変わると角運動量は保存しないのですか?
(歳差運動をしながら回転が続いているような状態はなんと言えば?)
元々聞きたかったことは、回転体がミソスリ運動を起こすときの法則
みたいなものです。立っているコマでいうと、重力が軸を倒す方向に
引っ張るがコマが回転していることによりそれと直角の方向に力が
かかり、軸が輪を描くみそすり運動が起きますね。ここに回転の向き
と軸を傾けようとする力(これも偶力で表すのかな?)とその結果
どちら向きの(回転?)力が発生するのか?という法則があるような
気がするのですが、用語を知らなくて検索もできない状態なのです。
68ご冗談でしょう?名無しさん:02/11/01 14:48 ID:???
>>67
俺も良く分からんが、角運動量はベクトルだから
向きが変わったら保存してるとはいえないな
69ご冗談でしょう?名無しさん:02/11/01 15:21 ID:???
>>42
答案返しちゃダメでしょ
70つばめさん ◆dpLvO.T.cY :02/11/01 15:37 ID:jll94iyz
71ご冗談でしょう?名無しさん:02/11/01 15:59 ID:???
>67
だから、その法則がdL/dt=Nなんだってば。
検索ワードは角運動量、トルク、ジャイロ効果など。
でも古典力学の基礎を知らないと理解できない話だぞ。
7235:02/11/01 20:27 ID:???
すいません。>>35です。

2)Hがtによらない場合にはS(t)=e^-iHt/h≡Σ(n=0から∞)(-iHt/h)^n/n!
は上の方程式の初期条件S(0)=1を満たす解である。

この問いはどのような方針で解けば良いんでしょうか?
73ご冗談でしょう?名無しさん:02/11/01 20:35 ID:???
それがわからないのは1)が理解できてない証拠。
74ご冗談でしょう?名無しさん:02/11/01 20:39 ID:E6twpxQf
ガビーン
あのボーアがサッカー選手だったなんて・・・今知った
しかも、銀メダル。
この板の住人のみなさんには常識でしたか?
75 :02/11/01 20:47 ID:???
7635:02/11/01 21:29 ID:???
>>73
すいません、分かりません。解き方の筋道を教えて頂けないでしょうか?
77:02/11/01 21:47 ID:Wixvu+3+
塩化第ニ鉄溶液(銅含む)にエタノ一ルを近づけると、溶液がうご<のですが・・・
78ご冗談でしょう?名無しさん:02/11/01 21:49 ID:???
>>42
摩擦がなければ答案を提出することすらできんな。
79ご冗談でしょう?名無しさん:02/11/01 21:57 ID:???
>>42 >>78
 この世界が存在することだって・・・・。
807:02/11/02 06:56 ID:8HbWfGJJ
見つけました。ありがとうございます。
81ご冗談でしょう?名無しさん:02/11/02 11:56 ID:rg2kctGK
学部レベルでも理学部物理と工学部応用物理では結構違うことやってますよね?
そこで質問なのですが、理学部物理(学部)→工学部応用物理(院)という転身と、
工学部応用物理(学部)→理学部物理(院)ではどちらが難しいのでしょうか?

それとも各々の学科はあまり関連性が無いというか、お互いに潰しが利くような
関係では無いのでしょうか?
82ご冗談でしょう?名無しさん:02/11/02 17:09 ID:EIJ+iIxk
今日、大阪市内で、小雨がぱらつきながらも夕日が差してました
で、当然のごとく虹が出ていたわけなんですが、くっきりと見える虹の上に
うっすらとぼやけて、もう一つ虹が出来ていました
虹が出来る原理はとりあえず知っているのですが、2つ同時に虹が出るというのは
どういう条件で出来るのかというのを知りません

どなたか、この現象の説明が出来る人はいませんか?
83ご冗談でしょう?名無しさん:02/11/02 17:56 ID:AJuoh6rQ
凸レンズでの焦点距離の測定をやったんですが凹レンズで測定ってできるんですか?
常に虚像しかできないので不可能かなと思ったんですが。
84ご冗談でしょう?名無しさん:02/11/02 19:01 ID:NWB0wuBA
>>82
はっきりしている=主虹は一回全反射、ぼやけている=副虹は二回(以上)
全反射。
詳しくは http://homepage1.nifty.com/tenki/weather/rainbow.html
このへんとか(横着)
85学部一年:02/11/02 20:01 ID:Hj9rpGsY
今、線形台数の教科書を復習しているのですが、定理の証明まで
理解しておかないといけないと思いますか?
現在は、定理を使いこなす勉強をしていますが、証明に関してはほとんどとばしています。
86ご冗談でしょう?名無しさん:02/11/02 20:48 ID:???
>>85
君が何専攻なのかによる。
物理専攻なら別にいらない。
知ってても損は無いが。
87ご冗談でしょう?名無しさん:02/11/02 20:54 ID:jrA/7LsE
>>83
レーザーのような平行ビームを凹レンズに入れてその後適当な
距離を置いて焦点距離のわかっている凸レンズを入れる。これに
よって細いビームを太いビームにすることができる。
凹レンズと凸レンズの距離をいろいろ変えてみて出てきたビームが
平行ビームになるように調整する。後はその距離と凸レンズの焦点距離
から凹レンズの焦点距離を求める。
88学部一年:02/11/02 21:06 ID:Hj9rpGsY
>>86
レスありがとうございます。
物理専攻ですので今までどうり勉強していきたいと思います。
89ご冗談でしょう?名無しさん:02/11/02 21:35 ID:???
>>88
まぁ・・程度の問題だと思うよ。理解しておくに越したことはないと思う。
もちろん、何も見ずに全て証明ができる必要はないと思うけど。
細かいところまでやりだすと本当にキリが無いけれど、
その定理が成り立つ前提が満たされていないのに
暗記した定理の結果だけを振り回しつづけるヤツもたまにいるから・・。
90ご冗談でしょう?名無しさん:02/11/02 22:04 ID:mrGaireX
一寸考えたのですが、太陽系の近くに中性子星が発見されたとしたら、大きな
望遠鏡を作れば、表面が見えますよね。そうすると表面では時間の進み方が、遠
方の地球から見てゆっくりだから、(観測者が中性子星にいる場合は、時間は普
通に進んでいる)観察される現象はユックリ進んでいるように見えますよね。例
えば、光の速さが通常の3/4位に見えるはずです。(地球でも地下と山の頂上で
は、山の頂上の方が僅かに時間の進み方が速いのが、観測されています。)
 
 同様に超銀河団だったら、質量も大きいし、重力も大きいから、中心部では
周辺部より時間がユックリ進んでいるんでしょうか  ?
 銀河系は観測では、超銀河団の中心にはなさそうなので、中心部を観測する
と、地球から見て、光がユックリ進んでいる現象とか捉えられるんでしょうか ?

それとも、超銀河団位の質量では、時間の遅れなんて観測できないんでしょうか ?
91大鳥居つばめさん ◆dpLvO.T.cY :02/11/02 22:07 ID:???
>>35>>89 のいうように証明を当時よりも覚えてないが)
基本として、
<Sφ、Sψ>=<φ、ψ>:Sがユニタリ
<Hφ、ψ>=<φ、Hψ>:Hがエルミート
HS=SH
92ご冗談でしょう?名無しさん:02/11/02 22:20 ID:???
>>91
上の二つはいいとして、HS=SH・・?
H,Sが>>35の定義ならいいけれど。
93ご冗談でしょう?名無しさん:02/11/02 22:31 ID:???
>>90
なんとなくだけど・・。
質量が大きい、とか重力が大きい、とか言葉が一人歩きしている
気もする。ある地点から別の地点を観測した時に、どういう条件の下に
時間がずれるかを考えてみては?
94ご冗談でしょう?名無しさん:02/11/02 23:00 ID:???
「物質が存在する」とはどういった状態の事ですか?
9590:02/11/02 23:08 ID:zVCmZo5I
93さん、すいません。もう少し考えて見ます。

 それと、光速度不変で、光の速度は変わらないのかな  ?
そうすると、見かけ上、物理法則が、変わってきちゃうし・・・・・
96ご冗談でしょう?名無しさん:02/11/03 02:57 ID:???
>>94
相互作用が可能な状態
97ご冗談でしょう?名無しさん:02/11/03 04:01 ID:???
3点以上電荷が存在した場合の電気力線は等角写像(あるいは他の何か)で
求められるのでしょうか?
98ご冗談でしょう?名無しさん:02/11/03 10:54 ID:???
フツーに電位求めてから勾配を計算すれば?
99ご冗談でしょう?名無しさん:02/11/03 11:01 ID:???
>>98
じゃあ、勾配をもとめてから電気力線を引く方法を教えてください。
100ご冗談でしょう?名無しさん:02/11/03 11:30 ID:???
>>99
電位をパラメタにして等電線をもとめて
その線に垂直に電気力線をひく
101:02/11/03 14:08 ID:WFS6qHJ3
>>77に愛の手を・・気になって寝れません
102ご冗談でしょう?名無しさん:02/11/03 14:11 ID:???
物質毎の電磁波の透過性は何に拠るんですか?透明って何かと。
教員に聞くも放棄・過去ログ検索済み・ググルは工業製品の解説で埋まる。
103ご冗談でしょう?名無しさん:02/11/03 14:15 ID:???
吸収できるフォトンのエネルギーが物質によって異なる。
104103氏に補足:02/11/03 14:41 ID:???
物質を構成する分子や原子は,とびとびのエネルギーレベルしか取れない.
それぞれのレベルの差のエネルギーしか吸収/放出する事が出来ない.
電磁波の波長に対応するエネルギー差が存在しない時は、電磁波は物質を素通りする.

キーワード: エネルギー準位 吸収 電磁波  「黒体放射」も参照.
105ご冗談でしょう?名無しさん:02/11/03 14:54 ID:BGbs6mZ0
今、物理の実験の「超音波測定装置で測った距離と実際はかった距離の誤差」
について、やっているのですが、
実際の距離と超音波測定装置での距離の誤差をだしてグラフをかかなければな
らないのですが、どっちを基準にすればいのですか。
実際の距離が114で、超音波測定装置で測った距離が109の場合はどうなりますか?
106ご冗談でしょう?名無しさん:02/11/03 14:54 ID:BGbs6mZ0
今、物理の実験の「超音波測定装置で測った距離と実際はかった距離の誤差」
について、やっているのですが、
実際の距離と超音波測定装置での距離の誤差をだしてグラフをかかなければな
らないのですが、どっちを基準にすればいのですか。
実際の距離が114で、超音波測定装置で測った距離が109の場合はどうなりますか?
107ご冗談でしょう?名無しさん:02/11/03 15:01 ID:???
実際測った値は複数あるの?
そうなら測定値から平均値と標準偏差出して真値と比較すれば?
108106:02/11/03 15:33 ID:BGbs6mZ0
もしも実際の距離を基準にすると114-109=5で、基準値より5上がったって
いう計算でいいのですか?
109ご冗談でしょう?名無しさん:02/11/03 15:33 ID:???
>>103-104
ありがとうございました。ていうか、もう少し考える癖着けようと思った。
110ご冗談でしょう?名無しさん:02/11/03 15:42 ID:???
>> 108
測定値は1つしか無いのね。それじゃ誤差の評価としては
「超音波計測により、真値に対して(5/114*100)%の精度で測定できた」かな。
上がったって意味無いだろ。
111106:02/11/03 15:48 ID:BGbs6mZ0
グラフを書かないといけなくて誤差のだしかたがわからなくて
例.
 +
 5??
 4??
 3??
 2??
 1??
  ├────────────────
−1??
−2??
−3??
−4??
−5??
112ご冗談でしょう?名無しさん:02/11/03 15:52 ID:???
横縦軸が「何の」グラフを書くの? 測定の変数は何で、何回測ったの?
誤差なんて、特に指示が無ければ、絶対値なり割合なり自分で定義すればいいだろ。
113106:02/11/03 15:53 ID:BGbs6mZ0
何を基準にしてもいいってことですか?
114ご冗談でしょう?名無しさん:02/11/03 15:55 ID:???
何を評価するよう指示されたのか分からないので、何とも言えないのです。
「何のグラフを書け」と言われたのか、>111 の横軸が何かをきぼんぬ。
115:02/11/03 16:07 ID:n4GAGd/A
106を読む限り、ワシなら真値に対する誤差をグラフにするな。

> 実際の距離が114で、超音波測定装置で測った距離が109の場合はどうなりますか
その時は、測定値−真値=−5 だな。
116106:02/11/03 16:08 ID:BGbs6mZ0
詳細をいうと超音波測定器(距離測定用)を使用して、音による距離を測定
しました。
装置は
┌────────┐
│ 発砲スチロ−ル│
│┌┐┌──┐  │
└┘└┘  └──┘

──────────←物(A)さしになっいる。
       ┌┐
       ││←超音波を出す装置
       └┘  
上のような装置を使って超音波を出す装置を使って、端(A)から1cmきざみに
距離を測るというものです。その跡に定規でまた、端(A)から1cmきざみに距
離を測っていく物です。
そしてから、定規で測った距離と超音波で測った距離の誤差をだすというもの
です。グラフの縦は誤差で横は何センチの位置で測ったかの線です。
このグラフは折れ線グラフです。
117107=110=112=114:02/11/03 16:10 ID:???
混乱させてスンマソン。無視してください。旅に出ます。。。
118106:02/11/03 16:14 ID:BGbs6mZ0
犬さんの解答が正しいのですよね〜?
119107=110=112=114=117:02/11/03 16:15 ID:???
そう思う。じゃあな
120106:02/11/03 16:16 ID:BGbs6mZ0
実験書には「実際の距離と超音波測定器での距離との誤差をだす」というものでしたが
121:02/11/03 16:17 ID:n4GAGd/A
犬だが、「どっちを基準にせよ」とかあるなら、符号はどうなるか知らんよ。
122106:02/11/03 16:20 ID:BGbs6mZ0
それは書いてないです
123:02/11/03 16:21 ID:n4GAGd/A
なら真値が基準で良さそうやな。
ただもっと正確に測定誤差を出したいなら、119氏の言ったように標準偏差
まで計算した方がいいな。
124106:02/11/03 16:23 ID:BGbs6mZ0
わかりました。どうもありがとうございました。
最後に質問なんですが真値が基準の場合、−が多くなるのですが何でですか?
125:02/11/03 16:29 ID:n4GAGd/A
測定環境や回数が充分でなくて有意な誤差となってないか、
または新しい物理現象を発見したかのどっちかやな。
前者の場合は、原因を考えてレポートに書くべし。測定方法とか反省して。
後者ならペーパー化して下さい。

学部生?
126106:02/11/03 16:37 ID:BGbs6mZ0
そうです。
127ご冗談でしょう?名無しさん:02/11/03 16:39 ID:???
超音波実験は鋳鋼じゃないか。で、音速の温度評価は大丈夫なの?
128106:02/11/03 18:31 ID:BGbs6mZ0
X線照射の電圧が違うとどうなりますか?
129ご冗談でしょう?名無しさん:02/11/03 19:01 ID:8VAoVjXM
リア工ですが、質問させてください。
メジャー行きを決めた松井選手は中学時代、
軟式のボール(新品)をバッティングで大量に割ったといいます。
ところが、某研究所でプレスを用いて10tの力を加えても
ボールは割れず、伸びるだけでした。
では、なぜボールは割れたのでしょうか?

バッティングでボールに力がかかるのは撃力で、Δtが0に近いので、
ボールが弾性体でなく近似的に(どう表現したらいいかわかりませんが)
堅い物体のようにふるまうからではないかと漏れは思ったのですが、
詳しい方、どうかご教授ください。
130ご冗談でしょう?名無しさん:02/11/03 19:37 ID:+ErLtQPc
BSFTとかVSFTってなんなんですか。
M理論の種類みたいなもんですか。

ど素人ですみません
131中卒塗装工:02/11/03 20:56 ID:???
>>130
あんまりMとは関係ないです。
ボソニック弦のタキオン凝縮を議論する為の道具です。
BSFT = Boundary String Field Theory
VSFT = Vacuum String Field Theory
132ご冗談でしょう?名無しさん:02/11/03 22:44 ID:365pkDTz
金属系の超伝導ですが、ボーズ、アインシュタイン凝縮の中で
導体の材質は Tc のどこに関わってくるのでしょうか?
つまり、 例えばPbのTc=7K, NbのTc=9K
この 7K と 9K の違いは、ボーズ凝縮の中で、どこに出てくるのでしょうか?
133ご冗談でしょう?名無しさん:02/11/03 22:49 ID:???
F=ma という事です。
134ご冗談でしょう?名無しさん:02/11/03 22:51 ID:???
>>132
4-Fermi引力の強さの違い。
135ご冗談でしょう?名無しさん:02/11/03 23:38 ID:nItO8RUf
この板でいいのかどうかわかりませんが、放射能(線)って、生物以外にも
影響があるんでしょうか?例えば、機械とか・・・。原子力施設では特殊な
設備が使われているのでしょうか?素人の素朴な疑問です。
136ご冗談でしょう?名無しさん:02/11/03 23:41 ID:???
>>135
あるね。線によっては電荷持ってるでしょ。
チェルノブイリではロボットが壊れまくって人海戦術に走ってあぼーん。
137ご冗談でしょう?名無しさん:02/11/03 23:47 ID:???
>>102-104から続いて:
あの後考えてたらまた分からなくなりました。
元素が陽子数で定義されてるなら、各電子の受ける静電気力は元素によって
(勿論殻毎に)一定で、だから励起前後のエネルギー差(従って反射できる光の
波長)は同じ元素なら全て同じ、だと思ったんですけど、じゃあ同素体、
例えば黒鉛とダイアの間の差異は何ですか?一瞬電子の位置が違うのかと
思いましたが、電子殻の半径って電子波の波長の整数倍で固定ですよね。
すいません長くて。
138135:02/11/03 23:48 ID:nItO8RUf
即レスありがとうございます。
へ〜、そうなんですか。
チェルノブイリの事故原因の一つにそれがあったとは発見でした。
質問ついででなんなんですが、電荷を持たない放射線は、機械に
影響無しと考えてよいのでしょうか?
139ご冗談でしょう?名無しさん:02/11/03 23:59 ID:a1io6Ero
黒鉛とダイアで違うのは結晶構造の違い。
140ご冗談でしょう?名無しさん:02/11/04 00:06 ID:???
>>138
事故原因ではなくて、飛散した放射性物質の回収時の話。
ガンマ線も強力なら電磁ノイズで電気機器は動かないと思われ。
141ご冗談でしょう?名無しさん:02/11/04 00:18 ID:???
>>137
同位体によって「電子の位置が違う」というのは近い。>>139氏 のおっしゃる通り、結晶構造が
違うと電子軌道の構造が異なり、そうなるとエネルギー準位も異なる。
ダイヤモンドは可視光に対応するエネルギーギャップを持たない。

キーワード: 共有結合 混成軌道 sp2 sp3
142141:02/11/04 00:31 ID:???
同位体→同素体

欝だ士農。。。
143ご冗談でしょう?名無しさん:02/11/04 00:47 ID:???
>>139>>141
度々ありがとうございます。知りたかった事が分かったついでに、
工房でしかも化学を取っていない(文系は理由にしない)俺には今は
これ以上理解できない事も分かりました。勉強してから調べたいと思います。
144ご冗談でしょう?名無しさん:02/11/04 00:51 ID:???

こいつ高校生か、何かすげえな。

            ∧_∧
     ∧_∧  (´<_`  ) 将来何目指してやがるんだろうな。
     ( ´_ゝ`) /   ⌒i  
    /   \     | |
    /    / ̄ ̄ ̄ ̄/ |
  __(__ニつ/  FMV  / .| .|____
      \/       / (u ⊃ 
        ̄ ̄ ̄ ̄ ̄
 
145ご冗談でしょう?名無しさん:02/11/04 01:59 ID:2BpEBQsY
>>137
> 黒鉛とダイアの間の差異

余談だが、バーベキューで使う炭と、鉛筆の芯の部分の結晶構造も違う。
片方は電気を通すし、片方は電気を通さない。
どっちかが体心立方でどっちかが面心立方(だったと思う)。

で、備長炭は例外的に電気を通す。
エジソンは電気を通す炭を探して、日本の竹を使ったそうだ。
146ご冗談でしょう?名無しさん:02/11/04 02:40 ID:???
>>145
>どっちかが体心立方でどっちかが面心立方(だったと思う)。

なわけないでしょ...

炭は今流の言葉づかいならカーボンプレーンとでもいうべきもの。
でもって、そのプレーンが分子間力でくっついている。
147ご冗談でしょう?名無しさん:02/11/04 04:50 ID:???
>>146
カーボンプレーンって、グラフェンシートとはちがうの?
148ご冗談でしょう?名無しさん:02/11/04 05:04 ID:???
>>147
同じ。
149ご冗談でしょう?名無しさん:02/11/04 13:26 ID:???
鉛筆の芯が電気を通さないのは粘土を混ぜてあるため。・・・違うの?
150ご冗談でしょう?名無しさん:02/11/04 14:12 ID:???
>>149
鉛筆の芯は電気通るよ
151ご冗談でしょう?名無しさん:02/11/04 14:21 ID:???
昔実験したな。シャーペンの芯に電流流すの。
152ご冗談でしょう?名無しさん:02/11/04 16:21 ID:???
153ご冗談でしょう?名無しさん:02/11/04 17:13 ID:???
>>138
 確か富士通かどっかで、中性子線がメモリにエラーを起こすことを
実験で確認したとか聞いたとこがあるな(違ったかも)。

 放射線でメモリエラーを起こすってのはメモリチップのメーカでは
よく言われる。放射線が電離を引き起こしてコンデンサの電荷を変えて
しまうから。チップを封止するプラスチックなんかも純度を高めてウラン
とかの含有率を低くしている。
154ご冗談でしょう?名無しさん:02/11/04 21:12 ID:WtZFv9v6
特撮板「空想科学読本VSこんなにヘンだぞ空想科学読本 7」より
http://tv.2ch.net/test/read.cgi/sfx/1034260429/667

>667 名前:名無しより愛をこめて 投稿日:02/10/30 03:43 ID:LPshiijd
> どうも83は根本的に勘違いしているね。例えばa=f/mの左辺の単位はm/s^2だ。
> 一方右辺の単位はN/kgだ。単位があわないようね。これは実は係数が隠れているのだよ。
> 例えばa=k*f/mとすればkの単位は(m/s^2)/(N/kg)だ。kは力と加速度の関係を表す係数だ。
> この値は1なのでこれを略しているのだよ。実際にはこの係数の値が1になると便利なのでそのようにNの単位を決めたわけが。


これってどんなもんでしょ?
これに端を発して、次スレhttp://tv.2ch.net/test/read.cgi/sfx/1036248817/
でもゴチャゴチャやってるんですけど。
155ご冗談でしょう?名無しさん:02/11/04 21:47 ID:zS9KusOG
なるべく早くSi,Au,Alの仕事関数[eV]を教えてくらさい。
156ご冗談でしょう?名無しさん:02/11/04 22:02 ID:???
>154
微妙につっこみたくなる文章だけど
その発言には何もおかしなところはない。
Nをどう定義するかの話だ。
157ご冗談でしょう?名無しさん:02/11/04 22:06 ID:8yp7F2Dt
検流計には復元力、抵抗力がどのように担われて減衰振動を実現しているのか教えて下さい。
158ご冗談でしょう?名無しさん:02/11/04 22:21 ID:???
>>156
 いやー、NはSI単位系なんだから、やっぱ変でしょう。
159ご冗談でしょう?名無しさん:02/11/04 22:56 ID:05Wu8nQf
地球の半径ってどうやって求められるんですか?
簡単な公式で求めることは可能?
160ご冗談でしょう?名無しさん:02/11/04 23:11 ID:???
>>159
エラトステネスで検索してみ。
161ご冗談でしょう?名無しさん:02/11/04 23:16 ID:???
朝日新聞がヘリから撮影した写真を「ブロッケン現象」であると
解説した記事について、「これは単なる虹ではないか」として、
ちょっとした論争になっています。

http://news2.2ch.net/test/read.cgi/newsplus/1036390329/

誰か詳しい方がいらっしゃいましたら、上記のスレに遊びに来て
くださいです・・・。
162161:02/11/04 23:18 ID:???
問題になっている朝日新聞の記事へのリンクも貼っておきます。
http://www.asahi.com/national/update/1104/007.html
163ご冗談でしょう?名無しさん:02/11/04 23:48 ID:???
>>159
重力加速度を測れば求まるんじゃないの?
F = GmM/r^2
ただし、遠心力も考慮して。
Mも分からなければ、標高や緯度の異なる2点で測る。
164ご冗談でしょう?名無しさん:02/11/05 00:03 ID:???
>158
だから、そーなるように定義したって話でしょ。
165ご冗談でしょう?名無しさん:02/11/05 01:27 ID:aNBEGtX4
安価で5x10x2cmくらいもしくはそれ以上の大きさがある
誘電率の高いものってなんですか?
今のところ消しゴムが候補なんですが…
166ご冗談でしょう?名無しさん:02/11/05 01:38 ID:???
>>165
材料物性板あたりで聞いてみては?
167ご冗談でしょう?名無しさん:02/11/05 09:25 ID:???
http://www.amazon.co.jp/exec/obidos/ASIN/4621027271/qid=1036455687/sr=1-5/ref=sr_1_2_5/250-0048962-8519432
↑の書籍データを見てよ。
新しい版はマンガになったの?
168総帥降臨:02/11/05 11:03 ID:sltA0v4t
学部卒と修士卒と博士卒ではどれが就職はいいのでしょうか?
研究職につくきはなく普通の企業に就職しやすいのはどれでしょうか?
169よっちゃん:02/11/05 15:33 ID:???
よく大学受験勉強していると
ニュートンリングの問題はすごく難しい
って言うけど何が難しいの?
170ご冗談でしょう?名無しさん:02/11/05 18:41 ID:rDv0jfkY
物理学と数学の根本的な違いが分からない。
数学は物理学に包括されているという考え方は正しいのか?

物理学者は数学者とも見れるが、その逆は無いと。
171ご冗談でしょう?名無しさん:02/11/05 18:50 ID:???
>>169
さぁ・・・そもそも、ニュートンリングの問題ってなんだっけ。
受験生のほうが詳しいんじゃないの
172ご冗談でしょう?名無しさん:02/11/05 19:22 ID:ebXXOwCB
2本の直線の傾きm1とm2の積がマイナス1になると
どうして2直線は直交するのですか?
教科書にも証明がありません。なぜマイナス1などという
半端(?)な数が出てくるんでしょう?
簡単で直感的な説明って出来ますか?
173ご冗談でしょう?名無しさん:02/11/05 19:56 ID:???
174ご冗談でしょう?名無しさん:02/11/05 20:02 ID:c9INcwGr
>>171
凸レンズに光をぶち込んで反射光とかが重なったところが強調されて縞々になる奴では?
光路差が整数か分数かで強弱が分かれたような分かれないような・・・

ところで熱力学でP=1000kPa V=10dm^3の理想気体を等温で外圧を500kPa一定に保持して膨張させたときの
qとwは分かるんだけど(多分)可逆的に500kPaまで膨張させたときはどうなるの?
175172:02/11/05 20:16 ID:DAAvGqmI
>>173さんすいません、それでもよくわかりません。
もう少し詳しくお願いします。
176ご冗談でしょう?名無しさん:02/11/05 20:31 ID:c9INcwGr
y=xのグラフを想像する・・・それと直行するグラフを想像する
そのグラフの式は?・・・y=-x
ほれ傾きの積は-1じゃん
177ご冗談でしょう?名無しさん:02/11/05 20:37 ID:dM4tdygd
着替え生中継カメラ只今無料!!!
(,,゚Д゚) http://www.cosmani.tv/gallery/gallery_frame.html
178ご冗談でしょう?名無しさん:02/11/05 20:45 ID:???
>>172
自分で一般化して計算してみればいいじゃん。
179172:02/11/05 21:15 ID:Jd/ow2Yn
どうやって一般化していいのかわからないのです。
具体的な関数を考えれば確かにそうなってはいますが。
180ご冗談でしょう?名無しさん:02/11/05 21:17 ID:???
傾きが「c」→x軸に1ずれたら、y軸にcずれる。

この線を90度回転、、、、「文章の中の x とyを入れ替えたもの」

で、-c × 1/c になる。
181大鳥居つばめさん ◆dpLvO.T.cY :02/11/05 21:28 ID:wzcf5dP3
計算でなら、直交するベクトルの内積から「−1」が導ける。
182ご冗談でしょう?名無しさん:02/11/05 21:29 ID:59/d76Od
次のベクトル場Aをxy平面上で図示し、その流線を求めてほしいです。i,jは単位ベクトルです。
(1)A=xi+yj,(2)A=-yi+xj
183172:02/11/05 21:58 ID:GRMQg7LJ
最後のーC*1/C がどこから出てきたのかわかりません。
184ご冗談でしょう?名無しさん:02/11/05 22:08 ID:???
y=axに直交するグラフをy=bxとする。(切片はy軸方向に平行移動するだけ
なので0とする)。 この2つのグラフはそれぞれ (1,a)と(1,b)を通る。
当然原点は通るのでこの2つのベクトル(1,a)と(1,b)が直交すればよい。

直交条件は 内積=0より 1+ab=0
よってb=- 1/a q.e.d

185ご冗談でしょう?名無しさん:02/11/05 22:19 ID:???
>>172
 傾きをtanθとすれば、
tan(θ+π/2) = sin(θ+π/2)/cos(θ+π/2) = cosθ/(-sinθ)=-1/tanθ
 これでどないだ?
186172:02/11/05 22:47 ID:0oSXibAN
おぉ!わかりました!
ありがとうございます!
187工房です。:02/11/05 22:55 ID:Muj0arl6
工業高校3年です。
交流回路の抵抗について考えたのですがどうも・・
交流電源とCだけの回路です。
V=sin(wt)とすると、
i=dq/dt、q=cvより、
i=c*dv/dt = cwcos(wt)

抵抗R=dv/diとすれば
dv/di=(dv/dt)(dt/di)=(dv/dt)(1/(di/dt))
=-cos(wt) / cwsin(wt)
となってしまい・・
目的の1/cwが得られないのです。
188ご冗談でしょう?名無しさん:02/11/05 23:14 ID:???
>>172
>教科書にも証明がありません
一体どんな教科書だ?
189ご冗談でしょう?名無しさん:02/11/05 23:21 ID:???
(練習1)とかになってる悪寒
190:02/11/05 23:29 ID:rWJA8pwc
地球上で真空の空間を作り、その中で2つの同じ物質で同じ質量の小球を同じ支点からそれぞれを糸で垂らして、仮に2つの小球をA、Bとおくと、Aを糸がたるまないように引き上げて初速度0でBにぶつけると2つはどんな動きをするのですか?
真空ではない状態では実験ができるのですが、真空ではどうなるのかわかりません。教えてください、おねがいします。
191ご冗談でしょう?名無しさん:02/11/05 23:33 ID:E5yxR66A
>>190
空気の抵抗がないだけで、大して変化ないのでは?
192ご冗談でしょう?名無しさん:02/11/05 23:38 ID:???
運動量が保存するから衝突されたBはAの最初の高さまで上がり、
戻ってきたBがAに衝突するとAが再び同じ高さまで上がり…
を繰り返すんでないの?

昔懐かしいアメリカンクラッカーだな。
193ご冗談でしょう?名無しさん:02/11/05 23:40 ID:???
摩擦がないとして運動方程式を解けばいいと思うけど。
基準点からの高さをhとでもすれば v=√(2gh)っていう速さでBと衝突
するんでしょ。あと2つの小球の間のはねかえり係数をeとでもして
運動量保存則使ってよ。
194ご冗談でしょう?名無しさん:02/11/05 23:48 ID:???
エネルギーは糸を伝わって外部に逃げるし、
どんどん熱エネルギーへ変化していく。

空気抵抗がなくなっても振動の寿命が少し延びるだけだと思うぞ。
195ご冗談でしょう?名無しさん:02/11/05 23:50 ID:E5yxR66A
>>192
永久機関完成でつか?
196ご冗談でしょう?名無しさん:02/11/05 23:51 ID:???
>>195
あほが。仕事してねーよ。
197ご冗談でしょう?名無しさん:02/11/05 23:53 ID:E5yxR66A
>>196
衝突の際に熱は発生せませんか?
198ご冗談でしょう?名無しさん:02/11/05 23:55 ID:???
弾性衝突っていう理想的状況をいっているだけでしょ。
199ご冗談でしょう?名無しさん:02/11/05 23:55 ID:???
運動量保存すんだろ? どうやってエネルギー減るんだよ。
200ご冗談でしょう?名無しさん:02/11/05 23:56 ID:xucRiMql
問題の設定が不十分だな。
201ご冗談でしょう?名無しさん:02/11/05 23:57 ID:???
>>199
運動量保存とエネルギー保存は別でしょ。高校物理ではねかえり定数
ってなんのために導入したのさ?
202ご冗談でしょう?名無しさん:02/11/06 00:00 ID:T9TyGnjo
内力しか働かない時は運動量保存。
簡単のため同じ質量の2つの球を考えて。衝突前後で速度が一定ということは?
203ご冗談でしょう?名無しさん:02/11/06 00:00 ID:???
だからエネルギー一定だろ。
204ご冗談でしょう?名無しさん:02/11/06 00:03 ID:???
>>202 衝突前後で速度が一定

というか、速度vを持ってる球が入れ替わると思うが。
205ご冗談でしょう?名無しさん:02/11/06 00:04 ID:T9TyGnjo
あ、スマソ。舌足らずでした。
206ご冗談でしょう?名無しさん:02/11/06 00:08 ID:cStQfj4B
ニュートリノって光速に近い速度をもってるの?
もしもってるとしたら、重さも相当なんやから、恐ろしいことにはならんの?
あ〜分からん。
誰か説明して下さい。
207ご冗談でしょう?名無しさん:02/11/06 00:10 ID:???
>>201
非弾性衝突の時は、最初の運動量の一部が壁がもらうが、それは見てない訳だな。
今の場合、壁のような不動の質量の代わりに、ぶつかる先の球も含めて
運動量保存するという事か。
208ご冗談でしょう?名無しさん:02/11/06 00:10 ID:???
>>202
だからそれはエネルギーが保存する状況ではエネルギーが保存する
って言ってるだけでしょ。はねかえり係数が1未満だったらそうはなら
ないからエネルギーは保存しない。でも運動量は保存する。
>>199は運動量が保存すればエネルギーも保存するといっているように
しか聞こえない。
209ご冗談でしょう?名無しさん:02/11/06 00:13 ID:???
>>206
>重さも相当なんやから、恐ろしいことにはならんの

の「相当なんやから」と「恐ろしいこと」ってどういうこと?
210202:02/11/06 00:15 ID:T9TyGnjo
まあ、途中からなんか弾性衝突のハナシになってたからねえ…。
文脈からそう取れないこともない。「問題設定」が悪いんでないの?
211199:02/11/06 00:17 ID:???
>>208
運動量とエネルギーが同じだって言ってる訳ではない。
「アメリカンクラッカーみたいに、速度が移るのなら…」を補完きぼんぬです。
212ご冗談でしょう?名無しさん :02/11/06 00:18 ID:2qbcEUnh
ボルツマン分布とエントロピー最大の法則の関係って何?
213ご冗談でしょう?名無しさん:02/11/06 00:18 ID:???
で、答えとしてはどうなの?
214ご冗談でしょう?名無しさん:02/11/06 00:22 ID:???
>>212
ひとことでいえばエントロピー最大の状態で熱平衡(ボルツマン分布)が実現する。
215ご冗談でしょう?名無しさん:02/11/06 00:26 ID:???
>>213
真空ではない状態と同じ様な運動をより長い時間する。
もちろん、有限の時間。
216ご冗談でしょう?名無しさん:02/11/06 00:35 ID:2qbcEUnh
>214
ありがとう(・∀・)!
もう少し聞くと、エントロピー最大の法則からボルツマン分布を
求めることってできますか(゚Д゚)?
217ブルーバックス:02/11/06 00:39 ID:cmmRIJ1z
 直接物理と関係のないことで恐縮なのですが日本がノーベル賞を受けるたびに
まともに評価しないチョン国にはブルーバックスのような科学の啓蒙書のシリーズ
があるのでしょうか?
218ご冗談でしょう?名無しさん:02/11/06 00:45 ID:???
>>216
できます。エントロピー最大の条件からミクロカノニカルアンサンブルが
さらにそれからカノニカルアンサンブル(ボルツマン統計)が導けるはずです。
219ご冗談でしょう?名無しさん:02/11/06 00:51 ID:2qbcEUnh
>>218
お恥ずかしい質問なのですが、
導出には具体的にどういった経路をたどればいいのですか?
220HAL ◆2eOaL2U.Gg :02/11/06 01:04 ID:???
>>219
極値をとるって条件を課せばいいんだよ。
221206:02/11/06 01:05 ID:cStQfj4B
>>209
光速に近づけば質量も無限大になるやろ?それ。
そしたら恐ろしいことになりそうやないかぁ。と考えてしまいます。
222ご冗談でしょう?名無しさん:02/11/06 01:11 ID:tB32K/W5
>>221
ですから、質量はありません。
223ご冗談でしょう?名無しさん:02/11/06 01:13 ID:???
>>220
おかげさまで少しずつ分かってきました。
統計力学の導入は岩波シリーズの長岡洋介で十分ですか?
224HAL ◆2eOaL2U.Gg :02/11/06 01:17 ID:???
>>223
私は、はじめにパリティの「統計物理」を読んで
熱力学の本を一冊読み次にあなたの言う本を読みました。
読んでてわからないことがあればどうぞ。

225HAL ◆2eOaL2U.Gg :02/11/06 01:20 ID:???
おっと質問に答えてなかった。

その本はよくできてます。統計力学は、基本応用もしっかり
身につくようになってます。(二次の相転移は難しいですが)

ただ、熱力学の記述もありますが、少ないのでほかの本で補ったほうが
いいでしょう。
226ご冗談でしょう?名無しさん:02/11/06 01:27 ID:???
>>224
やっぱりいきなり長岡さんは無謀でしたか...
私は高分子専攻なんですが、
統計力学もそろそろ必要になってきました。
あんまり人に頼りすぎるといけないので
もう少し自分で考えてみます。
ありがとうございました。

227:02/11/06 02:01 ID:ezwQVoTj
えぇ、遅くなりましたが、小球の話で、僕も同じようにだんだん運動エネルギーが熱エネルギーに変わって止まっていくのだと思ったんですが、
教授に言ったら、「じゃあhが大きかったら小球(ここでは銅)は熔けるんやな」と言ってきたんです。こんなことが本当にありえるんですか?
228ご冗談でしょう?名無しさん:02/11/06 02:11 ID:???
hが大きかったら小球は木っ端微塵に砕け散ると思います
229ご冗談でしょう?名無しさん:02/11/06 03:01 ID:???
>>222
うそつくなよ。

>>221
めちゃくちゃ軽いから、ちょっとのエネルギーで
簡単に光速近い速さになってしまうだけ。

ところで、君の思っている質量は相対論的質量というもので
エネルギー(静止質量エネルギー+運動エネルギー)と定数倍
異なるだけで同じもの。今は全然使われてないから
単に質量と書いてそれを使わないようにして。
君も混乱するし、他の人も説明に疲れる。
230ご冗談でしょう?名無しさん:02/11/06 03:34 ID:???
>227
教授?君は大学生か…。
条件しだいではそりゃとけるでしょう。
現実にはまずありえないが。
231206:02/11/06 04:22 ID:cStQfj4B
>>229
質量について教えて下さい。
232ご冗談でしょう?名無しさん:02/11/06 04:39 ID:???
Kittelの熱物理学も一つかと。
233ご冗談でしょう?名無しさん:02/11/06 05:48 ID:???
すみません。未来技術版の住人です。思いつきで人工重力を書いたら説明不足や
ら勘違いやらで一番は勉強不足で基地外扱いされる始末でお尋ねに来ました。

潮汐効果を利用して、電話ボックスの中を地球の重力に逆らって無重力にしよう
と思いました。超高速回転するディスク(質量があればなんでもいいですよ
ね。)を使用するとして、
電話ボックスを中心とした円を設定し、ディスクの回転軸をその円の円周が
貫通するようにディスクを設置します。ディスクが電気のこぎりだったら、
電話ボックスを縦に切るようなつもりでディスクを設置する形です。(切る直
前ですが)電話ボックスをぐるりと取り囲むように複数のディスクを設置し、
全てのディスクを電話ボックスから見て上向きに回転させます。

縦から見た図(○がディスク)
↓○↑[電話ボックス]↑○↓

上から見た図(□が電話ボックス |と―がディスク)
上から見ると電話ボックスと反対向きに回転させます。
      |(↑)
(←)― □ ―(→)
      |(↓)

衛星などは、惑星の自転運動のエネルギーをもらって公転半径が大きくなる
事がありますが(月の様に)それを応用したつもりです。これで地上の
重力に逆らった運動エネルギーを電話ボックスに与えられるでしょうか?
(勿論、現実にはそんな回転に耐える材質はないので、実現性は度外視
ですが)
どうかお願いいたします。
234デブ・ヲタ・ヒッキー:02/11/06 06:10 ID:???
>衛星などは、惑星の自転運動のエネルギーをもらって公転半径が大きくなる
>事がありますが(月の様に)それを応用したつもりです。

それとボックスの中に上向きの重力がかかる事の関係が全く不明。

回転する質量による時空の引きずり効果を狙ったなら、莫大な質量が必要。

もし、そんなモノが用意出来るなら、何も回転させなくても
ボックスの上に置くだけで、ボックス内に局所的に無重力が出来る。
235233:02/11/06 06:15 ID:???
>>234
早速のレスありがとう御座います。

衛星の軌道が自転によって変化するのは時空の引きずり効果によるものですか?
236ご冗談でしょう?名無しさん:02/11/06 06:15 ID:???
>>233
よくわからんが、ディスクは電話ボックスに接してるの? だったら電話ボックスを
ボールに置き換えればただの「バッティングマシーン」じゃん
237233:02/11/06 06:17 ID:???
>>236
いいえ、接していません。潮汐効果を狙った物です。
238デブ・ヲタ・ヒッキー:02/11/06 06:20 ID:???
>>衛星の軌道が自転によって変化するのは時空の引きずり効果によるものですか?

ちがう。

ついでに言っとくと、ディスクが一様なら、回転による潮汐力は発生しない。
質点の回転と、一様ディスクの回転は違う。
239デブ・ヲタ・ヒッキー:02/11/06 06:24 ID:???
さらに言っておくと、ニュートン重力だけの効果なら
ボックスの横でどんなモノを回転させても
ボックス内に上向きの潮汐力の様なものは発生しない。
240236:02/11/06 06:29 ID:???
やっとわかった。

浮かばせたい物がディスクを潮汐力で歪ませられるほど強い重力があれば浮くかもね。
でもそんな強い重力があるなら、ボックスの上にディスクを置けば浮かぶでしょ。
潮汐力の場合、ディスクが歪むことによる質量差しか上向きの力に寄与しないけど、
ボックスの上にディスクを置けば、ディスクの質量が丸々上向きの力に寄与するんだから
241233:02/11/06 06:41 ID:???
>>240
えーと、すみません。ディスクが重ければ言い訳で、潮汐力で可能になる場合
潮汐力が計算に入らないほど大きな重力をディスクが持っているのですね?
242デブ・ヲタ・ヒッキー:02/11/06 06:50 ID:???
>>241
240も言ってるように、そんな重いディスクがあるのならボックスの上に置けばよいだけ。

そもそも、潮汐力てのは単に重力が場所によって違う事により
見かけ上現れる力でしかない。重力が常に引力である事に変わりはない。

計算に入るとか入らないとかそういう性質の物ではない。
衛星の公転軌道がずれるのは、重力が小さくなったからではない。
極めて長期間に少しづつ衛星が運動エネルギを自転エネルギから得ただけ。
243233:02/11/06 07:06 ID:???
>極めて長期間に少しづつ衛星が運動エネルギを自転エネルギから得ただけ。

この現象は衛星の質量が大きい事と、アンバランスな重力を受けつづけた事に
よる物ですか?つまり安定した潮汐はありえない?
244デブ・ヲタ・ヒッキー:02/11/06 07:08 ID:???
ボックスの横で何かを回転させてる場合、潮汐力を経由して
回転エネルギがボックス内の物体に伝わる事はある。

しかし、重力が常に引力であるという事実は変え難く、
ボックス内の物体の上向きの運動エネルギーに変換される事はあり得ない。
大抵の場合、物体の熱エネルギーか回転のエネルギーに変換されるだけだろう。
245233:02/11/06 07:17 ID:???
馬鹿な質問を繰り返してすみません。
>>大抵の場合、物体の熱エネルギーか回転のエネルギーに変換されるだけだろう。
上向きの力が働いても、ほとんどあるいは全てが、ボックスを変形させるような
(満ち潮のような)などの力に変わってしまうのでしょうか?

それと
バランスをとって上向きにさせることは不可能という事でいいのでしょうか?
246233:02/11/06 07:20 ID:???
>>244
>ボックスを変形させるような
あと回転などのエネルギーでした。
247デブ・ヲタ・ヒッキー:02/11/06 07:20 ID:???
一瞬、上向きの力が働く事はあるだろう。ただ同様に下向きの力も働き、
トータルで継続した上向きの力がかかる事はない。

結論: 233のシステムではボックス内を無重力にすることは原理的に不可能。
248ご冗談でしょう?名無しさん:02/11/06 07:23 ID:1aA90kqL
実験で光速に近づくと時間の流れがゆっくりになる結果が出ているらしいけど本当なの?
光速に近いからではなく、光速に近づけるための加速の影響の可能性が有るのでは?
249デブ・ヲタ・ヒッキー:02/11/06 07:26 ID:???
>実験で光速に近づくと時間の流れがゆっくりになる結果が出ているらしいけど本当なの?
本当です。相対論では<<同時>>の定義は観測者による。

>光速に近いからではなく、光速に近づけるための加速の影響の可能性が有るのでは?
加速度の影響も当然あります。
250233:02/11/06 07:30 ID:???
だんだん分かって来ました。
潮汐を経由して回転のエネルギーを伝えても、そのままではなく重力を介して
いるので、常に引力として働くため上下に引き伸ばすなどのエネルギーとして
伝わる…でいいでしょうか?
たとえば、モーターが斥力と引力によって力を伝えてるが、この場合引力だけ
なので両方に引っ張られる…という感じでは?
251デブ・ヲタ・ヒッキー:02/11/06 07:33 ID:???
>>248
ミュウ粒子の崩壊とかのキーワードで探せば、色々見つかるだろう。

本当の意味で時間の進み方を比較するには、
同じ時空点で相対速度ゼロで時間を比較する必要がある。
この場合、加速度を考慮する事は必須となる。

>>249
だいたい良い。
252248:02/11/06 07:37 ID:1aA90kqL
なるほど、すでに両方とも影響が有る事が確認されていたのですか。
有り難うございます。
253236:02/11/06 07:43 ID:???
>>241
>ディスクが重ければ言い訳で
違う。

月がだんだん遠ざかっているのは、月が地球を歪ませて、その歪んだ質量分布による
重力が月を引っ張っているから、というのはOK?

これをキミの問題に焼き直せば、月が浮かばせたい物体、地球がディスクに当たるんだよね?
だったら、浮かばせたい物体の重力が充分強くてディスクを歪ませないといけない。

ディスクが回転していなければ、ディスクは浮かばせたい物体の方向に引き延ばされるね。
で、そのディスクをうまく回転させれば、引き延ばされる方向が多少上向きにずれて安定
することはあるかもしれない。そのとき多少上向きにずれた質量分布でできる重力が
上向きの力を生む可能性はある。

しかしずれた質量分布で上向きの力を与えるくらい重力が強いのなら、ディスクを上に
置いて、質量を丸々上向きの力に使えばらくらく上がるでしょ。
254ご冗談でしょう?名無しさん:02/11/06 07:47 ID:???
>>248
生まれたときから光速に近い(したがって加速は0)の粒子でも
寿命の伸びは観測されています

255233:02/11/06 07:47 ID:???
>>251
いろいろとお世話になりました。「だいたい」わかりました。「ちゃんと」理解
するよう、これからもっと突っ込んで勉強したいと思います。
(まずは微積分と力学から。)
256233:02/11/06 08:03 ID:???
>>253
>月がだんだん遠ざかっているのは、月が地球を歪ませて、その歪んだ質量分布による
>重力が月を引っ張っているから、というのはOK?
はい。分かります。

ボックスの方が強い重力を持っていて、ディスクの方は計算して回転させなけ
ればならないわけですね。
つまり、自らの重力で位置エネルギーを生み出すような物…
という事でいいでしょうか?
257233:02/11/06 08:22 ID:???
>>253
自分の重力により自分を引っ張る潮汐力を生み出すわけだから少しずつしか
エネルギーを取り出せないわけですね。

申し訳ありませんがあとは0時ごろからでないとレス出来ません。
今後の返答に対してはその後返答いたします。
皆様、いろいろと有難う御座いました。誤解が解けました。
258:02/11/06 09:29 ID:9T+uq6pV
いろいろありがとうございますた
259ご冗談でしょう?名無しさん:02/11/06 09:40 ID:tB32K/W5
小学生の問題なのですが、球をある高さから転がしたときの球の飛距離
(スキーのジャンプのようなもの)で
高さ  飛距離
10p→10p
20p→14p
40p→20p
60p→25p
80p→28p

のとき
160pでは何pになるかという問題で、答えは40pになるのですが、
理由はいまいち分かりません。分かる方がいればおねがいします。
260ご冗談でしょう?名無しさん:02/11/06 10:06 ID:tB32K/W5
>>259
自分で分かりました。
261ご冗談でしょう?名無しさん:02/11/06 10:12 ID:???
60cmは24cmでないかえ?
262ご冗談でしょう?名無しさん:02/11/06 10:15 ID:EjSFCeMp
   /)  /)
  /  ⌒  ヽ   / ̄ ̄ ̄ ̄ ̄ ̄ ̄ ̄ ̄ ̄ ̄ ̄
  | ●_ ●  | < 必殺のぽこちんフラッシュが毎回カットされてるのはなぜ?
 (〇 〜  〇 |  \____________
 /       |
 |     |_/ |

263ご冗談でしょう?名無しさん:02/11/06 11:14 ID:ggMFGPKR
なんで、鉄損て起こるの?
264ご冗談でしょう?名無しさん:02/11/06 11:22 ID:???
皆がマイカーにのるからです。
もっと電車に乗ってください
265ご冗談でしょう?名無しさん:02/11/06 11:48 ID:pZyeZqVm
これから銀座にデートに行きます。良いですか?
266ご冗談でしょう?名無しさん:02/11/06 11:55 ID:???
>>265
禁止
267ご冗談でしょう?名無しさん:02/11/06 14:20 ID:???
>>206
m=m。/√{1-(v/c)²}
v≒c
m。≒0
m=妥当な値

念のため言っておくが、現実に測定されるのはmとvで、それからm。が逆算される。
だから、vがどれだけcに近くても、mがおかしな値になることはない。
268ご冗談でしょう?名無しさん:02/11/06 14:30 ID:???
>>261
丸めの結果かな。
だとすると、(l/10cm)=k√(h/10cm)の係数は
k=1ではなく1.0002<k<1.025だな。
「40cmまたは41cm」が正解だ(w
269ご冗談でしょう?名無しさん:02/11/06 14:42 ID:???
>>250
君、基本的に潮汐力ってモノを理解してないのでは?
そもそも、じっとしてたら潮汐力は働かない。
270ご冗談でしょう?名無しさん:02/11/06 15:13 ID:???
>>267
よりによって相対論的質量使って説明するなよ。
271ご冗談でしょう?名無しさん:02/11/06 15:16 ID:+ZEkQFdF
<α|a|φ>=<0|exp[-(α*α/2)+α*a]a|φ>=((d/dα)+(1/2)*α)vac|exp[-(α*α/2)+α*a]|φ>
に出てくるvacってなんですか?
ちなみに
<α|はコヒーレント状態ベクトル
aは作用素
|φ>はスクイズド状態ベクトル
です
272よっちゃん:02/11/06 15:21 ID:JpXcI+gJ
ブラックホールの事をダークマタ−って言うのですか?
273ご冗談でしょう?名無しさん:02/11/06 15:23 ID:???
>>272
ダークマターの候補の一つってだけ
274ご冗談でしょう?名無しさん:02/11/06 16:57 ID:???
>>269
>じっとしてたら潮汐力は働かない。
んなこたーない。有限の大きさを持った物体では、外部の重力が物体の場所に
よって差が出る、その差が潮汐力であって、物体が動いている必要はない。

275大鳥居つばめさん ◆dpLvO.T.cY :02/11/06 18:11 ID:J4Fzx4Da
>>187
抵抗Rは、
R*時間平均(i^2)=時間平均(v^2)
R*最大値(i^2)=最大値(v^2)
などを満たすんではなかろうか。
一般には、オームの法則は Zi=v (各変数は複素数)で、R=|Z| となる。
276大鳥居つばめさん ◆dpLvO.T.cY :02/11/06 18:16 ID:???
訂正です。
R*√時間平均(i^2)=√時間平均(v^2)
R*最大値(i)=最大値(v)
でした。ついでに複素数では V=sin(wt)に対応してv=e^(iwt)なんかになる。
277ご冗談でしょう?名無しさん:02/11/06 19:02 ID:???
>>276
お前なぁ、仮にも物理学科卒なら工房に釣られて次元の合わない式書いてんじゃねぇ
278工房です。:02/11/06 20:56 ID:jcprjJXt
>>277べつにつろうなんて思ってませんよ。

>>276
ちょっと理解不能です・・

V=exp(iwt)ってなんでしょうか??
279ご冗談でしょう?名無しさん:02/11/06 20:59 ID:???
>>278
そいつに関わるな。本物のキチガイだ。
280ご冗談でしょう?名無しさん:02/11/06 21:13 ID:???
>>274
んなこたーない。有限の大きさを持った物体では、外部の重力が物体の場所に
よって差が出る、そしてその力を重心の自由落下運動に伴う加速度系で
あらわそうとすると慣性力として潮汐力と呼ばれる力が見えてくる。
等速直線運動していれば慣性力を持ち出す必要はないだろ。
まあ言葉の使い方の問題だが。
281大鳥居つばめさん ◆dpLvO.T.cY :02/11/06 21:33 ID:???
>>278
>>279のように言うが、>>277は、v=e^(iwt)を正しくはv=v0e^(iwt)とすべきというだけで、
>>276は合ってるんだけど、理論的には、
コンデンサーなどの線形素子による応答は、線形微分方程式で表されるので、
解として、実数関数のsin(wt),cos(wt)をとるならば、代わりに、
それらの線形結合で独立なexp(iwt),exp(-iwt)をとってよい。
となるでし。(勉強の過程で教科書で見る話)
282ご冗談でしょう?名無しさん:02/11/06 21:41 ID:???
ホントに答える気があるならもうちょっと分かりやすく書いてやれよ。
わざわざ高校ではでてこない式と言葉使って混乱させてるだけじゃん。

>>187はがいう抵抗ってのが文字通りの抵抗なのかインピーダンスを
指すのかは確かに分からんが。
283ゲート:02/11/06 21:42 ID:arUCyVtl
何処のジャンル&板で書き込もうか悩んだのですが、ここにしました。

量子ゲートについて詳しい参考書は何かないでしょうか(日本語で)
特にFTMゲートに関して詳しく解説しているものを希望します。
とりあえず皆様のご意見お願いします
284工房です。:02/11/06 22:14 ID:jcprjJXt
>>281
がんばって答えてくれているようですが・・意味不明です・・
線形微分方程式・・??
大学いったら詳しくやるのでしょうか??

>>282
インピーダンスです。
>>187でR=dv/diを計算すると、インピーダンスがtによってぐるぐる変わる。という理解でいいのでしょうか?
でも、無限大になったり0になったりよくわからんです。
電子基礎などの教科書に載っている1/wC,wLというのはどうやったらでるのでしょうか?
285工房です。:02/11/06 22:15 ID:jcprjJXt
そもそも、
R=dv/diとおいたのが間違いでしょうか?
286ご冗談でしょう?名無しさん:02/11/06 22:44 ID:???
>>280
自由落下系で見ると物体全体にかかる重力の平均が0になるので
わざわざ差をとらなくてもいい、というだけで、潮汐力を記述する
ためには必ず自由落下系で見なくてはならない、ということはない。

例えば重力とちょうど釣り合う噴射力のロケットでブラックホール
上空に静止したとすると、自由落下系ではないので潮汐力はないかというと
そんなことはない。もちろん、重力そのものが大きいので潮汐力は
目立たなくなってしまうけど消えるわけではない。
287ご冗談でしょう?名無しさん:02/11/06 22:44 ID:E4tdwGxW
>>285
コンデンサと抵抗を直列につないだ回路(いわゆるCR回路)は解けるかな?
解けるならそれを解いて抵抗R→0の極限をとってごらん。無限大が生じる
理由がわかると思うよ。
288ご冗談でしょう?名無しさん:02/11/06 23:09 ID:E4tdwGxW
ごめん>>287は嘘じゃないけどまわりくどいことやってるな。
電源の電圧をV(t)=V_0sin(wt)とする。時刻tでコンデンサにたまっている
電荷をQ(t)とすればここに生じる電位差はQ(t)/C
したがってQ(t)/C=V_0sin(wt)。回路を流れる電流I(t)は
I(t)=dQ(t)/dtだから上の式をtで微分して I(t)/C=v_0w cos(wt)
任意の時刻tで上の式が成り立つためには I(t)=I_0 cos(wt)
ただしI_0=C V_0 w
インピーダンスは v_0/I_0=1/Cw
289工房です。:02/11/06 23:17 ID:jcprjJXt
>>287
是は学校でやりました。
V=E-exp(-t/CR)/Cですね。
R→0
とすると、t→無限と同じなのでV=Eですね・・

>>288
つまり、電流と電圧の振幅の比がインピーダンスってことですか?
>>任意の時刻tで上野式が成り立つためには。
ってところがわかりません。
任意の時刻tで成り立つためとありますが、これは式からすでに任意の時刻でなりたつといっているのではないでしょうか?
290ご冗談でしょう?名無しさん:02/11/06 23:35 ID:E4tdwGxW
まずV=E-exp(-t/CR)/Cというのは多分直流に対するCR回路ですね。

交流のCR回路は解いてみれば分かるけど電圧=V_0 sin(wt)に対して
電流は I_0 sin(wt+α)というように位相がずれるんですね。
抵抗0の極限でα=π/2が示せて個人的にエレガントだと思っただけな
んで無視してください。

つまり電圧がsinに対して電流がcosで変化するんで直球勝負で比を
取ったら無限大になる場合があるのは当たり前ですね。だから振幅
の比をとった「抵抗もどき」をインピーダンスと呼ぶわけですが高校の
本ではなんて書いてあるんだろう?

任意のt云々は僕がこういう恒等式的な見方が好きだというだけ。趣味です。
291ご冗談でしょう?名無しさん:02/11/06 23:42 ID:???
まあ何百回と交わされた会話だとは思いますが、divやcurlの物理的
意味合いがファインマンを隅から隅まで読んでもわかりません。
gradだったらようは場の微分だからわかるんですけど。
発散とか回転と言われても。
292工房です。:02/11/06 23:58 ID:jcprjJXt
>>290
直列を直流とみまちがえてしまった・・・・すまんです。

教科書には、ただ、こうなるよ。ってかいてあるだけです。
抵抗もどきをインピーダンスですか・・
なんかインチキくさい気がしますね。
R=dv/diで定義されているなら(ホントは違うのか?)、直球勝負でも勝負できないのかと思いまして。
電流と電圧のグラフを書くと、あるところでは、電流が増えながら電圧が下がる、いわゆる負性抵抗だし、
またあるところでは0になったり無限大になったり。
そもそも抵抗(交流も含む)とは厳密にはどう定義されているのかと疑問に思いまして。
293ご冗談でしょう?名無しさん:02/11/06 23:59 ID:???
大鳥居の中でまた何かがはじけたようです。
http://science.2ch.net/test/read.cgi/sci/1030391037/897

こんな奴だってことは一応知ったうえで相手すれ>>278

294ご冗談でしょう?名無しさん:02/11/07 00:11 ID:???
>>292
>R=dv/diで定義されているなら(ホントは違うのか?)
違いますね。

複素表示ならば
V=Vo exp(iωt)とすると
I=CdV/dt=iωC Vo exp(iωt)で
インピーダンスはZ=V/I=1/iωCとなる。その大きさ(絶対値)が1/ωC。
というわけで何の無理もなく出て来る。

これを実数表示のまま記述しようとするとどうしてもこじつけが出て、
インチキくさいという印象を与えてしまう
295工房です。:02/11/07 00:21 ID:82zwDbpQ
exp(iwt)ってのはなんですか?
sinwtではないのですか?
296ご冗談でしょう?名無しさん:02/11/07 00:22 ID:Nq/0lMjh
抵抗Rならば V(t)=I(t) Rで例えば電圧がsinならば電流もsinですね。
コンデンサの場合、そうじゃなくて電圧sinに対して電流cosということは
いわば電圧に対する電流の応答がずれているんですよ。(このずれて
いるということに応用的な価値があるわけですね)。振幅の比を取る
ということはいわばこの応答のずれを考慮した上で電流と電圧の比
を取っている、と思うことはできませんか?
297工房です。:02/11/07 00:28 ID:82zwDbpQ
>>296 >>応答のずれを考慮した。

これ、おもいました。
マクロ的?定常的?まあ直流的にみるとそのまま振幅の比(このばあいは実行値の比かな?)
になるのかな?
と、でも瞬時値でみたらどうなるのかと。
298ご冗談でしょう?名無しさん:02/11/07 00:32 ID:Nq/0lMjh
つまり電流と電圧の比を直球勝負でとったら無限大が生じるというという
のは応答が即座に生じるという電気抵抗の見方をそのままコンデンサ
にあてはめてしまっているわけですね。コンデンサはそうじゃなくて電圧
が「おーい」といっても電流が即座に「おーい」と返答してくれないんです
ね。やまびこみたいなもんです。
299みとにくに:02/11/07 00:35 ID:jY3w2RPL
2つの完全導体球があって、それぞれ半径a,b a<b
両球体の中心間の距離をLとする。 このとき、両球体間の抵抗Rをもとめよ。ただし抵抗率ρとする。

この問題を電流密度から電場だして、計算してください。
全然うまいこといかない、、、、電磁気難しすぎる。。。
300HAL ◆2eOaL2U.Gg :02/11/07 00:36 ID:???
>>295
exp(iX)=cosX+i sinX
どす。でも工房は知る必要ないな
301HAL ◆2eOaL2U.Gg :02/11/07 00:43 ID:???
>>299
オームの法則をつかいましょう。

j=σEですな。シグマは電気伝導度で、抵抗率の逆数。つまり、ρσ=1

仮に完全導体間にIの電流が流れたとしましょう。
すると、電流が定常とすればj(r)が分かる
(ヒント: j(r) ・ 4πr^2 dr=一定を証明せよ。またその一定値は?)

j(r)が分かれば電場E(r)が分かる。電場E(r)が分かれば極版間の
電位差Vがわかる。IとVの関係が分かれば・・・





302みとにくに:02/11/07 00:49 ID:jY3w2RPL
>>300
そう考えたんです、うまくいかんのです。。。

電流密度の強さは、球体(r=a)の強さから球体(r=b)の強さを引くのでしょうか?
もしr=aのほうの中心を原点とした場合、、、
これがよくわからんのです。囲う面積を両方からとるのか、片方なのかの区別がいつもつかないんです。
303みとにくに:02/11/07 00:53 ID:jY3w2RPL
あ、、、どっちかが電位高いほうと決めて 高いほうの球体を囲えばいいの?
面積分のことです。
304ご冗談でしょう?名無しさん:02/11/07 01:13 ID:IiJouPkU
衝撃検流計について調べているんですが、google等で検索してもあまりいいページが見つかりません。
図書館で載ってそうな本を探してみたんですが、見つけることができませんでした。
どなたか、衝撃検流計について書かれている本をご存じないですか?
どうかよろしくお願いします。
305ご冗談でしょう?名無しさん:02/11/07 01:49 ID:rUsgituO
>>299
半径の大小があるって事はbの中にaがあって、中心がズレテルノ?
306HAL ◆2eOaL2U.Gg :02/11/07 01:58 ID:???
>>302
すまん。中心がずれてるなら球対称がやぶれるからむずいね。

わるい。さっきのほうは無理です。
307ご冗談でしょう?名無しさん:02/11/07 02:09 ID:???
俺はbとaは完全に離れている(一方がもう一方に含まれていない)
と思ってた・・。
308HAL ◆2eOaL2U.Gg :02/11/07 02:19 ID:???
>>307
なるほどそういうことか。・・・全然日本語読み取れてなかった鬱だ子嚢
309307:02/11/07 02:28 ID:???
>>308
いや、どうにでも読み取れそうで・・。
310ご冗談でしょう?名無しさん:02/11/07 07:30 ID:F2UGkHO+
>R=dv/diで定義されているなら(ホントは違うのか?)
線形抵抗ならわざわざそう置かなくてもいいじゃない。
R=v/iで。
トランジスタとか解析するわけじゃないんでしょ?
311大学生名無し:02/11/07 15:37 ID:h/DKXU5w
ちょっと、した質問じゃなくてごめんなさい(><)。
でも、明日までのレポートをどうしても仕上げなきゃいけないので、
すみませんが、お教えください。

今、物理学B(電磁気学)をやっているんですが、そこで「ガウスの定理の証明」
が宿題で出されたんです。お願いします、お教えください〜。
312ご冗談でしょう?名無しさん:02/11/07 15:44 ID:42XoannZ
お勧め!!
見れば分かるけど、安いし、女ウケいいから使える。
http://www.adultshoping.net/linkstaff.cgi?id=001951
313ご冗談でしょう?名無しさん:02/11/07 16:39 ID:???
>>311

ベクトルAが向いてる方向をz軸に取って、
A・dS = Az儡z = Az(x,y,z+凛/2)凅凉 - Az(x,y,z-凛/2)凅凉
= ∂z Az 凅凉凛

てとこでどうよ?
あとは任意の座標軸で成立する形に変形して、両辺に積分記号を付けるだけ。
数学科だと、これは証明ではなく説明だと言われるが。

分からなければ、ファインマンの電磁気読むべし。
オレはこの件については、これ以上の質問は受けんからな。
314大学生名無し:02/11/07 16:41 ID:h/DKXU5w
>>313

わ〜♪どうもありがとうどざいます(^0^)。
ファインマンの電磁気もよんでみますね♪
どうもありがとう♪
315アズサ:02/11/07 19:08 ID:iHRwZM9l
こんにちは!初めまして。
どうしてもわからないのでくださいm(--)m
豆電球&ダイオード&ツェナーの"I-V特性について”です。
参考になるHPでもかまいません。お願いします。
316187:02/11/07 19:34 ID:82zwDbpQ
>>300
exp(iwt)とおけば1/wCがでるのはわかりましたが、
なぜ、exp(iwt)と奥のでしょうか?
exp(iwt)=coswt+isinwt
としても、isinwtはよけいなのでは?
317ご冗談でしょう?名無しさん:02/11/07 19:49 ID:???
じゃあexpで式書いて虚部だけとってみな
318ご冗談でしょう?名無しさん:02/11/07 20:02 ID:???
>>316
いっそのこと複素数、ベクトルによる
交流理論を勉強してしまうことを勧める。
高校の範囲なんか越えてしまえ。
大したこと無いよ。工業高校では
やっているのではなかろうか

319ご冗談でしょう?名無しさん:02/11/07 20:03 ID:???
320みとにくに:02/11/07 20:10 ID:PBjkF1L1
すいません、両球体の中心間の距離がLです。つまり中心と中心を結んだ直線がL
どうなるのでしょうか?
321大鳥居つばめさん ◆dpLvO.T.cY :02/11/07 20:13 ID:SDcJSyKf
>>310
半導体のI-V関係を線形近似するためにdv/diを用いるとしても、
ダイオードとコンデンサーが並列になってる場合など、
コンデンサーにもR=dv/diを用いていいのだろうか?

>>318
工業なら複素数をやってると思ってた。けど、私の知識もつたないんですね。実際。

複素表示exp(iwt)にした場合、計算した結果の実数部分(または、虚数部分)をとるか、
共役なexp(-iwt)と足し合わせることによって、coswt、sinwt を導くかによって、実在波にする。

ところで、Z=R+iX (Z:インピーダンス、R:抵抗分、X:リアクタンス)という表示に従うと、
R=1/wC ではなく、X=−1/wC なんすね。
322爆弾魔(ボマー) ◆LClnpE3v3Y :02/11/07 20:22 ID:???
   ヽ`ヽ、| ヽi l/iノ |
  __ヾ l  `;       'Z__,..
  ヽ,`; '           /
ー-、,ゝ   _,.,ェ-ー''-‐z、、  ヾ、
  ヽ`'i‐'"゙       ヽ,   ゝ
    |`    ‐--、、_ `i   f
    l,‐'''  _,.-‐''"~./ヽl,-、. /
   ( ̄ ̄〕~ ヽ、_,/   |ヘ、V
   li''ー'i゙        |)ノ /
   l|  ゙' `       ト-イ
   {,   r_;:=-     ! |   勉強してないとどんな大人になるんですか?
    ヽ,  -‐     /  |
     ヽ     /    i,   __,,..
      |    /   /  ヽ/  ヽ--;
323爆弾魔(ボマー) ◆LClnpE3v3Y :02/11/07 20:23 ID:???
   ヽ`ヽ、| ヽi l/iノ |
  __ヾ l  `;       'Z__,..
  ヽ,`; '           /
ー-、,ゝ   _,.,ェ-ー''-‐z、、  ヾ、
  ヽ`'i‐'"゙       ヽ,   ゝ
    |`    ‐--、、_ `i   f
    l,‐'''  _,.-‐''"~./ヽl,-、. /
   ( ̄ ̄〕~ ヽ、_,/   |ヘ、V
   li''ー'i゙        |)ノ /
   l|  ゙' `       ト-イ
   {,   r_;:=-     ! | 勉強ばかりしてると君達みたいな変な人間になっちゃうんですか?
    ヽ,  -‐     /  |
     ヽ     /    i,   __,,..
      |    /   /  ヽ/  ヽ--;
324ご冗談でしょう?名無しさん:02/11/07 21:13 ID:XoW39J5W
N次行列のn乗の計算の手法を、簡単でいいのでおせーて管謝意。
なんかFourie変換使うとか言われちまいました
325ご冗談でしょう?名無しさん:02/11/07 23:29 ID:???
>>324
フーリエ変換ね〜。 普通対角化だわな〜。

1. まずAS=(diag)Sとなる行列を求める。固有値問題ですね。
2. det|A-λI|=0を解いて後はムニャムニャ(線形代数の教科書見てちょ)

すると、例えば、A^2は、
S^-1(A^2)S=S^-1A(SS^-1)AS = (S^-1AS)(S^-1AS)=(diag)^2=(固有値^2)

nの場合も同様で、
A^n = S(固有値^n)S^-1 になる。

これではダメですか?
326ご冗談でしょう?名無しさん:02/11/07 23:38 ID:ztUF0nZI
地球の質量は常に一定なのですか?厨房でスマソ
327ご冗談でしょう?名無しさん:02/11/07 23:49 ID:???
>>326
 どこまでを地球と考えるかによるんじゃない?
328ご冗談でしょう?名無しさん:02/11/08 00:09 ID:???
>>326
隕石落ちてるから増えてるんじゃないの?
でも人工衛星打ち上げててるからな〜。
プラスマイナスでどちらが多いのだろう?
329ご冗談でしょう?名無しさん:02/11/08 00:16 ID:???
身も蓋もない結論:ほぼ一定。
330ゴレイヌ ◆5m18GD4M5g :02/11/08 00:41 ID:g2ROLa5S
物理板のサンジくんと雫ちゃんが少年漫画板でスレ乱立してるぞ。
あんまお痛が過ぎると、物理板に糞スレ乱立させてつぶすぞ
331326:02/11/08 00:56 ID:IaAkLUcw
地球はほぼ一定か、でも、宇宙の重さは一定なのかな?
332ご冗談でしょう?名無しさん:02/11/08 03:31 ID:NYpsV6bM
スピンを発見した人は誰?
333ご冗談でしょう?名無しさん:02/11/08 04:23 ID:rqnN0Lj0
真空のエネルギーが宇宙の膨張の原理と考えてもいいの?
334ご冗談でしょう?名無しさん:02/11/08 05:57 ID:???
いい。
335名無しゲノムのクローンさん:02/11/08 12:23 ID:1iTO6Gh6
>>332
ウーレンベック&ハウトスミット
336ご冗談でしょう?名無しさん:02/11/08 12:46 ID:eHDkViym
私は携帯からなのですが、書き込みに長い文を書いても、エラーで長すぎる、と表示されてしまいます。みなさんの長い文には「省」というものがついていますが、何かそれは関係があるのでしょうか。どうすればいいか教えてください。
337大鳥居つばめさん ◆dpLvO.T.cY :02/11/08 12:55 ID:???
一行があまり長くならないように、改行してくれ。
338ご冗談でしょう?名無しさん:02/11/08 14:35 ID:1iTO6Gh6
隕石・宇宙塵など宇宙降下物は2〜6万ton/年
大型船1隻ぶんだ
地球の質量と比べれ
339ご冗談でしょう?名無しさん:02/11/08 14:35 ID:1iTO6Gh6
338は>>328
340ご冗談でしょう?名無しさん:02/11/08 14:47 ID:???
アノーマリーって高次元の弦理論でしか消せないんですか?
4次元で消す方法も、発明されてるんでしょうか。

(当方ど素人なのですが、高次元の統一理論には
なんとなく不安を覚えてしまいます・・)
341ご冗談でしょう?名無しさん:02/11/08 15:48 ID:cRhGKfuC
ビッグバンに匹敵するというアテナエクスクラメーションは実在しますか?
また、これに耐えられる黄金聖衣はどんな素材でできていますか?
342ご冗談でしょう?名無しさん:02/11/08 16:42 ID:wggb8uar
ある物体をある位置からある位置まで動かしたときの変位の平均変化率と
平均速度を求める問題ですが、平均変化率と平均速度の違いがよく
わかりません。
それぞれどのようにして計算すればいいんでしょうか?
どっちも変位Xとかかった時間tを使って求めるってことだけはわかってるんですが。
343ご冗談でしょう?名無しさん:02/11/08 17:30 ID:SZu3Uwkh
物質中にたまっている電荷って、原理的にはどうやって測定するんですか?
344ご冗談でしょう?名無しさん:02/11/08 17:47 ID:???
∇・D=ρ
345ご冗談でしょう?名無しさん:02/11/08 18:49 ID:EVsT8goo
地球から逃げていく大気はどれくらい?
346ご冗談でしょう?名無しさん:02/11/08 22:25 ID:???
BRST変換のBRSTって何の略?
347ご冗談でしょう?名無しさん:02/11/09 00:00 ID:???
4元力f^μ=(f^0,f^1,f^2,f^3)はどうやって求めるのですか?

静止系では、ニュートン力学での力(F1,F2,F3)を用いれば静止系で
4元力は(0,F1,F2,F3)になることは理解できます。

よって、四元力の求め方は

・物体静止系で4元力(0,F1,F2,F3)を求める。Fは普通の力学での力
・これを考えている系にローレンツ変換する

でよろしいのでしょうか?いろいろ本をみても、四元力の求め方が
書いてないのです。ただ、あるとすれば方程式がたてられるとしか
書いてないのです・・・。入門コースとパリティ物理コースをみました。
348ご冗談でしょう?名無しさん:02/11/09 00:14 ID:???
>>346 人の名前。
349ご冗談でしょう?名無しさん:02/11/09 00:15 ID:???
4人の頭文字
350:02/11/09 00:15 ID:MNwmBqnd
エーテルって存在しないはずなのに、高校の化学の教科書に「麻酔性がある」ってかいてあるんだけど・・・・
351ご冗談でしょう?名無しさん:02/11/09 00:17 ID:???
たぶん同音異義語ってやつ
352ご冗談でしょう?名無しさん:02/11/09 01:08 ID:SRj6eC42
光に圧力があるって初めて知ったのですが、
圧力ある=質量があるってこととは違うのですか???

あと、彗星の尾は軌道上の太陽と逆側に流れているのは、
その圧力のせいだという人がいるのですが、遠心力と違うのでしょうか?

たぶん超初歩的質問かもしれませんが、どなたか教えていただけないでしょうか?
353ご冗談でしょう?名無しさん:02/11/09 01:27 ID:???
>>352
圧力あり=運動量ありです。
光の場合は、p=E/c
彗星の尾のことは知らん。
354ご冗談でしょう?名無しさん:02/11/09 09:36 ID:???
>>352
彗星は自由落下軌道にある(遠心力と重力がほぼつりあっている)ので
遠心力では尾は本体と分離しません。

彗星の尾はダストテイルとイオンテイルの2種類あって、どっちかが光圧、
どっちかが太陽風で太陽と逆側に伸びます。
355353さんに補足すると:02/11/09 09:37 ID:???
質量がなくても運動量は持ち得るのです
356ご冗談でしょう?名無しさん:02/11/09 13:02 ID:vfCRLLR4
ベクトルポテンシャルの意味するところを答えられる人いますか?
357ご冗談でしょう?名無しさん:02/11/09 13:06 ID:???
そりゃー、ポテンシャルだって言うんだからポテンシャルなんだろ?
本人もポテンシャルがあるって言ってるんだから、ここは一つ長い目で見守って行こう
じゃないかということでどうだい?
358HAL ◆2eOaL2U.Gg :02/11/09 14:28 ID:L2Edq5Ab
>>356
ベクトルポテンシャルの意義ってことかな?

電場磁場の相互の作用を明確にあらわせることかな。

たとえば、真空での電磁波の方程式は、EとBの波動方程式を
導くけど、あたかもそれぞれが独立であるかのように見えるよね。

これを、ベクトルポテンシャルとスカラーポテンシャルの波動方程式で
書き下すと、EとBの関係(直交するという)が、明白になったいする


こんな説明じゃだめかい。

359福田和也:02/11/09 18:46 ID:tNAwKqei
物理数学ってなんなのさ。ベッセル関数とか,ルジャンドルなんたらとか。
どうやって勉強すればイイのかわからん。教えてくれ,
数学科三回生です。
360ご冗談でしょう?名無しさん:02/11/09 20:30 ID:???
>>359
うちの大学にも物理数学の授業がありますが、担当講師ですら学生時代に物理数学をよく理解できなかったようです。
その担当講師の友達の、現東北大学の化学関係の教授も、物理数学には学生時代に挫折したようです。
それほど難しい学問らしいです。だから基礎的入門的な教科書から入るといいらしいです。
361ご冗談でしょう?名無しさん:02/11/09 20:31 ID:???
↓の議論は一体誰が正しいのでしょうか?
http://pc.2ch.net/test/read.cgi/unix/983119362/73-87
362やらして大鳥居さん:02/11/09 20:42 ID:UKd8nD53
雫ちゃんに中出ししたいです。
どうすればいいんですか?
効果的なピストンの速度を誰か計算してください。
363助けてください死にそうです:02/11/09 20:48 ID:LfnNso3j
状態量がわからないです。
本をみたら、「系の状態だけで一義的に決まる量」と説明されています。
なんとなくわかるのですが、「熱や仕事は状態量とは言わない」というのがよくわからないです。

例えば、断熱材で出来ているゴムボールで手で握りるような現象を考えてみます。
また、ゴムボールを系として捉えて以後は書きます。
ゴムボールは、閉鎖系なので次の関係式が成り立ちます。
(内部エネルギー変化)=(系がなされた仕事)+(系に入った熱)
この関係式において、今の場合、断熱材を用いているのだから(系に入った熱)は0だと思います。
だから、(内部エネルギー変化)=(系がなされた仕事)になります。
系の、つまりゴムボールの、握られる前と握られている時とを比較して、両者の内部エネルギーの差を求めれば、系がなされた仕事は確定できます。
それでも、(系がなされた仕事)は、状態量ではないのですか?
364ご冗談でしょう?名無しさん:02/11/09 20:55 ID:???
>>356

アハラノフ・ボーム効果

電子が磁場や電場がなくてもベクトルポテンシャルがあれば
その影響を受けることをいいます.
1959年にY. AharonovとD. Bohmにより予言され、
いまでは完全に実験によって確かめられています.
365ご冗談でしょう?名無しさん:02/11/09 21:00 ID:IWlTpo9V
なぜ鏡は左右逆に映るのですか。

質問文そのものがなんかおかしい気もしますが
それも含めて誰かわかりやすく解説してくらさい
366RAM ◆.6r6oheRAM :02/11/09 21:03 ID:JFd7R7bd
>>363
それは「断熱材を用いた」という特別な場合でのみに成り立つことで、
一般的には成り立たない。摩擦力に対して行った仕事は状態量にはならないよ。
367ご冗談でしょう?名無しさん:02/11/09 21:03 ID:UKd8nD53
しずくさんをおかずにマスターベーションしたいです。
効果的なおかずと射精精子量の関係をグラフにしましょう。
368ご冗談でしょう?名無しさん:02/11/09 21:07 ID:???
>>358

だめ。
マックスウェル方程式みりゃわかる。
369助けてください死にそうです:02/11/09 21:11 ID:LfnNso3j
>>366
つまり、状態量は、ケースバイケースで流動的に判断される性質のものでは無くて、
あらゆる状況において成り立つものなんですか?

だから、状態量を次のように理解してもいいのですか?
状態量=「系の状態だけで、あらゆる状況において一義的に決まる量。」
370ご冗談でしょう?名無しさん:02/11/09 21:19 ID:???
ちがいます。
371大鳥居にハメる:02/11/09 21:22 ID:UKd8nD53
しずくさんをおかずにマスターベーションしたいです。
効果的なおかずと射精精子量の関係をグラフにしましょう。
372助けてください死にそうです:02/11/09 21:22 ID:LfnNso3j
じゃあ状態量とは何ですか?
教えれ。
373ご冗談でしょう?名無しさん:02/11/09 21:23 ID:???
「ケースバイケースで流動的に判断され」る性質のものだったら
「一義的に決まる量」にはならないでしょう。

例えば温度っていったらある例えば水なら水があればそれだけで決まる。
その水がかつて氷だったかとかそんなことにはよらない。

仕事とか熱はそうじゃなくてどんな風に変化させたか(等圧的とかなんとか)
によるから水をさして「この水の仕事はいくらだ」とか「熱はいくらだ」とか
いえないから状態量でない。
374ご冗談でしょう?名無しさん:02/11/09 21:23 ID:???
>>365
鏡は左右じゃなく鏡に垂直な方向を入れ替える
鏡を地面にして立ってみると上下が入れ替わって、左右はそのまま
375大鳥居つばめさん ◆dpLvO.T.cY :02/11/09 21:30 ID:kJtDfcio
>>373
その上、力学で力が保存量で無い場合の仕事と違って、
(系がなされた仕事)、(系に入った熱)のように、
熱力学ではエネルギーを変化させるためのこ項が2つあって、
実際の変化では、それらのいずれも0には決してならないのだ。
376ご冗談でしょう?名無しさん:02/11/09 21:31 ID:???
>>373
グッド
377ご冗談でしょう?名無しさん:02/11/09 21:38 ID:???
>>372
あのまんこにされた仕事は?
聞かれてもそのまんこの履歴をしらなかったらわからないだろ?

あのまんこの温度はいくつだ?
と聞かれたら温度を測定すれば履歴に関係なくわかるだろ? 
378ご冗談でしょう?名無しさん:02/11/09 21:43 ID:???
んでじゃあ熱は状態量じゃないのにそれを絶対温度で割ったエントロピーは
なんで状態量なんだ、とか言われると新たな泥沼にはまるわけだな。
379大鳥居にハメる:02/11/09 21:45 ID:UKd8nD53
しずくちゃんへの愛のためにこのスレをつぶします。
見ててね、しずくちゃん。
380大鳥居にハメる:02/11/09 21:47 ID:UKd8nD53
やってやる、やってやるぞーーーー
381365:02/11/09 21:58 ID:IWlTpo9V
>>374
なるほど。ありがとう
382大鳥居つばめさん ◆dpLvO.T.cY :02/11/09 22:34 ID:kJtDfcio
>>378
それは、dU(P,V)のように内部エネルギーが2変数以上の関数だからだと。
>>375は>378を意識して書いたのだー。

訂正;
>力学で力が保存量で無い場合
・・・保存力・・・・
383ご冗談でしょう?名無しさん:02/11/09 22:37 ID:???
>>378
Tは積分分母だから。

>>382
死ね
384ボスケテください:02/11/09 23:38 ID:???
熱とは何でしょうか?
調べたところ、「位置エネルギーと運動エネルギーの総和」だと書かれていました。
そうして考えていくうちに熱と温度の違いがわからなくなりました。
熱とは一体なにで、熱と温度の違いは何なのでしょうか?
385ご冗談でしょう?名無しさん:02/11/09 23:42 ID:???
マンコは何でピンクなの?
386ボスケテください:02/11/09 23:45 ID:???
検索したら沢山ヒットしました。
色々見てから分からなかったらもう一度きます。
387332:02/11/09 23:51 ID:YMkQ1kM5
>>335

名無しゲノムのクローンさん
ありがとうございました。:)
388ご冗談でしょう?名無しさん:02/11/09 23:58 ID:???
日常生活では「温度が高い」という意味で「熱がある」というけど物理でいう
熱とはちょっと違うから混乱の元なんだよなあ。
389大鳥居つばめさん ◆dpLvO.T.cY :02/11/09 23:59 ID:kJtDfcio
>>384
「熱ていったい? 」
http://science.2ch.net/test/read.cgi/sci/1032529886/
一応、2chのスレをあげておこう

>>379
ノンフィーアー、ノンペイン、愛の前に立つ限り〜
ラブリードリーム、愛の力を信じましょう〜
390ぶちゅり:02/11/10 11:21 ID:7KAar7xs
マイケルソン干渉計の歴史的意義について知りたいのですが検索してもイイのがありません。
どこかいいところがあったら教えて下さい。
391助けてください死にそうです:02/11/10 12:00 ID:???
>>373.375.378.382
馬鹿なのでまるでわからないです。
状態量の定義から書くと、「状態が決まれば、一義的に決まる巨視的な量。」

「一義的(=1つの答えしかない)に決まる量」の解釈について、
どんなものでもある条件下で定められているので、
「一義的に決まる量」というのも・・・「ある条件下において一義的に決まる量」ということではないですか?
また、内部エネルギーの変化前と、変化後という状態が決まったとします。

ここで、閉鎖系で断熱的という条件下で、なおかつ、内部エネルギーの変化前と変化後という状態が決まったのだから、
あと、数量をある一つの答えに定めることが出来れば、状態量の定義から、定まった量は状態量の仲間入りと出来ませんか?

上のような場合、
(内部エネルギー変化)=(系がなされた仕事)+(系に入った熱=0J)
となり、
(系がなされた仕事)を定めることが出来て、(系がなされた仕事)は状態量に思えて仕方がないです。
392ご冗談でしょう?名無しさん:02/11/10 12:08 ID:???
ある(熱平衡にある)物質があればそれだけで決まるのが状態量。
仕事や熱はそれだけじゃ決まらないから違う。

物理じゃなくて定義の問題だな。
393大鳥居つばめさん ◆dpLvO.T.cY :02/11/10 12:13 ID:5VytVfWu
ある状態の確率と、状態から変化する確率の違いみたいな。
394HAL ◆2eOaL2U.Gg :02/11/10 12:45 ID:wSQu59HK
>>391
仕事、熱は状態変化に対して定義される量だからね。

状態量は、状態そのものの関数。
395ぶちゅり:02/11/10 14:04 ID:7KAar7xs
マイケルソン干渉計の歴史的意義について知りたいのですが検索してもイイのがありません。
どこかいいところがあったら教えて下さい。

待ってます♪
396ご冗談でしょう?名無しさん:02/11/10 14:05 ID:fJ5rYsV9
物理板の看板はファンデルワールスですか。
397ご冗談でしょう?名無しさん:02/11/10 14:08 ID:K5lBUWxH
2000年東京大学前期試験★次の問いに答えなさい。

問1、Gスポットの座標を求めなさい。

問2、右の乳首から左の乳首までA君が時速0.5kmで舐めた時、何秒かかるか求めなさい。

問3、A君が毎秒3回の速さで手マンした時、Bさんが192秒後にイク事を証明せよ。なお、A君は中指を使ったものとする。

問4、A君がBさんに1時間手マンした時の仕事率を求めよ。

問5、BさんがA君の上で上下運動している時の重力を求めよ。なお、πは3.14とし小数点は四捨五入する事。
398ご冗談でしょう?名無しさん:02/11/10 15:34 ID:6iDAej31
相対性理論の入門書を読んでてわからないことがあります
*ガリレオの相対性原理
*ガリレイ変換
この二つの意味がよくわかりません
相対性理論はガリレオの相対性原理に従うがガリレイ変換には従わない
と言っているみたいで最初矛盾してるように感じました
ガリレオの相対性原理は時間の進み方がゆっくりになることに矛盾しないのですか?
教えてください
399ご冗談でしょう?名無しさん:02/11/10 15:44 ID:???
>>398
ガリレオの相対性原理は時間の進み方は関係ないよ
等速直線運動している座標系の上でも物理法則は変わらないってだけ
その意味では相対論の本質に関する分かりやすい一例
400来年院生ですが:02/11/10 16:01 ID:Za2zj6CM
すいまそん
コンピュータゼンゼンわかんないんですが
FORTRANに習熟したいっす!
でもコンピュータって何からやっていいかわからない(入り口がわからない)
ですので詳しい方、オススメの本なんかあったら教えてください
FORTRANは、なんかシュミレーションに使うらしいっす
あとできれば”どんなふうに習熟していったか”についての過程を
簡単でいいので教えてくださいませ
401ご冗談でしょう?名無しさん:02/11/10 16:03 ID:???
プログラム板で聞いたほうが詳しい人おおいんじゃない?
402ご冗談でしょう?名無しさん:02/11/10 16:05 ID:???
>>400
ラリー・ニーホフ+サンフォード・リーストマー著
「入門Fortran90」(株)ドキュメントシステム

この本は結構いいと思う
あと、習得は具体的に課題をこなしつつやったほうが早い
シミュレーションは技術じゃなく根気ってことも忘れずに・・
403ご冗談でしょう?名無しさん:02/11/10 16:10 ID:6iDAej31
>>399
ありがとうございます
もうひとつ教えてください
ガリレオの相対性原理の考え方に従った上で
自分と動いている人との間に
*ガリレイ変換(同じ時間が流れる)
*ローレンツ変換(時間の流れは違う)
この二つの変換方法が考えられて実験の結果ローレンツ変換が正しいことが確かめられた
この理解でいいですか?
404来年院生ですが:02/11/10 16:11 ID:Za2zj6CM
おお!はやい!
そういえば趣味レート板ってのがありましたね(かなり初心者)
402さんどうも
その本には自宅のパソコンでもできる練習問題みたいなものも
ついてますか?
405ご冗談でしょう?名無しさん:02/11/10 16:19 ID:???
>>403
質問の意図が分かりにくいが・・
ガリレオ変換は動いている人の速度は遅いんで
時間の流れもたいして変化しないと考えるべき
ローレンツ変換をV/C〜0としたときの近似

>>404
ちゃんとあるよ
ただ演習問題じゃあまりやる気になれないから
院性としての実際の課題からコードを組んで、
分からないとこは本を読むってほうが習得は早いと思う
406405:02/11/10 16:29 ID:???
>>404
あっ、そうそう
fortran90がコンパイルできる環境じゃないと>>402の本は意味ないです
fortran77しか使えない研究室とか多いから
407398:02/11/10 16:45 ID:6iDAej31
>>405
すいません
動いている人の速度が遅いときはガリレオ?変換でも大きな差はないけど
それでもやっぱりローレンツ変換の方が正確なんですよね?
408HAL ◆2eOaL2U.Gg :02/11/10 17:04 ID:???
>>407
厳密にはそうだけど、
0.00000000000000001(0の個数は適当)ぐらいの差しかない
とすれば、ガリレイ変換は正確といえるんじゃない?
試しに計算練習として時速360km/h=100m/sで計算してみるといいんじゃないかな。

ct'=γct - γβx
x'= - γβct + γx

γを展開して二次で止めると

γ=(1-β^2)^2〜1-(1/2)β^2

よりβを一次までにすれば

t'=t-β^2 x・v〜t
x'=x -vt

より、ガリレイ変化に一致します。
βを二次までにすると

ct'=(1-(1/2)β^2)ct -βx
x'=(1-(1/2)β^2)x -βct

だよね。c=3*10^10cm/s=3*10^8m/sであるから、
v=100m/sのとき、β〜10^-6

さあ、あとは計算して、ガリレイ変換とほとんどかわらないことを
確かめてください。
409HAL ◆2eOaL2U.Gg :02/11/10 17:09 ID:???
t'=t+(-(1/2)β^2t-βx/c)
x'=x-vt-(1/2)β^2)x

と変形したほうがわかりやすかったかもね。
後ろの項がガリレイ変換に対する補正。


410398:02/11/10 17:48 ID:6iDAej31
>>408 >>409
ありがとうございます
うわ、計算式むずいので遠慮しときます
つまり時速300キロくらいの問題ならガリレイ変換もローレンツ変換も差はないよということですね?
もう二個だけ質問

1 本によるとニュートンは絶対空間を信じていたとあります
ガリレイ変換は特別な空間は無いとしているのですよね?
ではニュートン力学はガリレイ変換でもなくローレンツ変換でもないものなのでしょうか?

2 本によるとローレンツ変換は相対性理論の実証実験によって確かめられたと書かれています
しかしインターネットでは電磁気理論のマックスウェル方程式が採用している変換となっているようです
ローレンツ変換が正しいとする根拠はどこから?

おねがいします
411ご冗談でしょう?名無しさん:02/11/10 17:53 ID:???
時速300kmどころか秒速3000kmでもニュートン力学と相対論の誤差
は1/20000の程度だよ。
412HAL ◆2eOaL2U.Gg :02/11/10 17:56 ID:???
入門書って数式を使わないもののことでしたか。

ちょいと説明を変えるので考えさせてください。
413ご冗談でしょう?名無しさん:02/11/10 17:57 ID:???
HAL ◆2eOaL2U.Gg さんこんなに2chにいて大丈夫?
414ご冗談でしょう?名無しさん:02/11/10 17:58 ID:???
>>410
ニュートン力学は絶対空間を必要とはしていない
ニュートンが勝手に絶対空間を信じていただけ
ガリレオ変換に対してニュートン力学は不変と解釈していい


>インターネットでは電磁気理論のマックスウェル方程式が採用している変換

マックスウェル方程式はガリレオ変換には対応せずローレンツ変換に対応
しかしローレンツ変換を暗示していただけで
根拠としては観測、実験からと思ったほうがいいかな
415398:02/11/10 18:24 ID:6iDAej31
>>411
秒速3000kmということは3000*60*60=時速1080万キロ?でも差は無いよということですね?
気にしなくてもいいということはわかりました
ありがとうございます
>>412
HALさんすみません
>>414
ニュートン力学=ガリレイ(ガリレオ)変換ですね
相対性理論の示した奇妙な世界が正しいのかについては
現実がローレンツ変換に従うという観測、実験結果が根拠として結論が出た感じですか
ありがとうございます
416HAL ◆2eOaL2U.Gg :02/11/10 20:56 ID:???
>>413
まあ、支障無い程度にね。
つばめさんには負けますよw

417ご冗談でしょう?名無しさん:02/11/10 23:11 ID:woaom3s3
解析力学で初歩っぽい質問があります。

「仮想変位」という概念がどうしても理解できません。
なぜ、時間を止めているのにごくわずかな変位だとしても動くことが可能なのですか?
極わずかな時間を考えて、それが極限で0に近付くときに動いた距離というのなら
理解できるのですが、時間を完全に止めてしまった上で、その変位を考えるというのはどうしても
受け入れ難いのですが・・・。

その後のラグラジアン方程式を導く過程とかは疑問はないんですけど
肝心の出発点のところがどうしても理解できません。
「仮想」という言葉を付けるにしても、そもそも有りえないものを想定している時点で
「物理」と乖離してしまっているような気すらします。

2冊ほど本を読みましたが、どうも・・・あ、3冊目を読んだら解答らしきものが・・・ありました。
この場合、ある物体についての「変位」を考えているわけではなくて、ある物体からわずかに
「変位」した場所を考えている・・・ということでいいんですね?
418ご冗談でしょう?名無しさん:02/11/10 23:40 ID:???
>>416つばめさんは大丈夫じゃないんです。
彼は相当支障をきたしている。
419質問です:02/11/11 00:06 ID:+scAgFg2
物理の問題(大学受験レベル)の解答の中に書いてあることで、

電場の強さEは、単位面積あたりの電気力線の本数で定義されている、


がどうしてもわかりません。それはどうやって導くのですか?
420ご冗談でしょう?名無しさん:02/11/11 00:09 ID:???
「定義」って辞書で引いてください。
421質問です:02/11/11 00:11 ID:+scAgFg2
じゃあ、公式として扱っていいんですか?
422ご冗談でしょう?名無しさん:02/11/11 00:14 ID:???
まあ、ご自由に。電場と電気力線のどっちが基本的な物理量かって考えると
むしろ電気力線の本数を定義していると考えたほうが個人的にはしっくりいくが。
423ご冗談でしょう?名無しさん:02/11/11 01:13 ID:R5Y9yXQv
ベクトルポテンシャルの意義、意味を答よ。明確に。
静電場におけるポテンシャル量の意義、意味を答よ。明確に。

↑試験問題で、解答率100%で正解率0.1%だった問題です
424ご冗談でしょう?名無しさん:02/11/11 01:18 ID:R5Y9yXQv
419>> じゃ書いてみ。単位面積あたり本数が多いところが電場が強い所なんだから、実際2つの異符号等量電荷の場合どうなるかね?
    クーロンの法則からわかるだろ?
    じゃ単位面積あたり本数がすくない→1本一本がまばらで、すきすきなところは弱い。
    実際遠くのほうならすきすきだ。
425ご冗談でしょう?名無しさん:02/11/11 01:19 ID:???
そもそもなにが正解か解釈によって異なるからそりゃ0.1%にもなるだろうな。
大物理学者か手抜きかのどちらかだろう、出題者は。
426ぶちゅり:02/11/11 01:28 ID:CvbzHvO5
マイケルソン干渉計の歴史的意義について知りたいのですが検索してもイイのがありません。
どこかいいところがあったら教えて下さい。

待ってます♪


427423:02/11/11 01:33 ID:R5Y9yXQv
つうか答えられるのかどうかだって、答えてみ。
教科書読んでも無駄やで,物理的な感覚を試してるわけ
428HAL ◆2eOaL2U.Gg :02/11/11 01:46 ID:???
>>423
上にも同じような質問が合ったな。
>>358と答えたが、>>368というご批判を受けました。
たしか、ガウスの法則をあわせれば直交性がいえるんだっけ。

(じゃ、アハロノフボーム効果っていうのはだめ?)

反論はできません。「意義」の定義が曖昧だからな・・・

僕もみなさんの考える意義をきいてみたい気がします。

静電ポテンシャルの方はわかりやすいね。
1つの成分の関数で3成分の電場を表現できるからわかりやすいでしょ。
同じ説明はベクトルポテンシャルには適応できないわけだが。

ベクトルポテンシャルの旨味は、上で説明したように、電場磁場の結合が

E=-∇φ - ∂A /∂t
B=∇×A

と明快になることだとおもうが、違うかな〜?
429ご冗談でしょう?名無しさん:02/11/11 01:46 ID:???
>>423
つーかキミのところは学生が1000人もいるのかと・・・(略
430ご冗談でしょう?名無しさん:02/11/11 01:47 ID:???
つうかベクトルポテンシャルの意義っていっても多分出題は古典電磁気
の範囲でしょ。AB効果がどうたらとか言えばいいわけ?
431HAL ◆2eOaL2U.Gg :02/11/11 01:47 ID:???
きっと、この答案では、99.9%の方に入るんだろうね。
厳しい問題だな。いまそのテスト受けると「優」とれない予感。
432HAL ◆2eOaL2U.Gg :02/11/11 01:48 ID:???
>>429
ワラタ。たしかに1000人以上いることになるな。中国の大学か?
433ご冗談でしょう?名無しさん:02/11/11 01:50 ID:???
>>423が「模範解答は何ですか?」と聞くのが一番いいな。
それにたいしてみんなも色々つっこみがいがあるし。
434ご冗談でしょう?名無しさん:02/11/11 01:53 ID:???
あ、もちろんこの先生に聞くという意味ね。
435HAL ◆2eOaL2U.Gg :02/11/11 01:54 ID:???
>>426
MITの特殊相対論の教科書がその辺の記述が詳しかった気がしますが。
他にもいろいろな本があるので、図書館にいってみては?
436ご冗談でしょう?名無しさん:02/11/11 01:58 ID:BFQG/t1Z
437ご冗談でしょう?名無しさん:02/11/11 01:58 ID:???
>>423
意味はベクターボソンの波動関数。
そのままではローレンツ不変に見えないマクスウェル方程式を
ローレンツ不変な形に書き直すことができるというのが最初の意義。
そのまま相対論的量子力学になっているので、
場を量子力学で扱えるという意義もある。

って、ゲージ自由度とかまで詳しく話さないといけませんか?
438ぶちゅり:02/11/11 02:05 ID:CvbzHvO5
>>435
>>436
どうもです♪
439ぶちゅり:02/11/11 02:07 ID:CvbzHvO5
よめません♪
440ご冗談でしょう?名無しさん:02/11/11 02:09 ID:???
ワロタ
441ご冗談でしょう?名無しさん:02/11/11 02:35 ID:L9dhUkH5
どこかで翻訳できるはず?
僕はよく知らないけど。
442ご冗談でしょう?名無しさん:02/11/11 03:05 ID:???
>>423
模範解答キボンヌ!

出題者は一体、何を意図したんだろう?


洩れの答え:

(意味) ローレンツ多様体上のU(1)群ゲージ主束に入った接続を、
ひとつの代表元として、代数に値をもつ接続1-フォームとして表したもの。

(意義)
この主束には荷電粒子に対応するスピン束などが随伴しており、
接続やスピン切断などを値域としてもつ作用汎関数が電磁相互作用を記述している。
と神様が決めた。
443ご冗談でしょう?名無しさん:02/11/11 09:57 ID:???
444場違えでなければ教えてください 四苦八苦してます:02/11/11 10:29 ID:UjDbvvxi
ばね定数:4.9〔N/m〕 の軽いつるまきバネの下端に質量 0.020〔kg〕 の皿を吊るしその皿の上に質量 0.050〔kg〕 のオモリを乗っけた。
オモリを乗せつりあった位置から 010〔m〕 下方に伸ばし静かに放すと単振動を始めた。

(1)最高点でのオモリの加速度は?
(2)最高点で、皿から見たオモリに現れる慣性力はどの向きに何N?
(3)最高点で、オモリが皿から受ける垂直抗力の大きさは何N?
(4)つりあいの位置より、何m下方に伸ばして手を放すと、最高点でオモリが離れるようになるか? 

(1)はF=−mω^2χ=−κχ と a=−ω^2χ の公式より 下向き 7.0m/s^2と出ました
(2)上向き 0.35N
(3)0.14N
(4)0.14m
と答えだけは分かってますが解き方が分かりません 助けてください
お願いします
445ご冗談でしょう?名無しさん:02/11/11 12:17 ID:Qq4G+iqf
>>444
(2) 7(m/s~2)*0.05(kg)=0.35(N)

(3) ( 9.8(m/s~2)-7(m/s~2))*0.05(kg)=0.14(N)

(4) 9.8(m/s~2)*0.07(kg)/4.9(N/m)=0.14(m) ((1)の式を使えばいい。)
446ご冗談でしょう?名無しさん:02/11/11 12:52 ID:???
>>403
歴史的な順序はこう。

1. ニュートン力学とマクスウェル方程式の矛盾が明らかになった。

2. 実験(M-M実験)により、マクスウェル方程式が正しいことがわかった。

3. ニュートン力学で実験結果を説明するため、ローレンツ短縮≠ェ考え出された。
 この式はローレンツ変換と同じだが、時空の変換ではなく、エーテルの抵抗で物質が短縮すると解釈された。

4. アインシュタインが光速度不変則からローレンツ変換式を導き出した。
 この時点で初めて、ガリレオ変換はお払い箱になった

だから、
>この二つの変換方法が考えられて実験の結果ローレンツ変換が正しいことが確かめられた
というのはちょっと違う。
447孫悟空 ◆yGAhoNiShI :02/11/11 17:28 ID:???
ドラゴンボールZ
フジ(関東)で毎週月曜16:30〜放送中!!

  ::., :.;;;:: ;;;;;;;;;;;;;;;;;;;::::.::;;;;;;;;;;;;;;:;;;:;;;;;;;;;;;;: ...: :: ..: :::::::;;;;;;;;;;;;;;;;;;;:..:: :;;;;;;;;;;;;;;
  ;;;;::.::;;:::::::::::.::::..::::::::..,:;;;;;;;;;;;;;;;;;;;;;;;;;;;;;;:;;;;;;;;;::::::;;;;;;;;;;;;;;;;;;::::::::::::::.:;;;;;;;;;;;;;;;;;
  ;;;;;;;;;;;;;:::.:::.;;;;;;;;;;;;;;.:::;;;;;;;;;;;;;;;;;;;;;;;;;;;::...::...,;;;:..:.:::::::: . ;;;;;;;;;;;;;;;;;;;;; :;...::.:;;;;;;;
  ::.::..:.:::;;:::;;;;;;;;;;;;::::.;;;;;;;::::::;::.;;;;;;:::..     .::::.,::;;;;;;:::;;;;;;;;;;;;;;;;;;:::: ::;;;;;;;;;;;;;;
  ;;;;;;;;;;;::: ::::;;;;;;:: ::: ::::...  .: . . _.∩_  ..:;;;:;;;;:;;;;;;;;;;;;;;;;;;;;;;:::: :;;;;;;;;;
  ;;;;;;;;;;;;;;;;;;;;;;::::::...         ヽヘ;;. 人丿ス  :: ::::::;;;;;;;;;;;;;;;;;;;:::;;;; .:;;;;;;;
  : :. :;;;;;;;;;;;;;;;;;;..    从    θ斤:エh u    .:::::;;;;;;;;;;;;;;;;;;;;;;;;;;;;;;;::;;;;
  ;;;;:;,:.:;;;;;;;;;;;;:::.  __ 《Y》_   ∪レ..... 弋|      :::::;;;;;;;;;;;;;;;;;;;;;;;;;;;;;;;::;
  ;;;;;;;;;;;;;;;;;;;;::...  .uヘ人iイ  .  (. .」_ ノ    ...::::;;;;;;;;;;;;;;;;;;;;;;;;;;;;;;;;;;;;;:
  ;;;;;;;;;;;;;;;;;;;;,:..     (∨ヘ      |....|: .)    .:::;;,,;;;;;;;;;;;;;;;;;;;;;;;;;;;;;;;;;;;;;;
  ;;;;;;;;;;;;;;;;;;::::....    .|;|レ'      .(_;);;.| -〜、 ..:..:..:,,;,;;;;;;;;;;;;:::::: :::: ::::
  、 ._  _.:;〜⌒^^⌒⌒´⌒` ̄ ̄ ....::,...⌒~^⌒ ̄ ̄`〜._:::;:..:::
   ⌒ ⌒    ....::::::::::.:::.::;: :::... .:::: :::.  ..::  :::::::: ;::::::;:;.;:;,;,.,,; ...::⌒
  :;;;:::;::;: :::;:;;:::::::..::::::::::::::::;::::::::::::::::::::::::::::::::::::.  :; ..;::::::;::;: :::;:;;:;:::::;: :::;:;::
と〜けたこおりのな〜かに〜♪恐竜がい〜たら〜たまのりし〜こ〜みたいね〜♪
448HAL ◆2eOaL2U.Gg :02/11/11 20:41 ID:???
>>437
なるほど。分かりやすい。
ローレンツゲージいれたらダランベルシャンで書けて一目でわかるもんな。

>>442

うう、、わからない。どんな本を読めば>>442さんの発言が理解できるの?
449ご冗談でしょう?名無しさん:02/11/11 20:58 ID:???
>>417
君は変分・汎関数てものが分かっていない。(分りかけてはいるようだが。)
まず変分法を勉強するべし。

「その後のラグラジアン方程式(?)を導く過程とかは疑問はないんですけど」

前後の文章からして、xとv(=dx/dt)を独立変数(実は変数ではないだが)として
扱う事にも疑問を抱いてしかるべきかと。
450423:02/11/11 21:07 ID:1DQA1K6r
いや電磁気学の範囲だよ。だって院試だもん。
451423:02/11/11 21:08 ID:1DQA1K6r
つまり本当にわかってるやつは1000人のうち1人いないかいるかってことだな。
452ご冗談でしょう?名無しさん:02/11/11 21:27 ID:???
>>423
で、模範解答はないのか?
その1000人に一人の解答ってのを見たいな。

>>428>>437>>442
院試の答案として別段間違ってるとは思えんけど?

453大鳥居☆つばめ ◆dpLvO.T.cY :02/11/11 21:30 ID:67ngcgkB
いちおう、任意のベクトル場Vには V=-∇φ - ∇XW を満たすWが存在する。がいえる。

>>437
>ゲージ自由度
とは、電磁気学レベルなら、A→A+∇ψ φ→φ+ψ(t)
が可能ということくらいですか?
454ご冗談でしょう?名無しさん:02/11/11 21:31 ID:???
物理板が荒しにあってるのは何故ですか?
数学板みたいに荒涼としてきましたね。以上
455ご冗談でしょう?名無しさん:02/11/11 21:32 ID:???
何故よ?
456ご冗談でしょう?名無しさん:02/11/11 21:36 ID:???
>>453 微分が抜けてる。
457ご冗談でしょう?名無しさん:02/11/11 21:50 ID:1Fnlrh5a
摩擦損失水頭ってどういうことでしょうか・・・・・・
そもそも水頭ってどういうことなんですか?
本当にバカでスミマセン
458ご冗談でしょう?名無しさん:02/11/11 22:02 ID:apJmNj86
だれか、電磁のリアクタンスについて簡単に説明してちょ。
459ご冗談でしょう?名無しさん:02/11/11 22:05 ID:ClSFMwwj
しつもん何ですが・・・

平均直径22mm、ピッチ3mmのねじを用いて、
長さ25cmのスパナで80Nの力を加え物体を締め付けた。
締め付け力はいくらか。ただし、ねじ面の静摩擦係数は0.1

という宿題が出ました。
やってるんですが、どうもよくわかりません。
スパナをつかうからモーメントを考えるのではないかと思っていますが、
どう関連付けるかわかりません。
公式でねじの締め付けを出すものがないです。(あるかもしれないんですが)
ねじで持ち上げるという公式はあるんですがそれは重さが関係していて、
ここには使えないんじゃないかと。
どうやって問題を解けばいいんでしょうか??
460ご冗談でしょう?名無しさん:02/11/11 22:24 ID:MozbNg6P
スパナによってねじに加えられた回転方向の力が、
ねじの螺旋部分を通して締め付けられた物体に対し
螺旋部分に垂直に働いていて
(螺旋部分を物体に対し平行であると近似して)、
それに対して静止摩擦力が働くって感じかな?

461ご冗談でしょう?名無しさん:02/11/11 22:35 ID:Wyy7gJsq
物理でプレゼンテーションがあるんですが
どんなトピックが面白くて理解が簡単ですか?
ちなみに課題は生活で使われる物理です。

最初音の作り方を(シンセサイザー等)を調べようと思ったんですけど
全然情報が見つかりませんでした。
お願いします。
462ご冗談でしょう?名無しさん:02/11/11 23:28 ID:MozbNg6P
生活で使われるものと摩擦の関係・・・。
例えばねじとか・・・。
じゃだめかな?

和音はなぜきれいに聞こえるか?
純正律と平均律とは何か?
とか?


463ご冗談でしょう?名無しさん:02/11/11 23:55 ID:jT+GBUvL
>>462
レスほんとに感謝です。
摩擦も調べてみます。
なにせ、木曜日に迫っているもので、、、。
464ご冗談でしょう?名無しさん:02/11/11 23:56 ID:7RzgfjmB
照明関連は?
簡単なものだと、蛍光灯の発光の原理とか。
465423:02/11/12 00:29 ID:9uDTFbP+
453>>あってるよもちろん。
466ご冗談でしょう?名無しさん:02/11/12 00:48 ID:???
>>465
つうか、キミは答えを知ってるのか?
467ご冗談でしょう?名無しさん:02/11/12 00:54 ID:???
>>453
>任意のベクトル場Vには V=-∇φ - ∇XW を満たすWが存在する。がいえる。
そんなこと言えたら、任意のベクトル場Vが必ず∇×V=0を満たすことになるが?

468ご冗談でしょう?名無しさん:02/11/12 00:58 ID:???
質問でーす。可視光望遠鏡で観測できる天体の特徴を教えてくださーい。
469教えれ。:02/11/12 01:04 ID:kIBvEmZL
同じ物質の、
100℃の液体 2mol と 200℃の液体 1molを混ぜたら、何℃になるか教えれ。それを導く公式とか教えれ。
470ご冗談でしょう?名無しさん:02/11/12 01:04 ID:FRdBt53v
>>468
可視光で光っている。
471ご冗談でしょう?名無しさん:02/11/12 01:05 ID:???
>>468
可視光を発する。可視光を反射する。可視光を吸収する。
472ご冗談でしょう?名無しさん:02/11/12 01:07 ID:???
>>469
>>1を100万回読み直して出直せ
473ご冗談でしょう?名無しさん:02/11/12 01:08 ID:???
>>471
吸収は関係ないような
474ご冗談でしょう?名無しさん:02/11/12 01:08 ID:???
>>469
比熱は温度によるので、そのままでは計算不可。
475ご冗談でしょう?名無しさん:02/11/12 01:09 ID:???
>>473
食。
476ご冗談でしょう?名無しさん:02/11/12 01:12 ID:???
比熱一定が多分前提なんだろうが、比熱一定だとしたとき150℃以上に
なるか以下になるかくらいは常識から分かるだろ。まずそれが分からなきゃ
論外だな。
477ご冗談でしょう?名無しさん:02/11/12 01:13 ID:FRdBt53v
>>473
暗黒星雲とか
478ご冗談でしょう?名無しさん:02/11/12 01:17 ID:???
>>468
471+大きな質量がある。
479ご冗談でしょう?名無しさん:02/11/12 01:19 ID:vM8W/EKZ
>>478
渋い
480教えれ。:02/11/12 01:21 ID:kIBvEmZL
>>476
予測はつくが、>>469を求める公式は無いのか?教えれ。
481ご冗談でしょう?名無しさん:02/11/12 01:23 ID:???
>>480
どっちだと思う?あと比熱一定ってことは熱量が温度差に比例する
と思って一次方程式解けばいいんだよ。
482追記:02/11/12 01:27 ID:???
混ぜたあとの温度をx℃とでもして一次方程式解けばいい。
483教えれ。:02/11/12 01:30 ID:kIBvEmZL
>>481
まるでわからない。
馬鹿にもわかるように教えれ。
484ご冗談でしょう?名無しさん:02/11/12 01:31 ID:???
まずどっちだと思うかを答えろ。
485教えれ。:02/11/12 01:33 ID:kIBvEmZL
>>484
では150以上だと賭けた。
486ご冗談でしょう?名無しさん:02/11/12 01:36 ID:???
「論外」のほうだな。20℃の水99mlと40℃の水1mlを混ぜたら温度は
30℃以上になると主張してんのと同じだ。
487教えれ。:02/11/12 01:43 ID:kIBvEmZL
>>486
では150℃以下に賭けた。  
488ご冗談でしょう?名無しさん:02/11/12 01:43 ID:???
あまりに人をなめてるな。サヨナラ。
489教えれ。:02/11/12 01:45 ID:kIBvEmZL
謝るから待て。
式だけでも教えれ。
490教えれ。:02/11/12 01:47 ID:kIBvEmZL
ケチ。
491教えれ。:02/11/12 01:48 ID:kIBvEmZL
ケチは泥棒のはじまり
492ご冗談でしょう?名無しさん:02/11/12 02:06 ID:???
まぁ一番のケチは脳味噌をこれっぽっちも使おうとしないID:kIBvEmZLだな
493ご冗談でしょう?名無しさん:02/11/12 03:26 ID:kg3OTwDe
熱エネルギー(カロリー)の計算式

モナー×比熱×ギコ×重さ×ヒッキー×上昇温度×しぃ
494ご冗談でしょう?名無しさん:02/11/12 03:30 ID:???
100molが失ったモナー=200molが得たモナー
という式を立てて下さい。
495ご冗談でしょう?名無しさん:02/11/12 03:47 ID:kg3OTwDe
>>461
力学
惑星の運動
振り子
★コマとコリオリ力と才差運動
投げたボールが自由落下する軌跡
さまざまな滑車や歯車
遠心力と道路のカーブ
非弾性衝突(バットでボールを打つ)
ベルヌイの定理と飛行機
496ご冗談でしょう?名無しさん:02/11/12 03:53 ID:kg3OTwDe
>>461

雷の光と音がずれるのは、波の伝搬速度が違う
レーザーの周波数とCDのギザギザの大きさ
宝石と様々な偏光
カメラのレンズと、プリズムみたいに色が分かれるのを防ぐ工夫
497ご冗談でしょう?名無しさん:02/11/12 04:00 ID:kg3OTwDe
>>461
電磁気
静電気
様々なモーター
ビデオテープと保磁力
★ビデオデッキのヘッドは何故回転しているの
火を使わずに磁力の力で料理ができる鍋
498ご冗談でしょう?名無しさん:02/11/12 04:04 ID:kg3OTwDe
>>461

熱放射とcpuのファン
カルノーサイクルと発電機や冷蔵庫
499ご冗談でしょう?名無しさん:02/11/12 04:13 ID:???
>>495
コリオリはコマと関係ない。
コリオリが関係するのは、例えば「台風の回転方向」
500ご冗談でしょう?名無しさん:02/11/12 14:21 ID:55fToD6w
地球を半径6,4×10^6[m]の球とし、地表での重力加速度を9,8[m/s^2]として
以下の問いに答えよ
(1)円軌道をえがいている人工衛星の地表からの高さが、ちょうど地球の直径
に等しい時、人工衛星の周期を求めよ
(2)この人工衛星の速さを求めよ
(3)この人工衛星が、進行方向にごく短期間の逆噴射を行ったところ
楕円軌道に移り、その周期が変化した。新しい軌道上で地球にもっとも近い
点での地表からの高さが、地球の半径に等しいとき、新しい周期はもとの
周期の何倍であるか。

(3)についての長半径がわからないのですがよろしくおねがいします。
501ご冗談でしょう?名無しさん:02/11/12 15:11 ID:???
なぜこうも>>1の注意書きを読まずに宿題を丸投げする輩が後を絶たないのか
502(゚Д゚)高校生:02/11/12 15:41 ID:qXXFUMLK
1個のウラン(235.92U)→ヘリウム(4.2He)+Th(231.90)となる。
ウランは始め静止していて、Heの運動エネルギー7.0*10^-13(J)とすると、
Thの運動エネルギーはいくらか?

質量欠損が運動エネルギーの和と考えればいいのでしょうか?
でも核反応の時、ひょっとしたらγ崩壊しているかもしれないですよね?

さらに、Th(231.90)→Pa(x.91)→Ac(y.89)となる。
x、yを決定するにはどうしたらよいのでしょうか?

よろしくおねがいします。
503ご冗談でしょう?名無しさん:02/11/12 15:55 ID:???
>>500
軌道を一周したら同じ場所に戻って来るだろ
そこが遠地点だ

>>502
・起こると書いていないことは起こっていない
・x, yを「求めよ」という問ではないだろそれ
504(゚Д゚)高校生:02/11/12 16:07 ID:qXXFUMLK
>>503さん
>>・起こると書いていないことは起こっていない
了解しました。

>>・x, yを「求めよ」という問ではないだろそれ
いいえ。求めるみたいです。、ここでx、yはPa.Acの質量数である。とは
ありますが、質量や質量数は与えられてないみたいです。
505ご冗談でしょう?名無しさん:02/11/12 16:15 ID:???
>>502
運動量は保存されるのでは?
それから速度を求められる。
506(゚Д゚)高校生:02/11/12 16:22 ID:qXXFUMLK
運動量とエネルギーのホゾンソークの連立でしょうか?
問題見直したら原子質量はひとつもあたえられてませんでした。
これででるかな?
507ご冗談でしょう?名無しさん:02/11/12 16:23 ID:???
>>502
Th->Paでは原子番号が1増えてるね。原子番号が1増えるような崩壊はどういう崩壊で、
そのとき質量数はどうなる?

Pa->Acも同様
508(゚Д゚)高校生:02/11/12 16:27 ID:qXXFUMLK
原子番号+1→ベータ崩壊で質量数は変化しない。でしょうか?
でも、 α崩壊をしたとしても、β崩壊しまくったら、結果として
原子番号+1もありえますよね?

でもそれじゃ問題がとけないのかなあ?
509ご冗談でしょう?名無しさん:02/11/12 16:44 ID:???
>>508
何かヒネクレてるね。1個の矢印は1回の崩壊と考えれ
510(゚Д゚)高校生:02/11/12 16:53 ID:qXXFUMLK
1個の矢印は1回の崩壊と考えれ

いわれてみればそうですね。
逝ってきます・・・・
511ご冗談でしょう?名無しさん:02/11/12 17:10 ID:???
>>506
この問題では崩壊後のαの運動エネルギーが与えられてしまっているので、
あとは運動量保存則だけでOK。

エネルギー保存則の式を作ろうにも、質量欠損が与えられていないので無理。
むしろ、その問題を解いた結果、質量欠損がいくらであったか解る。

512ご冗談でしょう?名無しさん:02/11/12 17:49 ID:???
>>499
もっとよく考えてから書け
513ご冗談でしょう?名無しさん:02/11/12 18:15 ID:???
>>512
よく考えても分かりません
コリオリはコマと関係あるのでしょうか?
514ご冗談でしょう?名無しさん:02/11/12 18:18 ID:???
ビデオデッキのヘッドは何故回転しているの?
何でなの?
515ご冗談でしょう?名無しさん:02/11/12 18:33 ID:WZp2pD/x
後もう少しヒントくれませんか?ほんとに申し訳ないんですけど>>503
516343:02/11/12 18:35 ID:u5oqfgu3
343 :ご冗談でしょう?名無しさん :02/11/08 17:30 ID:SZu3Uwkh
物質中にたまっている電荷って、原理的にはどうやって測定するんですか?
344 :ご冗談でしょう?名無しさん :02/11/08 17:47 ID:???
∇・D=ρ

という回答をいただいたのですが、頼ってばかりで申し訳ないですが
具体的な測定方法も教えていただけませんか?よろしくお願いします。
517ご冗談でしょう?名無しさん:02/11/12 19:03 ID:???
>514
テープとの相対速度を速くするため
518あきちん:02/11/12 19:38 ID:uLTiiui7
テトラエンの吸収する光の波長を考えているのですが,π電子の数が四個まではわかったのですが、
波長の長さが10^3 単位になりおかしくなってしまいます。
誰か計算結果とともに教えてください。
519ご冗談でしょう?名無しさん:02/11/12 20:13 ID:???
>513
ないよ。
520ご冗談でしょう?名無しさん:02/11/12 20:53 ID:mX0KejZy
量子力学の本のスピンの説明の所で必ず
「磁気モーメント」という言葉が出てきますが、
どういう意味なのですか?電磁気の本をみても、
「磁気双極子モーメント」の説明しかありません。
両者の違いを教えてください。お願いします。
521ご冗談でしょう?名無しさん:02/11/12 20:58 ID:???
>>520
量子力学より先に 電磁気学を勉強することをお勧めします
522mnbv:02/11/12 21:05 ID:UQUGReFl
水平面内で角速度ωで回転する半径rの円盤の縁に固定されていた質点
(質量m)が、t=0に縁から離れて飛んでいくとき、それを円盤とともに回
転する座標系から見たとき、質点の運動方程式を求めよ。

相対運動が分からない・・・。誰か教えて下さい。
523ご冗談でしょう?名無しさん:02/11/12 21:09 ID:???
力学の教科書の回転運動の章を読めば必ずわかる。
教科書を持って無かったり読む気が無いのなら立ち去れ。
524520:02/11/12 21:29 ID:nK3GLo4Z
>>520を教えてください。お願いします。
525ご冗談でしょう?名無しさん:02/11/12 21:38 ID:???
このスレ見りゃ数十分でレスがつくことのほうが少ないことくらい分かるだろ。
30分レスがつかなかったくらいであげるんじゃねーよ。
526ご冗談でしょう?名無しさん:02/11/12 22:35 ID:???
>>523
キーワードは「座標系の変換」「回転座標系」
527ご冗談でしょう?名無しさん:02/11/12 22:42 ID:???
528ご冗談でしょう?名無しさん:02/11/12 22:43 ID:???
>>515=500?
逆噴射がごく短時間なので、その時点では人工衛星の軌道はほぼ元のまま。
でも角運動量を失っているのでそこが長径になる。
あとは長径と周期の関係から求める。
529ご冗談でしょう?名無しさん:02/11/12 23:22 ID:???
>>516
ミリカンが電子の電荷を測定した方法は。
たしか油を磁場の中に飛ばして、軌跡を調べたはず。
530ご冗談でしょう?名無しさん:02/11/12 23:41 ID:4GzImokR
>>520を教えてください。お願いします。
531ご冗談でしょう?名無しさん:02/11/12 23:45 ID:???
長半径の長さがわかりました。ありがとうございます。
>>503さん>>528さん
あとageてました。すみません
532ご冗談でしょう?名無しさん:02/11/12 23:57 ID:???
>あとageてました。すみません
普段どういう板に出没しているのか・・・
533ご冗談でしょう?名無しさん:02/11/13 00:12 ID:???
>>529
地場じゃなくて電場だと思うが。
534ご冗談でしょう?名無しさん:02/11/13 00:43 ID:???
>>530
モーメントは理解したの?
理解できたのなら、ちょっと説明してみ
535ご冗談でしょう?名無しさん:02/11/13 01:09 ID:U0AqKBo3
素人のくせに、ニュー速のスレでとある書き込みをしました。
あっているかどうか、天才達君、アドバイスください。

ここの >>201 他214、216です。
http://ex.2ch.net/test/read.cgi/news/1036946883/201-300


【ご紹介】ちなみに俺はここの1です。【ブリブリ】
http://science.2ch.net/test/read.cgi/math/1030368533/
536ご冗談でしょう?名無しさん:02/11/13 01:11 ID:Rsagv1lw
>>520を教えてください。お願いします。
537ご冗談でしょう?名無しさん:02/11/13 01:17 ID:???
質問には答えず一方的に「教えてください」か。放置推奨。
538ご冗談でしょう?名無しさん:02/11/13 01:26 ID:???
質問する厨房の質まで下がってやんの
物理板もほとんど生物板波
もうだめぽ
539ご冗談でしょう?名無しさん:02/11/13 02:04 ID:LPtJgy5M
>>520を教えてください。お願いします。
答えてもらえるまでがんばります。
540ご冗談でしょう?名無しさん:02/11/13 03:46 ID:???
心なき身にも哀はしられけり鴫たつ沢の秋の夕暮
541ご冗談でしょう?名無しさん:02/11/13 07:26 ID:VRdl/VPd
>>520 >>539

521さんの発言のように、電磁気学を勉強するのが
良いように思います。磁気のあたりを。
自分が理解していない部分を、自分で見つける作業が
大事だと思う。
542ご冗談でしょう?名無しさん:02/11/13 08:14 ID:???
>>519
コリオリ力がどうやって発生するか
考えれ
543ご冗談でしょう?名無しさん:02/11/13 08:17 ID:???
>>539
教えてくんは駄目です。
どういう努力をして、どのあたりまで理解して、
どこの部分でつまづいているのか報告して下さい。

本当に答えてほしいのなら。
544あきちん:02/11/13 08:57 ID:B3YhaGgN
=518
何とか自力でテトラエンの問題を解きました。
というか、一切無視だったのは問題が難しかったのでしょうか??
色んな意味で寂しかったです。
545ご冗談でしょう?名無しさん:02/11/13 11:54 ID:???
>544
甘えすぎ
546ご冗談でしょう?名無しさん:02/11/13 12:03 ID:???
>542
コリオリの力の大きさがどれくらいか
考えれ
547ご冗談でしょう?名無しさん:02/11/13 12:05 ID:Gq8OcFmD
>>520を教えてください。お願いします。
548ご冗談でしょう?名無しさん:02/11/13 12:24 ID:???
549ご冗談でしょう?名無しさん:02/11/13 12:41 ID:LPtJgy5M
電磁気の本に磁気モーメントがありません。
双極子ならありますが。同じものなんですか?
もし違うなら>>520を教えてください。お願いします。
双極子モーメントの方は理解しています。
550ご冗談でしょう?名無しさん:02/11/13 12:49 ID:EBDOmKhm
写真はファンデルワールスですかって質問まじめに書いて
今日見たら、堂々とWan Del・・・って書いてた。

                          はず歌詞。
551ご冗談でしょう?名無しさん:02/11/13 13:47 ID:???
>>549
双方の定義を見比べて自分で確かめればいいだろう
552ご冗談でしょう?名無しさん:02/11/13 13:53 ID:avsWE1zc
>>549
M=mL
M:磁気モーメント[Wb・m] m:磁荷[Wb] L:棒磁石の長さ[m]
553ご冗談でしょう?名無しさん:02/11/13 15:50 ID:???
無限に広がる平行平板導体間に誘電率εをもつ誘電体を入れた場合の、単位面積あたりの静電容量を求めたいんですけど、
C=εS/d
の式に単位面積S=1を代入した
C=ε/d [F]
で答えは合ってるんでしょうか?
554ご冗談でしょう?名無しさん:02/11/13 16:33 ID:???
>>529
磁場じゃなくて電場です。電場をかけた場合とかけない場合で油滴のドリフト速度を比較した。
555ご冗談でしょう?名無しさん:02/11/13 16:44 ID:???
>>553
単位に気をつけないと。
556ご冗談でしょう?名無しさん:02/11/13 16:46 ID:???
>>546
もしかしてコリオリ力は地球の自転によるものしか眼中にない?
コマの自転運動に乗った回転座標系で現れるコリオリ力は重要な寄与をするよ
557ご冗談でしょう?名無しさん:02/11/13 17:26 ID:YHfQ5ppm
密度1.05g/立方cmの食塩水を入れた水槽の中に、一辺の長さ10cm
のアルミニウムの立方体が沈められている。このアルミニウムの受ける浮力
(g)はいくらか?
558ご冗談でしょう?名無しさん:02/11/13 17:40 ID:???
皆さん
丸投げ質問にはNoレスを!

>>557
それはAlの密度が分からないと解けません。
559堅苦しいこというなYO!:02/11/13 17:55 ID:YHfQ5ppm
>>558
アルミの密度は書いてありません。工学適性検査の問題です。
560ご冗談でしょう?名無しさん:02/11/13 17:57 ID:???
>>549
> 双極子モーメントの方は理解しています。

ちょっと説明してみれ
561ご冗談でしょう?名無しさん:02/11/13 17:58 ID:???
>>558
浮力の算出だからアルミの密度はいりません。

丸投げ無視は賛成
562ご冗談でしょう?名無しさん:02/11/13 18:06 ID:YHfQ5ppm
放置プレイ・・・・・きゃはは〜
563ご冗談でしょう?名無しさん:02/11/13 18:08 ID:???
>>561
「アルミの密度はいりません」て、あ〜そうだったんかいな。
>>557
すまん間違えた。溶かした食塩の量が分からないと溶けません。

564ご冗談でしょう?名無しさん:02/11/13 18:09 ID:YHfQ5ppm
あらし〜荒らし〜オーイェイ
565ご冗談でしょう?名無しさん:02/11/13 18:11 ID:YHfQ5ppm
フェローに聞いてみます。
566ご冗談でしょう?名無しさん:02/11/13 18:27 ID:???
>>563
それもいらない

丸投げ虫は酸性
567ご冗談でしょう?名無しさん:02/11/13 18:38 ID:???
>>561,>>563
一般の人は「浮力」と聞くと、多分 (浮力)−(重力) をイメージするんだろうね。
だからどうだ、というわけじゃないけど。
568ご冗談でしょう?名無しさん:02/11/13 18:42 ID:???
>>566
こんどこそ!
アルミの水中における体積が重要なので、アルミの弾性係数。
569ご冗談でしょう?名無しさん:02/11/13 18:51 ID:???
>>568
それがあると嬉しい
570ご冗談でしょう?名無しさん:02/11/13 18:57 ID:???
>>569
ワーイ、やったー! これで問題解決だ。
良かったな〜>>557
571ご冗談でしょう?名無しさん:02/11/13 19:03 ID:???
>>568
なぜ水槽が置かれている場所の重力加速度を指摘しない?
572ご冗談でしょう?名無しさん:02/11/13 19:28 ID:vbqI6GON
>529,554
ありがとうございます。確かにミリカンはそれで素電荷は測っています。
しかしミリカンは小さい球状の油滴で測定しているため、簡単な
ローレンツ力の計算ですむのですが、今回の問題は一般の形状、大
きさの帯電体についての電荷を図るという問題ですのでその方法ではは
かれないと思います。申し訳ありませんがこの場合はどうすればいいのか
お願いします。
573553:02/11/13 19:29 ID:???
>>555
Cの単位はファラド(F)じゃないんでつか?
574ご冗談でしょう?名無しさん:02/11/13 19:37 ID:???
単位面積あたりだったら面積の次元で割らないと。
575553:02/11/13 19:41 ID:???
>>574
回答自体間違ってます?
面積の次元で割ると言われても、広さが無限大なんで…。どーしたらいいんでしょうか。
教えて君でスンマセン
576ご冗談でしょう?名無しさん:02/11/13 19:53 ID:???
例えば密度っていったら g/cm^3のような次元でこれは単位体積あたり
の質量でしょう。密度をgとかkgとかいう次元で書いたらおかしいでしょ。
577553:02/11/13 20:00 ID:???
>>576
なるほど。なら答えは
C=ε/d [F/m^2]
でつね。ありがとう!
578ご冗談でしょう?名無しさん:02/11/13 20:06 ID:???
検索エンジンへの登録代行の宣伝リンクスタッフを
募集しています。今ならバナーを貼っていただくと
1000円のキャッシュバックがあります。
http://accessplus.jp/staff/in.cgi?id=3973
579ご冗談でしょう?名無しさん:02/11/13 20:09 ID:???
質問です
微分回路と呼ばれるものと、積分回路と呼ばれるものがありますが、
それぞれ何故そのような名前で呼ばれているのでしょうか?
580ご冗談でしょう?名無しさん:02/11/13 20:13 ID:???
入力信号を微分ないし積分した波形に近い出力が得られるから
581ご冗談でしょう?名無しさん:02/11/13 20:17 ID:???
>580
ありがとうございました
582ご冗談でしょう?名無しさん:02/11/13 21:14 ID:ylSodMHi
物体の軌跡からその物体に働く力の大きさと向きを出すには
どうしたらいいのでしょうか?
583sage:02/11/13 21:17 ID:ylSodMHi
初歩の質問かもしれないので申し訳ありませんが
物体の軌跡から物体に働く力とその向きを出すには
どのようにしたらいいのでしょうか?
584ご冗談でしょう?名無しさん:02/11/13 21:17 ID:tF+SczN6
2回時間微分して加速度を求める。
585ご冗談でしょう?名無しさん:02/11/13 21:19 ID:???
運動方程式 ma=Fから。
586ご冗談でしょう?名無しさん:02/11/13 21:18 ID:???
すみません、意味不明の2重カキコになってしまいました
587ご冗談でしょう?名無しさん:02/11/13 21:21 ID:???
>584さん>585さん
ありがとうございます
588520:02/11/13 23:02 ID:psdtztCm
>>552
ありがとうございます。
でもどうして電磁気の本に書いてないんですかね?
入門コースとテキストシリーズみたんですが。
589もうちょっと詳しく説明して:02/11/13 23:14 ID:???
>>572
っていうか何がしたいの?
具体的に説明して欲しい。
590ご冗談でしょう?名無しさん:02/11/13 23:22 ID:???
>>588
> でもどうして電磁気の本に書いてないんですかね?

>>549
> 双極子ならありますが。

書いてあったんじゃないのか?
591ご冗談でしょう?名無しさん:02/11/13 23:24 ID:nODs1Va0
2ch kondo effect って何ですか?
592ご冗談でしょう?名無しさん:02/11/13 23:32 ID:???
普通の近藤効果は知ってる?知らないなら近藤効果で検索いっぱい
引っかかるからそれで調べて。
593ご冗談でしょう?名無しさん:02/11/13 23:35 ID:lEk8bZoD
ヒッグス粒子:発見目指し産学で研究会 実験施設建設を計画
http://www.mainichi.co.jp/news/flash/shakai/20021114k0000m040118000c.html

>電子などの素粒子は本来質量ゼロ、ヒッグス粒子と反応
そうすると電子などの素粒子には本来エネルギーもないし、
運動エネルギーを与えることも不可能なのか?

よくわからなくなってきますた。ヒッグス粒子についてご教授ください。
594ご冗談でしょう?名無しさん:02/11/13 23:36 ID:cc7KkTBX
595ご冗談でしょう?名無しさん:02/11/13 23:40 ID:???
>>593
質量0でもエネルギーは持ちえますが。
596591:02/11/13 23:42 ID:???
伝導電子と局在電子の相互作用で局在スピンが打ち消される
ということしか知りません。
低温で電気抵抗が上昇する現象に関連しているんですよね?
2chとは??
597ご冗談でしょう?名無しさん:02/11/13 23:42 ID:lEk8bZoD
>>594
??
598ご冗談でしょう?名無しさん:02/11/13 23:43 ID:???
普通はmultichannelって言い方だと思う。
599ご冗談でしょう?名無しさん:02/11/13 23:44 ID:lEk8bZoD
>>595
Σ( ̄□ ̄;マジ?
600ご冗談でしょう?名無しさん:02/11/13 23:47 ID:p3/OhY7q
荷電粒子砲って作れるの?実際。
できるとしたらどのくらいの電力でどのくらい破壊でき
るの?
601ご冗談でしょう?名無しさん:02/11/13 23:48 ID:???
>>599
光ってエネルギーあるでしょ。(エネルギーがなかったら”見え”ない)
602ご冗談でしょう?名無しさん:02/11/13 23:50 ID:psdtztCm
>>590
双極子はあるのにどうして磁気モーメント
はのってないのか、という質問ですよ。
603ご冗談でしょう?名無しさん:02/11/13 23:51 ID:lEk8bZoD
>>595
その場合、

E=mc^2 E:エネルギー m:質量 c:光速

はどう説明できるのでしょうか?
604HAL ◆2eOaL2U.Gg :02/11/13 23:58 ID:???
>>603

大学生ですか?いろいろな説明がありますが簡潔なものは、

E := p^μ u_μ

でエネルギーの定義です。pが四元運動量ベクトルで、uが
観測者の四元速度ベクトル。
観測者瞬間静止系をとれば、

E=cp^0

pの成分は、p^μ=(hν/c,・・・)

ですから、

E=hν

あなたの言う式は、静止エネルギーですよ。
一般化した上の式を覚えておきましょう。

605しょうちゃん:02/11/14 00:01 ID:UIEerBrR
606HAL ◆2eOaL2U.Gg :02/11/14 00:04 ID:???
>>600
どういう仕様でつくるんですか?
何を発射するの?電子??

磁場をはれば、ドリフト運動するので容易にシールドできて意味なさそう。
必要な磁場の大きさは、荷電粒子の発射速度をVとすると、

(1/ω_c) V = R
ω_c=eB/γm (MKS)
γ=1/√1-(v/c)^2

⇒ B=γmV/eR

Rにシールドの典型的サイズをぶち込めば、必要な磁場の
大きさが見積もれるぞ。
607ご冗談でしょう?名無しさん:02/11/14 00:04 ID:FVVZH2IN
>>603
親切にありがとうございます。

静止エネルギー、運動エネルギー、全エネルギー、肝に銘じておきます。

上記説明と光電効果の項を見直してみたらなんとなく納得がいきました。
どうも、ご迷惑をおかけしました。
608ご冗談でしょう?名無しさん:02/11/14 00:04 ID:???
>>593
higgs機構でぐぐれ
609ご冗談でしょう?名無しさん:02/11/14 00:05 ID:FVVZH2IN
>>608
どうもです。調べてみます。
610HAL ◆2eOaL2U.Gg :02/11/14 00:06 ID:???
>>607

実用上は

E=cp_0が使いやすいです。

さらに、普通の物質ならば、

E=γmc^2(全エネルギー)

を導きます。これも覚えとくと役立ちます。
もちろんv=0で静止エネルギーに一致します。

611ご冗談でしょう?名無しさん:02/11/14 00:12 ID:???
>>602
その質問は、
結局おまえが磁気双極子モーメントを理解していない
ことに起因すると多くの回答者は考えており

521である俺もそう思っている

もしかしたら おまえは モーメントって何ものか知らないのでは?

じゃ 力学からだな
612ご冗談でしょう?名無しさん:02/11/14 00:44 ID:???
>>604
なんかよー分からんが、普通、E^2=m^2+p^2じゃないか?
これと、 E := p^μ u_μがうまく一致しないようだが??
おれの計算ミス???
613美保子:02/11/14 00:57 ID:???
 私は広島大学で3成分(NaCl−H2O−CO2)系ににおいてそれぞれ3つの成分のモル分率が変化するときの三角ダイアグラムを書くと言う研究を行っております。
しかし私はこれまで遊んでばかりでやっとここ1ヶ月熱力学についての勉強を始めたところなんです。教授にも卒業が危ないって言われちゃって。それで唐突なんですけどお願いがあります。
熱力学に詳しい方相談に乗って頂けませんか?
614ご冗談でしょう?名無しさん:02/11/14 00:57 ID:???
授業の実験でのことなんですが
交流回路で、周波数と各素子の関係が理論どおりに行かないのは何か理由があるんでしょうか?
それとも俺の実験がだめぽなんでしょうか?
一応、抵抗等測定したのですが、
LR回路 V/I=1444 理論値=2200等ずれが・・
615美保子:02/11/14 01:03 ID:???
 613のみほこなんですけどいまの所状態方程式からNaClsolution中の各モル
分率に対するCO2のhenry's law constant は分かるんです。これからある温度における液相と気相の割合を求める式ってできるもんなんでしょうか?
 ほんとにやり方分からなくって、アドバイスお願いします.
616ご冗談でしょう?名無しさん:02/11/14 01:06 ID:???
>>614
> 交流回路
この説明で、どういう回路か理解できたら神。
回路図そうとうの情報を提供せよ。

発振回路なのか?
617ご冗談でしょう?名無しさん:02/11/14 01:11 ID:???
>>615
圧力は?
618大鳥居☆つばめ ◆dpLvO.T.cY :02/11/14 01:13 ID:IKDRi7Wc
>>614
測定器に容量があるとかじゃないのか?
619ご冗談でしょう?名無しさん:02/11/14 01:16 ID:???
>>615
Gibbsアンサンブル法?
620614:02/11/14 01:21 ID:???
ごめんなさい
オッシレータ、コイル、電流計(内部抵抗あり)を直列で繋ぎIを測定。
コイル〜電流計の電圧もはかります。

>>618
容量?
621ご冗談でしょう?名無しさん:02/11/14 01:22 ID:???
今夜は都心も晴天で星が綺麗だ。
星座は太古の形と少しも変わらない。
諸君、今すぐ窓外の空を見上げて卑小なおのれの
有り様を実感せよ。
モル分率がなんだ、LR回路がどうしたというのだ!
622ご冗談でしょう?名無しさん:02/11/14 01:25 ID:???
>>615
二成分系の平衡は理解できていますか?
623ご冗談でしょう?名無しさん:02/11/14 01:26 ID:???
コイルのインダクタンスも測定したのかな?既知として使ってるのかな?

>>621
物理法則も太古の昔から変わらないから帰れ。
624614:02/11/14 01:29 ID:???
>>623
レスありがとうございます。
コイルのインダクタンスは既知のものとして使いました。

実験の前に、「この実験は理論どおりにいかないからね」と教授がぼそぼそ言っていたので理由が気になるんです。
625ご冗談でしょう?名無しさん:02/11/14 01:36 ID:???
>>614
位相が45度ずれているのが理解できていないような気がするが、
きっと気のせいだろう。

理論値=2200←これをどうやって計算したのか、書いてみて
626大鳥居☆つばめ ◆dpLvO.T.cY :02/11/14 01:36 ID:IKDRi7Wc
>>621
きらめく星座が〜おまえをよんでる〜、これが選ばれた〜戦士のあかし〜♪

ギブスエネルギーの変化が0
627ご冗談でしょう?名無しさん:02/11/14 01:39 ID:wtxhuVLZ
>>620
> オッシレータ、コイル、電流計(内部抵抗あり)を直列で繋ぎIを測定。

全部直列でつないで数値がおかしいのなら、
まずは、1個1個、個別に測定してみる。

あと、周波数教えて。
628大鳥居☆つばめ ◆dpLvO.T.cY :02/11/14 01:39 ID:IKDRi7Wc
>>621
タイムスリップした場合、見える星座の状況で、いつに来たかが分るようですね。
629ご冗談でしょう?名無しさん:02/11/14 01:43 ID:???
抵抗 周波数 known でコンダクタンスがunknownなら実験から
コンダクタンス求めて次に抵抗変えて実験してみる。その結果が
合うか?
630ご冗談でしょう?名無しさん:02/11/14 01:50 ID:wtxhuVLZ
>>614
> LR回路 V/I=1444 理論値=2200等ずれが

理論値と実測値のずれが約1.4倍ですね。
多分、次の授業で、何故ずれたのかの説明があります。
事前に知りたければ、教科書を読んでみましょう。
631ご冗談でしょう?名無しさん:02/11/14 01:54 ID:???
「理論と合わない」ってそういうことなのかな?学部何年の実験か知らんけど。
632ご冗談でしょう?名無しさん:02/11/14 01:56 ID:oYaGU+aN
一次元ポテンシャル中を運動する粒子の波動関数の連続性と有界性はどのように
して保障されるのでしょうか?有界性は不確定性原理からだと思うのですが、
連続性がよくわかりません(例えばδ関数型ポテンシャルの場合でも波動関数
の連続なのか?など)
誰か明確な解答を御願いします。
633614:02/11/14 02:44 ID:???
二重すまそ
>>618
LR回路のほかにLCR回路についても同じ機器を用い実験を行いました。
共振する振動数の実測値と理論値がほぼ同じでした。
計測器に容量があるとしたら振動数等もうちょっとずれてくる気がするのですが。
634614:02/11/14 02:52 ID:???
周波数 100k
R,L,Cすべて既知です。

LC回路での2200計算
VとIを直列につないだ場合、
L*dI/dt=V
V=Vm * e^(jωt+α)
よってI=V/jωL
オームの法則と比較して、コイルをjωLの抵抗を持つ素子として計算しました。
レス遅れてごめんなさい
635ご冗談でしょう?名無しさん:02/11/14 03:04 ID:???
>>634
まさかとは思うがRとLの抵抗値をそのまんま足したのではないか?
自乗平均しないで。
636ご冗談でしょう?名無しさん:02/11/14 03:28 ID:hG2qkjAt
レスありがとうございます

>>635
RLCについて、電流についての微分方程式を立てると、
I=V / (R+j(wL-1/wC))となります。
つまり、抵抗=R+j(wL-1/wC)

RL回路については解いてませんが、おそらく同じようになると思います。
そのまま足してよいのでは?

ωをwで表記しました
637ご冗談でしょう?名無しさん:02/11/14 03:31 ID:hG2qkjAt
追記
過渡現象をを無視し、t→∞で
q=A exp(-Rt/2L) sin(w't+α) +V/(-Lw^2+1/C)+jwR
一項を消しています。
638ご冗談でしょう?名無しさん:02/11/14 03:36 ID:???
計算してみるから
V,R,L,Cそれぞれの値を教えて下さい。
639ご冗談でしょう?名無しさん:02/11/14 03:45 ID:hG2qkjAt
なんかすごいミスをした気が・・
大きさは2乗して・・・すいませんでした。

>>638
ありがとうございます。
LR回路
L=3.5mH R=74.61Ω
f=1.0*10^4
V=770.3mV
I=3.2mA
640ご冗談でしょう?名無しさん:02/11/14 04:09 ID:???
>>639
やってみましたが計算が合いませんでした
f=10KHZ
では3.5mHのリアクタンスは220Ω
ですからR=74.6Ωと直列のインピーダンスは自乗平均で
232Ωになります。
一方実測値のv=770mVでi=3.2mAならインピーダンスV/i=240Ω
になるのでそう違わないのでは?
641ご冗談でしょう?名無しさん:02/11/14 04:25 ID:???
>>640
ありがとうございます。
自分>>637で変なこと言ってますが一応理論値は二乗平均で出しておりました。

f=100KHZ
V=2186.6
I=1.5のデータを書けばよかった・・・
一応計算方法は同じなので実1444と理2200が出てくるはずです…。(LとRは>>639と同じ)

……わからなくなってきた。
こんど授業で聞いてきます。夜遅くすいませんでした。
ありがとう。
変なことばっかり言ってスマソ、もう寝ます。
642ご冗談でしょう?名無しさん:02/11/14 04:46 ID:???
>>641
f=100khzではLの値がデカイから理論2200でよいですね。

そうか、わかったぞ、
電流をオシロで読みとってるんではないか?
波高値を実際値(√2分の1)と勘違いしたに相違ない!
1.5mAを波高値とすると実効値は√2分の1だから
インピーダンスはじつは1444の√2倍で
理論値と合うじゃないか。

もう寝ただろうから勝手なこと言っちゃって、ゴメン。
643ご冗談でしょう?名無しさん:02/11/14 08:24 ID:???
学習組に早々解散命令を出したのは
破防法適用逃れの為
http://www.sankei.co.jp/news/020902/morning/02iti001.htm


644520:02/11/14 13:58 ID:gsBozQCJ
結論としては、磁気双極子モーメントと
磁気モーメントは同じものなんですね?
645ご冗談でしょう?名無しさん:02/11/14 14:17 ID:???
磁気双極子モーメント:極性ベクトル
磁気モーメント:軸性ベクトル
646迫る本番!大学受験生:02/11/14 14:41 ID:pNBYuzWR
交流回路で普通にただ「コイルにかかる電圧を求めろ」「電流を求めろ」
って言われたら、瞬時値じゃなくて実効値で答えるんですか?
この瞬間に流れてる電流とかを求めるのかと思って瞬時値出そうとしたら電源の関数が与えられてなくて、ハァ?
って思って回答みたらサラッと実効値で出してあったんで‥
647ご冗談でしょう?名無しさん:02/11/14 15:21 ID:???
電源の関数が与えられてなかったら実効値だって出せないじゃん。
648大鳥居☆つばめ ◆dpLvO.T.cY :02/11/14 15:29 ID:N0O+Ya1w
>>646
分らなかったら、時間変化も最大値も実効値も全部、書けばいいと。
649ご冗談でしょう?名無しさん:02/11/14 15:36 ID:???
普通、計算問題だけなら混同はないようになっているけど
今回の641の件は測定値と計算値の比較で、
食い違いがちょうど√2であることから、もしかして
計測器の読みの問題かなと思っただけ。
650ご冗談でしょう?名無しさん:02/11/14 15:38 ID:86aVjVFF
651受験生:02/11/14 15:41 ID:9LShRMmY
関数は無かったけど、電源とRCL回路のRにかかる効電圧は与えられてました
やっぱ普通なら「実効値で」とか「瞬時値で」
ってあるべきなんですよね?
652ご冗談でしょう?名無しさん:02/11/14 15:52 ID:???
>>651
受験生は心配しなくても大丈夫。
どちらで回答するのか分からない問題は
出っこない。もし出たら欠陥問題として
みんな正解にしてくれるし。w
653ご冗談でしょう?名無しさん:02/11/14 16:10 ID:Qj/IFSgS
屈折率Nの媒質Aが屈折率N”の媒質Bにはさまれた層状の光学素子がある。
ただし、外側の空気の屈折率を1とし、N>N">1であるとする。
(1)外側から入射角Mで光が媒質Aに入射したとき、屈折角αと入射角M
との関係を求めよ。
(2)媒質Aに入射した光は媒質Bとの境界面で一部が反射し一部が媒質B
に入る。光が媒質Bに入るときの屈折角βと角αの関係を求めよ。
(3)光が媒質Aに入射するときの角度Mがある条件を満足すると、媒質A
に入った光は媒質Bとの境界面で全反射して媒質Bには入らない。
その必要十分条件をとめよ。

についての、答えをだす仮定から分からないので、詳しい質問から誰かお
願いします。
654ご冗談でしょう?名無しさん:02/11/14 16:10 ID:83s9iJfR
そうですよね?
やっぱ指定してきますよね!
この問題集が手を抜いただけか‥
もしかして交流を扱う時は実効を使うのが常識だからわざわざ指定しないのかと思いました(^o^;
655ご冗談でしょう?名無しさん:02/11/14 16:14 ID:1ZjlNGfV
>>1こと子豚よ・・・いい加減にしたらどうだ(WWWW
お前は今までずっとそうやって生きてきたな(WWW自分にとって都合のいい話しか聞かない豚野郎(WW
不快になる、反論される話は全く聞く耳を持たないんだろ(WW豚きわめりだな(WW
なぜ自分の卑小さを省みず、常にそんな傲慢な態度をふるえるんだ(WW
お前のような豚は、常に自分の精神状態を気持ちよくする事しか考えてないからだろ(WW
この世を自分中心…豚中心(WW豚世界(WWお前が人間と会話する時は、論議するとか、
意味のある話をしようとか、そういうのがまったくない。ただアフォ豚が気持ちよくなれればそれでいい
自己豚満足しか頭に無い、典型的オナニー豚(WWW
まさに幼児豚がする会話。幼稚豚の典型(Wお前の話は、ゴミだよ豚ちゃん(WWW
イカレ豚のオナニーその最もたるは、お前が書いてきたレスだよ。そして自演ことバレ豚芝居(WWW
なぜあんなレスをしかできない?あんなサトラレ豚芝居をする?自分ではわからないだろうな(W
それは、ただ自分が気持ち良くなりたいという豚望の結果ですよ(WWW
真正面から否定する文は、オナニー豚には通用しまい(Wお前は誰にも論破できない(WW
論破できないというよりは、議論自体できない訳ですが(WWW
豚は豚を不快にする文を受け入れられるような理論的人間じゃないからだ。
つまりオマエが豚だからだよ豚野郎(WWW話の通じない狂豚。狂気豚見参(WWW
ハナから戯言と決めつけることによって、どんなことをいわれても豚の精神状態を
安定させようとする。豚に都合の悪い事は見えません、豚目、豚耳、豚口(WWW
豚のお前にしてみれば、豚が不快になる文は、「バカじゃん」「ただのキチガイ」「で?」で済まされてしまうだろう(WWWそんな事をしていては、他人と論ずる事などできる訳がない(WWW論ずる事など元からアフォ豚にはできませんが(WWできる事はコピペと豚芝居(WWW
とどのつまり、豚ちゃんはハナから他人と論ずるだけの脳味噌を持っていないってこと。
そして、そのレスはすべて何の価値も持たないゴミだということだ。
オマエには何にもできないよ豚ちゃん。ネタ職人などと都合の良い冠が欲しいのか?(WWW
656キャット忍伝:02/11/14 16:44 ID:fwwclFXW
質問です。
コイルを二つくっつけたトランスってありますよね。
コイルをつないでいる鉄芯は鉄板が重なって出来てるじゃないですか。
Why?何故に?
重ねる必要あるんですか?鉄塊じゃだめなんですか?
657ご冗談でしょう?名無しさん:02/11/14 16:54 ID:???
>>656
だめです。渦電流が流れて鉄が熱せられる。
珪素鋼板は磁束を通すけれど重ねてある縦方向には
電流を流さないためのものです。
658キャット忍伝:02/11/14 17:09 ID:fwwclFXW
>>657
ありがとうございます。
わかった気がします。
659ご冗談でしょう?名無しさん:02/11/14 17:10 ID:Qj/IFSgS
>>653
だれか答えて下さい。
660ご冗談でしょう?名無しさん:02/11/14 17:33 ID:???
>>659
これって教科書の光の屈折の項の一番最初に
でてくるはなしだよ。
スネルの法則そのまんま。凄い単純な話だから
そっちを見てね
661ご冗談でしょう?名無しさん:02/11/14 17:41 ID:wtxhuVLZ
>>624
>「この実験は理論どおりにいかないからね」と教授がぼそぼそ言っていたので理由が気になるんです。

俺も気になる。
662ご冗談でしょう?名無しさん:02/11/14 17:53 ID:???
>>661
そーなんですよ。実効値と波高値の取り違え
なんて事を教授が予想するわけないしね。
それに電圧の測定も電流と同じだろうから
両方そろって取り違えていたら答えは正しくなるし。
それはわかっていたけれど俺も眠かったもんで・・・・
663391:02/11/14 20:32 ID:nfadsPWQ
>>392-394
今日、物理化学の教授に聞いたところ、断熱材を用いた場合の、(閉鎖系にする仕事)は、状態量らしいです。
例として、エンタルピーを例に挙げて説明されました。「エンタルピーは、あくまで熱であるけども、しかし状態量だ。」と説明されました。
つまり、熱も仕事も場合によったら、状態量らしいです。
664ご冗談でしょう?名無しさん:02/11/14 20:39 ID:???
凄い教授だ
665ご冗談でしょう?名無しさん:02/11/14 20:41 ID:???
そりゃある状況下での仕事や熱に等しくなるような状態量を定義して
いるんでしょ。エンタルピーとかヘルムホルツの自由エネルギーとか。

ある状況下でされる仕事がある状態量の変化に等しいことと仕事が状態量
であることは違うよ。
666ご冗談でしょう?名無しさん:02/11/14 20:44 ID:???
>>664-665
百科辞典から、

物質系の巨視的状態について測定できるいろいろな性質の間の関係は熱力学によって定量的、かつ数学的に表現することができる。この場合、系の条件、したがって状態を固定させるか、変化させるために十分に直接的に操作できるような量を状態量という。
普通は状態量として意味があるのは温度、圧力、組成だけの場合が多いが、特別な場合には機械的、電気的、磁気的、重力、あるいは表面などのポテンシャルが重要となることもある。(抄録)
667ご冗談でしょう?名無しさん:02/11/14 20:49 ID:???
よく分からなくなってしまいました。
ある状態を写真に撮影して、別の場所でその写真から元の状態を
再現したら、それは状態量が等しくて、
温度がなんで状態量でないかというと、原子の動きのスピードは
写真には写らないからで、
仕事がなんで状態量でないかというと、力×距離だから

みたいなイメージですか?
668大鳥居☆つばめ ◆dpLvO.T.cY :02/11/14 20:51 ID:fddpR5wI
数学的には、
状態量の関数は状態量(例:H=U+PV)で、
状態量の微分(例:dQ=TdS)は状態量になるとは限らない。
669ご冗談でしょう?名無しさん:02/11/14 20:55 ID:???
物質系の状態によって定まり、直接測定の対象となる量。

例えば熱力学的な平衡状態において定まった値をとる
エネルギー・体積・圧力・温度・エントロピーなど。

熱や仕事は状態の変化に伴う量であって状態量ではない。
670ご冗談でしょう?名無しさん:02/11/14 21:00 ID:???
>>668
数学的には、
「10℃+20℃=30℃」です。
数学では、同じ単位は足すことが出来るからです。

しかし、物理的には、間違いです。
数学は、物理によって意味を与えられるものです。
言いかえれば、数学のみによって、嘘か真実なのかの判断は出来ないです。
671ご冗談でしょう?名無しさん:02/11/14 21:03 ID:???
http://www.ms-5.mech.kobe-u.ac.jp/weblec/frame/main/003/003.htm

状態量の説明は、ここがよさげ。

エンタルピ、エントロピは状態量だが、
熱量は状態量じゃないです。
672ご冗談でしょう?名無しさん:02/11/14 21:31 ID:???
とりあえず「温度が状態量でない」ってのはまったく誤解。
673大鳥居☆つばめ ◆dpLvO.T.cY :02/11/14 21:39 ID:???
>>667
温度は状態量だけど、そのイメージは合ってると。

>>670
そうですね。言ってみれば、>>668は物理から導いた数学的な必要条件ですね。逆が成り立たない。
674HAL ◆2eOaL2U.Gg :02/11/14 21:39 ID:???
状態量とは、次のような写像のことです

F:熱力学的状態→R

つまり、熱力学的状態を指示すれば、それに対応する関数値が
一意に決まる関係のことです。

f(状態)=実数

となるもののことです。
「状態」は有限個の変数で指定されます(気体では、V,T)等。
では、実際には変数が状態指示する個数より多い場合は、
その変数間の関係が存在することを意味します。
これが、「状態方程式」です
(たとえば、気体では、P,V,Tの3個の変数に対して2個しか、
状態を指示するのに必要ありません。この変数の間には
関係(理想気体ではPV=NkT)があることを意味します。)
675ご冗談でしょう?名無しさん:02/11/14 21:46 ID:Bm8KsErm
皆さんアドバイス下さい。

交流モータがある程度の安定した負荷のある回転状態から、
空回り状態(脱輪・空転状態)になったとします。
その時モータの電源ラインの電流値・電圧値はどのように
変化するのでしょうか?

私の考えだと電圧一定で電流値が上昇すると思うのですが....。
皆さんいかがお考えでしょうか?
676ご冗談でしょう?名無しさん:02/11/14 21:52 ID:???
>>675
無負荷になってなんで電流がふえると思うのか?
反対に減少する。正確に言うと励磁電流は
変わらずに有効電流が減少する。
677HAL ◆2eOaL2U.Gg :02/11/14 22:21 ID:???
>>612
>なんかよー分からんが、普通、E^2=m^2+p^2じゃないか?
>これと、 E := p^μ u_μがうまく一致しないようだが??
>おれの計算ミス???

ごめん。見落としてました。符号の間違いかな?
v^μ := d(x^μ)/dτ
(⇒ v^μv_μ = -c^2 )
p^μ := mv^μ
⇒p^μp_μ = -m^2c^2
p^μ= (p^0, vec P )

-m^2c^2 = -(p^0)^2 + P^2
⇔ (p^0)^2 = P^2 + m^2 c^2

u^μ=(c,0,0,0)をとる。
E = - p^μu_μ = cp^0

∴ E^2/c^2 = P^2 +m^2 c^2

もちろんc=1(自然単位)で、 E^2 = P^2 +m^2
678HAL ◆2eOaL2U.Gg :02/11/14 22:24 ID:???
>>612

ああいう書き方をしたウマミは、
「力学的エネルギーという量は、観測者を指定しなければ決まりませんよ。」
ということをいってるわけ。観測者さえ決めれば、vectorと1-formの縮約
なのでローレンツ不変な量となるわけだ。
679ご冗談でしょう?名無しさん:02/11/14 23:11 ID:2b4g03WN
ジュールの法則によって、ジュール熱が計算できますが、この式は電球のように
光を放つ器具に対しては使ってはいけないのでしょうか。

その場合、普通の電熱線に大電流を流すと赤熱しますが、こうした時にも
ジュールの法則は適用できなくなるのでしょうか。
680ご冗談でしょう?名無しさん:02/11/14 23:20 ID:???
>>679
適用して問題無し。
681679です:02/11/14 23:28 ID:2b4g03WN
ということは、消費電力が等しければ発熱量も等しいということになりますが、
光のエネルギーはどこから出てくるのでしょうか?
682ご冗談でしょう?名無しさん:02/11/14 23:31 ID:???
>>681
熱。
683679です:02/11/14 23:36 ID:2b4g03WN
ということは、
一旦ジュール熱として放出されたエネルギーが光エネルギーに変わる。
ということなのでしょうか?
それでしたら納得いたしました。ありがとうございました。
684海洋物理学:02/11/15 00:03 ID:playjumt
ちょっと分野が違うのですが(土木学科の科目)、海の流れにおいて
鉛直方向の流速分布が分かっているとき摩擦速度U*と粒径ksを求めることができるらしい
のですが どうやって求めるのですか?かなりマニアックなのでご存知ない方は放置Playで
いいです
685ご冗談でしょう?名無しさん:02/11/15 00:16 ID:60umVoJ2
高校の物理では、質点の地球表面からの脱出速度を求めるときに、
質点には地球との万有引力だけが働くと仮定…(*)した上で、
「地球表面から質点を投射して、無限遠に到達しうる最小の初速」
と「質点の初期エネルギーが負でない最小の初速」
が等しいものと扱っていますが、これは(仮定(*)は認めるものとして)
正しいのでしょうか?

地球の中心と質点との距離:r→∞としたとき、
U→0よりE=Kが成立し、エネルギー保存則から、
E=初期エネルギーE0より、K=E0であり、

「無限遠に到達する」⇒「初期エネルギーが負でない」

は少なくとも成立しそうだ、というのが私の考えです。ただ、

・質点が距離rの点に到達する⇔距離rにおけるK≧0
の同値関係は成立するのか?
・r→∞としたとき、rの関数であるKが有限確定値をとることは、
E=K+U=(一定)から自明としていいのか?

ということがよくわかりません。よろしければ教えてください。
686ご冗談でしょう?名無しさん:02/11/15 01:23 ID:???
数学的detailばっかりで完全に物理を見失ってんじゃないの?
察するにKは運動エネルギーでしょ? K=m v^2/2<0ってどんな状態?

等々色々言いたいことがあるんだが。とりあえず無限遠でポテンシャル
が0になるようにして(つまり U(r)=-GmM/rと取る)。初速 v_0で地球表
面から打ち上げた。地球の半径をRとして全エネルギーは
 E=m v_0^2/2-GmM/R
であるとかこういうことはOKなのかな?Gが万有引力定数。Mは地球の
質量。
687ご冗談でしょう?名無しさん:02/11/15 03:14 ID:uEexiZQI
ニトログリセリンって昔は、熱しようが凍らせようが、絶対に結晶化
しなかったってほんとですか?
1)やっぱり、とんでもな、話なんでしょうか?
2)物理定数の変化が影響するとかありますか?

結晶という現象自体が良くわかっていないです。


688おばか:02/11/15 03:19 ID:scgv97gt
突然ですが連立微分のとき方がわかんないんですけど
どうやって解くんですか?
 
x'-x-y=3exp(2t) , x(0)=2
4x+y'-y=2exp(2t) , y(0)=-1

答 x=exp(t)(cos2t+(sin2t)/2)+exp(2t)
y=exp(t)(cos2t-2sin2t)-2exp(2t)
689685:02/11/15 04:20 ID:60umVoJ2
>>686
言葉足らずですみません。
原点を地球の中心にとった運動方程式から、
質点の速度v、地球の中心からの距離rとしたとき、

E=m v^2/2-GmM/r=m v_0^2/2-GmM/R(=一定)

が導出され、K=m v^2/2, U=-GmM/rとおくと、
質点はK=0⇔v=0なる点で静止し、
地球に引き戻されてしまうと考えられるので、
質点が距離rに達し得るためには、距離Rからrまで、
常にK>0が成立すること…(1)が必要と考えました。

(つづく)
690685:02/11/15 04:21 ID:60umVoJ2
E=K+U ∴K=E-U=m v_0^2/2-GmM/R+GmM/r

より、Kはrの単調減少関数になることを利用すると、

(1)⇔距離rにおいてK≧0…(2)

と考えられるので、質点が実際に距離rに達し得るか否かの考察に、
(2)が利用できるのではないかと考えました。
そこで、質点が無限遠に到達し得るか否かは、

r→∞のとき、K→m v_0^2/2-GmM/Rが負でない条件

を考えることで処理できるのかな、と思ったのですが、
質点の運動やvの値はエネルギー保存則のほかにも、
運動方程式から得られるその他の束縛を受けているはずで、
高校レベルの私の数学力では、運動方程式を解くことができず、
エネルギーの関係を導出することしかできなかったので、
この方法が正しいのかどうか、確信が持てなかったということです。
691一回性:02/11/15 04:21 ID:lVKFDIEE
すみませんが、どなたかこの式を示してください。
→  →
grad(φψ)=gradφ・A+φdivA
692一回性:02/11/15 04:22 ID:lVKFDIEE
ずれていました。Aがベクトルです。
693ご冗談でしょう?名無しさん:02/11/15 04:30 ID:???
とりあえず左辺にしかψがなく右辺にしかAがないのはなんなんだ?
694一回性:02/11/15 04:38 ID:lVKFDIEE
すみません、訂正です。
grad(φA)=gradφ・A+φdivA
Aは全てベクトルです。
695ご冗談でしょう?名無しさん:02/11/15 04:55 ID:???
>>690
力学的エネルギー保存の法則って運動方程式から導かれたんでしょ。
696ご冗談でしょう?名無しさん:02/11/15 04:58 ID:???
>>694
一見して微分の分配則には見えるが、ベクトルの grad って何だ?
697一回性:02/11/15 05:00 ID:lVKFDIEE
なんか教授が計算練習だといって、、。
よう分からんのですよ。
698ご冗談でしょう?名無しさん:02/11/15 05:00 ID:???
もしかして示したいのは
div(φA)=gradφ・A+φdivA
ではないか?
これなら微分を分配するだけ。
699ご冗談でしょう?名無しさん:02/11/15 05:02 ID:???
ベクトルのgradientも分からないけど、
左辺は何かのgradientだから多分ベクトル量で右辺はスカラー量というのも?。
700ご冗談でしょう?名無しさん:02/11/15 05:06 ID:uEexiZQI
googleでしらべると
電磁気学B 試験問題
になるぐらいだから有名なのでは・・
701ご冗談でしょう?名無しさん:02/11/15 05:07 ID:???
>>700
???
702一回性:02/11/15 05:09 ID:lVKFDIEE
φ・A
これはφはスカラーでAはベクトルなんですけど
これは内積ではないですよね。


703ご冗談でしょう?名無しさん:02/11/15 05:10 ID:???
>>702
ただの掛け算
704ご冗談でしょう?名無しさん:02/11/15 05:12 ID:uEexiZQI
700です。
すいません。下のページ見たんですが、式がちがってますね。
http://members10.tsukaeru.net/kubocch/PastExam/EM/H11/EM_H11.htm
705ご冗談でしょう?名無しさん:02/11/15 05:14 ID:???
もし右辺が gradφ・A + φdivA なら、左辺は div(φA) となる。
もし左辺がベクトルグラジエント grad(φA) なら 右辺はテンソル量になるはず。
706ご冗談でしょう?名無しさん:02/11/15 05:15 ID:???
あれだな、練習問題が必要だ
grad A
div A
rot A
と書いたときそれぞれのAに入るのはベクトル?スカラー?
またそのとき grad A 、div A、 rot Aはそれぞれベクトル?スカラー?
707ご冗談でしょう?名無しさん:02/11/15 05:15 ID:???
(gradφ)・Aとしたほうがいいか

gradの右にくるのはスカラーじゃないとだめ
だから>>694の左辺がおかしい
708ご冗談でしょう?名無しさん:02/11/15 05:16 ID:???
704の問題なら、
a,bは成分にばらして微分を分配すれば良い。
cはバラすだけ。
709ご冗談でしょう?名無しさん:02/11/15 05:19 ID:???
>>707
まあ、世の中にはベクトルグラジエントやらベクトルラプラシアン
もマイナーだがあるからなぁ...
710707:02/11/15 05:21 ID:???
>>709
正直ワシはそれ知らん
それより一回生にそんなこと言ったら混乱しちゃうよ
711一回性:02/11/15 05:22 ID:lVKFDIEE
>706
grad A=ベクトル A=スカラー  
div A=スカラー A=ベクトル
rot A=ベクトル A=ベクトル
712ご冗談でしょう?名無しさん:02/11/15 05:24 ID:???
>>711
それを踏まえると>>694おかしいでしょ?
713一回性:02/11/15 05:24 ID:lVKFDIEE
これでいいですかね、、。
714ご冗談でしょう?名無しさん:02/11/15 05:25 ID:???
>>710
まあ、そうだな。

でも電磁気やってるといずれ出てくる。
(∂/∂t) A^μ − △ A^μ = 0 の△とか。

とりあえず、一回生はこのレスを無視してくれ。
715ご冗談でしょう?名無しさん:02/11/15 05:25 ID:???
>>713
φA はスカラーかベクトルか考えてみ
716一回性:02/11/15 05:26 ID:lVKFDIEE
そうですね。左辺がベクトル量で右辺がスカラー量に
なっていますね。
717714:02/11/15 05:27 ID:???
時間微分は2階だった。
718一回性:02/11/15 05:28 ID:lVKFDIEE
φはスカラーでAはベクトルですから
φAはベクトル量と思います。
719ご冗談でしょう?名無しさん:02/11/15 05:30 ID:???
>>718
つまり左辺がおかしいんだよ
たぶんgradじゃなくdivだと思う
720一回性:02/11/15 05:38 ID:lVKFDIEE
ということは
grad AでAにベクトル量が入ることは
ありえないのですね。
721☆☆☆☆☆:02/11/15 05:39 ID:S3J0WVsf
722ご冗談でしょう?名無しさん:02/11/15 05:45 ID:???
一応ベクトルのgradも存在するらしいが
http://www.google.co.jp/search?hl=ja&ie=Shift_JIS&c2coff=1&q=vectorgradient&lr=
このくらいマイナーだ。とりあえず僕はM1だけど一度も見たことない。

ベクトルのラプラシアンはよく使う。 A=(Ax , Ay ,Az)
にたいして△A=(△Ax , △Ay , △Az)
723ご冗談でしょう?名無しさん:02/11/15 06:00 ID:???
>>722
ただし、変換性がスカラーのラプラシアンとは違うので、注意が必要。
デカルト座標では722と書けるなんで...
電磁気なら最初から4元ベクトル形式つかえば、
なんも問題ないんだけどね。

昔、悩んだ事があった...
724ご冗談でしょう?名無しさん:02/11/15 06:17 ID:CUY9sPOn
ケータイって、1.5GHzなのに、なんでラジオから雑音が出るの?
ラジオって、455kHzぐらいしか増幅能力内じゃん?
725ご冗談でしょう?名無しさん:02/11/15 06:30 ID:???
>>724
電気機器だったら回路からノイズが出る
携帯に限ったことじゃない
726ご冗談でしょう?名無しさん:02/11/15 06:34 ID:CUY9sPOn
は?いみわかんね一よ。もっと分かりやす<説明レろボけ
727ご冗談でしょう?名無しさん:02/11/15 09:40 ID:???
>>722
その書き方だと、vectordivergenceにいたっては
ヒット件数は1件なんだが、どーよ。
728ご冗談でしょう?名無しさん:02/11/15 10:58 ID:???
>>688
x'-x-y=3exp(2t)をy=..の形にしてy'=。。を求めて第2式に入れる。
後はxについての2回微分方程式を解くだけ。yの初期条件がx'に関する初期条件を
与えるので、それで一般解を求める。これ以上の質問は受け付けんので後は
自分でやるべし。

>>685
数学科の連中が物理の本を読んで分からんというのは,こういう事かいなと分からせて
くれるような。。。
率直に言おう。
君は物理に向いていない! 数学を目指したまへ!
729ご冗談でしょう?名無しさん:02/11/15 12:51 ID:???
>>687
純度だろ
730>>728:02/11/15 13:20 ID:???
偉そうに。
何様?
731_-):02/11/15 15:59 ID:???
私の物理担当教師に聞きに行くのが怖いので、ここで質問させてもらいます。
電磁気の運動量を表す式で
eV=1/2LI^2=1/2mv^2
ってのは成り立つのでしょうか?
参考書見てもチンプンカンプンですた。どなた教えてくだされませ。
732ご冗談でしょう?名無しさん:02/11/15 16:30 ID:???
>>731
質問見てもチンプンカンプンですた。何を聞きたいのかさえわからん。もう少し状況を説明してくれないと。
そもそもそいつらエネルギーに関するもので運動量じゃないし。
733ご冗談でしょう?名無しさん:02/11/15 17:33 ID:jWaM2DYO
光の干渉で暗くなる場所に薄い板を置くとしてその板はエネルギーを受けませんよね
そうすると光のエネルギーはどうなりますか?板を突き抜けていったりしますか?
734ご冗談でしょう?名無しさん:02/11/15 17:48 ID:hGnTKmbd
今日、実験室のAFMのカンチレバー折っちゃったんだけどあれいくらぐらいするもんなのかねぇ。
しかも機械の誤作動くさいのにかなり起こられた。
735トヲリスガリ:02/11/15 18:20 ID:vDO7zYam
>734
よく知らんけど、あれって探針と一体になってるから、ちょくちょく
取り替えて使うもんじゃな〜い?
→という事はそんなに高価でも無いと思うが。

むしろ、そんなもん折れるような使い方した事で怒られたんんだと思う。
736ご冗談でしょう?名無しさん:02/11/15 18:24 ID:???
>>734
>しかも機械の誤作動くさいのにかなり起こられた。
誤作動させたのはあなたなんでしょ。機械は言われたとおりに動作しただけ

737ご冗談でしょう?名無しさん:02/11/15 18:25 ID:CmffD7Pb
>> 734
廉価版で一本3000円くらい、
高アスペクト比のやつになると一本数万てとこかな。
AFMは最初は何本か折っても仕方ないやくらいの
つもりでいないとAFMマスターにはなれんよ。
(どのみち何十枚か画像とったら針なんてすぐ劣化するし。)
738ご冗談でしょう?名無しさん:02/11/15 19:49 ID:???
>>734
原因を分析しないと、同じ失敗を繰り返しちゃうよ。
739ご冗談でしょう?名無しさん:02/11/15 19:52 ID:???
738がいいこと言った
740ご冗談でしょう?名無しさん:02/11/16 00:17 ID:aQXuUoRa
光は電子にぶつかるかどうか、教えてください。
741ご冗談でしょう?名無しさん:02/11/16 00:22 ID:???
ぶつかる
742ご冗談でしょう?名無しさん:02/11/16 00:24 ID:???
>>740
Googleで「光電効果」くぐる

というか、なんでそんな質問が出てきたの。
質問丸投げするだけじゃなくて、
この疑問が出てきた背景とかをちょっと教えてよ。
743ご冗談でしょう?名無しさん:02/11/16 00:30 ID:???
電子加速器で加速した電子をレーザー光の中に飛ばすと、
電子の軌道は変化する。って事ですよね。
744ご冗談でしょう?名無しさん:02/11/16 00:46 ID:???
Compton産卵じゃねーか?
745740:02/11/16 00:53 ID:aQXuUoRa
皆さんありがとう。
丸投げでごめんなさい。

「弱いエネルギーの光は電子に当たりにくくなる」
という、文章を読んだもので。確実に当たるのかどうかなぁっと。
746ご冗談でしょう?名無しさん:02/11/16 01:35 ID:???
光電効果って、光と電子が当たっていると言えるのかなぁ?
金属に光りが当たっているのは確かだが。
747sage:02/11/16 01:58 ID:???
>>746
アフォか
748ご冗談でしょう?名無しさん:02/11/16 01:59 ID:???
>>746
ネタか?
749真面目に読めお前ら:02/11/16 07:07 ID:???
〆切と肉体のせめぎ合いの果て 滅びゆく人間性─────
しかし 幾千幾万 夥しい犠牲の向こうに─────
http://wwwwwwwwwwwwwwwwwwwwwwwwwwwwwwwwwwwwwwwwwwwwwwwwwwwwwwwwwwwwwwwwwwwwwwwwwwwwwwwwwwwwwwwwwwwwwwwwwwwwwwwwwwwwwwwwwwwwwwwwwwwwwwwwwwwwwwwwwwwwwwwwwwwwwwwwwwwwwwwwwwwwwwwwwwwwwwwwwwwwwwwwwwwwwwwwwwwwwwwwwwwwwwwwwwwwwwwwwwwwwwwwwwwwwwwwwwwwwwwwwwwwww
例外が存在する!!!
http://wwwwwwwwwwwwwwwwwwwwwwwwwwwwwwwwwwwwwwwwwwwwwwwwwwwwwwwwwwwwwwwwwwwwwwwwwwwwwwwwwwwwwwwwwwwwwwwwwwwwwwwwwwwwwwwwwwwwwwwwwwwwwwwwwwwwwwwwwwwwwwwwwwwwwwwwwwwwwwwwwwwwwwwwwwwwwwwwwwwwwwwwwwwwwwwwwwwwwwwwwwwwwwwwwwwwwwwwwwwwwwwwwwwwwwwwwwwwwwwwwwwwww
日に30時間の執筆という矛盾!! この矛盾をひたすら執筆の密度を高めることで補い続ける─────
オーバーワークの弊害が常識化した近代思想には 暴挙とも言えるこの信仰
信仰が──暴挙を生み─────
暴挙のみが─────奇跡を生む!!!
http://wwwwwwwwwwwwwwwwwwwwwwwwwwwwwwwwwwwwwwwwwwwwwwwwwwwwwwwwwwwwwwwwwwwwwwwwwwwwwwwwwwwwwwwwwwwwwwwwwwwwwwwwwwwwwwwwwwwwwwwwwwwwwwwwwwwwwwwwwwwwwwwwwwwwwwwwwwwwwwwwwwwwwwwwwwwwwwwwwwwwwwwwwwwwwwwwwwwwwwwwwwwwwwwwwwwwwwwwwwwwwwwwwwwwwwwwwwwwwwwwwwwwww
丹念に造り上げられた巨大な論理は 空論と断ぜられ淘汰され───切り捨てられ───
無知と見紛うほどに細く引き絞られた 高密度な論理はまさに─────
http://wwwwwwwwwwwwwwwwwwwwwwwwwwwwwwwwwwwwwwwwwwwwwwwwwwwwwwwwwwwwwwwwwwwwwwwwwwwwwwwwwwwwwwwwwwwwwwwwwwwwwwwwwwwwwwwwwwwwwwwwwwwwwwwwwwwwwwwwwwwwwwwwwwwwwwwwwwwwwwwwwwwwwwwwwwwwwwwwwwwwwwwwwwwwwwwwwwwwwwwwwwwwwwwwwwwwwwwwwwwwwwwwwwwwwwwwwwwwwwwwwwwwww
ダイヤモンド!!!      


(省略されました・・全てを読むにはここを押してください)


750大鳥居☆つばめ ◆dpLvO.T.cY :02/11/16 08:15 ID:???
創価板からの荒らしか?
751ご冗談でしょう?名無しさん:02/11/16 16:04 ID:kSTV1fYh
926 :   :02/11/16 13:16 ID:S/pY7LMo
>909  檀君の墓って、これの事ですね。
「平壌の東三〇キロ程の江東郡(北朝鮮)には、檀君の陵と称するものがある。
李氏朝鮮王朝時代から檀君陵といわれていたところであるが、一九九三年に発掘され、
高句麗的墓制に基づく墓であることがわかり、なかから出土した人骨が、「電子スピン共鳴法」によって測定した結果、
五〇一一年±二六七年前のものであるとされた。
そのためそれこそ壇君の遺骨で、檀君の実在が確かめられたとし、実際の檀君は檀紀よりもさかのぼる、
というような理解がされるようになった。
北朝鮮では、一辺五〇メートルの正方形の基壇に、高さ二二メートルまで石を九段積み上げた、高句麗の将軍塚のような檀君陵をあらたにつくり、
古朝鮮のはじまりもそこまでさかのぼらせて考えるようになった」(新版世界各国史2・朝鮮史」山川出版社)

------
「電子スピン共鳴法」でそんなことまで分かるのでしょうか?
というかここで言っている「電子スピン共鳴法」って何?
752baka:02/11/16 16:25 ID:zlWwA6Rk
すいません。昨日一日中考えても分から無かったので教えてください、、、。
複素関数の事です。
「e^eをべき関数として展開すると、exp(eloge)となり、
これはexp(e(ine+2niパイ)となります。
これはさらに式を変形して、exp(e)*exp(2neiパイ)
となり、n=0のときは成り立つのですが、それ以外のnについては
様々な複素数が出てきます。僕が聞きたいのは、二つの複素数が等しい
時、実部と虚部が等しい時だと思うのに、例えばn=1の時は
e^e=(e^e)*(exp(2eiパイ)=(e^e)cos(2eパイ)
+(e^e)sin(2eパイ)となって、もしこれが正しいのなら、
右辺の実部はe^e,虚部は0で無ければなりませんよね?という事は
cos(2eパイ)が1!!?とすると2eパイ=2Kパイとなって、
e=K!!!???   ????」
といった具合です、、、、。僕は一体何を勘違いしているのか教えてください、、。
このままではべき関数の定義が怪しく見えてたまりません。
どうかお願いします!!!!!!!!!御教授ください!!!!!!!
753ku:02/11/16 16:38 ID:yBJIV4c5
BH近くのカラーをでみつけましたが
http://www.isas.ac.jp/dtc/xrey/PRESS4.html
Q  銀河のBHまで何万光年ですか?
この光は何年前の光ですか?
754ご冗談でしょう?名無しさん:02/11/16 17:34 ID:+WVI9N3H
ブラックホールの蒸発速度の公式ってありますか?
755ご冗談でしょう?名無しさん:02/11/16 17:53 ID:???
     〃      ヽ     .|
     i   ノノノ))) ))    | ブラックホールが好きなんですか?
.    i i ( 'l_] l]〈リノ    .<
     ヽヽ >  />      \_____
     Y/ヽ!ェl〉        /ヽ/ヽ
      !! l卯 l         / // /
      i ll8| !       / // /
.       l l「| l       , ´-‐ ‐- ヽ
        /!_」l|_j、    {   l ,,  } ぅん
.      / /゙´l"ヽ >   ゝ. ‐  ノ
     'ーr‐ r r ´       l !
.       l  l. |           | |
.       |  ! |           l. |

       -―-
     〃      ヽ
     i   ノノノ))) ))   / ̄ ̄ ̄ ̄ ̄
.    i i ( 'l_] l]〈リノ < ネオグランゾンとかも好きなんですか?
     ヽヽ > n/>    \_____
     Y/ヽ!ェl〉        /ヽ/ヽ
      !! l卯 l         / // /
      i ll8| !       / // /
.       l l「| l       , ´-‐ ‐- ヽ
        /!_」l|_j、    {   l   U } ……
.      / /゙´l"ヽ >   ゝ. っ  ノ
     'ーr‐ r r ´     ⊂ニl !
.       l  l. |        | |
.       |  ! |        l. |
756ご冗談でしょう?名無しさん:02/11/16 19:26 ID:???
複素数のlogと実数のlogがごっちゃになってるんだよ。
複素数のlogを z=r exp(iφ)=r exp[i(φ+2nπ)] として (r>0) 
LOG z=log r+i(φ+2nπ)で定義しよう。(あえて実数との違いを強調するため
わざとLOGと書いた。独自記号なのでほかで使わないように)。右辺のlogは
正の実数の場合のlog。
φの選び方には任意性があるから例えば -π<φ≦πという風に
取ったとしよう(ここがポイント)。このとき LOG e=1+2nπi。
このとき LOG eとlog eが一致するのはn=0のときだけ。すべてのnで
LOG eとlog eが一致すると考えるのが混乱の元なの。
例えばひねくれた人がいてφを 4π<φ≦6πとか取ったとしよう。
このとき LOG e=1+(6π+2nπ)i。複素数のLOGは無限多価関数だから
φの定義によらず一致するけど今度はlog eとLOG eが一致するのは
n=-3のときだけなわけ。

 
757追記:02/11/16 19:41 ID:???
だから exp(e)=exp[elog(e)]と書くのは位相φをある範囲、と決めて
しまったらある特定のnに対してしか成り立たないわけ、複素数の場合。
758ご冗談でしょう?名無しさん:02/11/16 21:06 ID:02zDWYTf
ものすごくレベルの低い質問で申し訳ないのですが、教えてください。
連続体力学で、断面2次モーメントの断面が半径aの円の場合の積分について
どう積分したら良いでしょうか?
つまり、2∫[-a,a]y^2√(a^2-y^2)dy です。
759ご冗談でしょう?名無しさん:02/11/16 21:10 ID:BjHBDrAz
大人の玩具激安販売!
ピンクローター190円から!!
他の商品も激安販売中!
http://www.king-one.com/
760758:02/11/16 21:51 ID:???
数学板のほうが良さそうなので、逝って来ます…
761大鳥居☆つばめ ◆dpLvO.T.cY :02/11/16 22:21 ID:8uNSpnTT
>>754
ブラックホールから質量ってとりだせないの?
http://science.2ch.net/test/read.cgi/sci/1033110142/
を単位系に注意して見れば、式はできる。
762a:02/11/16 22:30 ID:mxDUbpsF
すみません。友人から出されたこのクイズがわかりません。

にらまつみこ
こくらひこら
?ここひらた
 りみみいこ
 ここははら

ヒントは「パソコンがあると楽かな」だそうですが全然わかりません。お願いします。
763ご冗談でしょう?名無しさん:02/11/16 22:31 ID:Ny5T845A
次のような問題が出たのですが、わからないので教えてください。
「回路を構成する際に、ANDやNOT,ORゲートのみでなく、NANDやNOR
ゲートが用いられる。その理由を記せ。また、NANDやNORゲートを
用いた場合の欠点を記せ。」
764ご冗談でしょう?名無しさん:02/11/16 22:33 ID:???
>>763
なんでここ?
765ご冗談でしょう?名無しさん:02/11/16 22:34 ID:Ny5T845A
>>764
電子回路の問題なので・・・
766ご冗談でしょう?名無しさん:02/11/16 22:37 ID:???
>>765
じゃあ、機械工学にどうぞ。
767ご冗談でしょう?名無しさん:02/11/17 02:40 ID:???
>>763
丸投げ厳禁。
自分でどういう風に調べたらどこまで分かって、
どこでつまずいているのか等々、できるだけ細かく書くように。

それから、電子回路は「理系全般」がいいと思う。
768ご冗談でしょう?名無しさん:02/11/17 02:56 ID:???
>>763
MOSトランジスタでゲートを作ると、NAND、NORの方がトランジスタの数が少ない。
AND,ORはNAND、NORの後ろにNOTを接続して実現している。

とかなのかな?
769ときお:02/11/17 21:01 ID:eARwyW5I
磁石を上下に二個置いて、磁界を作ってその中に一巻きコイルを入れて
モーターにする実験あるじゃないですか。何で動くか原理はわかるんですけど
「モーターの回転原理を磁石による磁束とコイルによる磁束との関係を
用いて説明せよ。」
っていう宿題が出たんです。どういうことかわかりますか?
解説お願いします。
770ご冗談でしょう?名無しさん:02/11/17 21:04 ID:???
君の2レス前に書いてある、「丸投げ厳禁」って言葉をよく見て。
771ご冗談でしょう?名無しさん:02/11/17 21:18 ID:???
まず磁束の定義を調べる
次にN極とN極が反発する事や、S極とN極が引き合う事を磁束を使って説明してみる。
というか、どういった電磁場が発生しているか克明に絵に描けば、勝手に答えになっている。
772ご冗談でしょう?名無しさん:02/11/17 22:55 ID:???
ばかかおまえ?
773ご冗談でしょう?名無しさん:02/11/17 23:35 ID:???
772 が自分の事を言ってるスレはここですか?
774ときお:769:02/11/18 00:26 ID:BeEfCJ8D
>770〜773
すいませんでした。
もう一回、一から出直してきます。
磁石だもんなぁ・・・・・
775ご冗談でしょう?名無しさん:02/11/18 11:04 ID:???
>>773が釣れてしまった・・・
簡単に釣れるバカはリリース!
776PURE-GOLD:02/11/18 11:26 ID:7xMZg1bS
■■■■■■■■■■■■■■■■■
=================
★ 総合芸術情報ポータルサイト ★
=================
◎個人・法人広告掲載募集中!!◎
◎個人・法人情報掲載募集中!!◎
--------------------------------
http://www.pure-gold.jp/koten/
--------------------------------
■■■■■■■■■■■■■■■■■
777mnbv:02/11/18 13:19 ID:Ebp2zwGC
質量m1,m2,m3の3質点が、相互間の万有引力のみで結ばれ、1辺Rの正三角形の形を保ちながら運動している系を考える。各質点の位置ベクトルをr1,r2,r3、加速度をa1,a2,a3とする。このとき、重心を原点とする座標系から見た各質点の運動方程式を求めよ。

上の問題で私は
 m1a1=[m1G{m2(r2-r1)+m3(r3-r1)}]/R^3
と考えたのですが、模範解答は
「質量相互間に働く力は万有引力ゆえ
  a1=[m1G{m2(r2-r1)+m3(r3-r1)}]/R^3 」
と書いてありました。
私の考え方のどこがまずいでしょうか?誰か教えて下さい。
778ご冗談でしょう?名無しさん:02/11/18 14:23 ID:???

模範解答、dimensionが間違ってるじゃん。
779ご冗談でしょう?名無しさん:02/11/18 14:36 ID:Nl3TSBFZ
X線のγ線吸収についての実験で
図書館で調べてきたんですがその本がないのかアルミと鉛の
吸収係数がわかりません。
どなたか数値をおしえてくれないでしょうか?
インターネットでも調べたんですがわかりませんでした
よろしくおねがいします
780トヲリスガリ:02/11/18 16:47 ID:RWpImXaO
>779
波長わかんなきゃどうしようもないぞ。

質量吸収係数 [cm^2/g] @0.1オングストローム
Al 0.15
Pb 3.41
γ線域の値の表はしっかりした本じゃないと無いかも知れない。 
781ご冗談でしょう?名無しさん:02/11/18 17:49 ID:???
みなさん相対論のパラドクッスには食傷してるでしょうが、友人と議論
してても、分からないんです。

仮想的な、超高速で走る車があったとします。
道にどんな大きながあっても速度を上げて穴をLorentz収縮させてしまえば
穴には落ちない。

でも道路上の静止系からすると、車が速度上げて短くなってしまうので
小さな穴でも落ちてしまう。

どこが間違っているのでしょうか?
もちろん、これで相対論が間違っているなどと主張するつもりは毛頭ありません。
なので、「そうそう! だから相対論は間違ってる!」て人はレスしないでください。
よろしくお願いします。

782ご冗談でしょう?名無しさん:02/11/18 18:36 ID:???
>>781
ローレンツ収縮は、短くなったように「見える」こと
実際に短くなるわけではない
783ご冗談でしょう?名無しさん:02/11/18 18:40 ID:???
そんなことに疑問をもつこと自体、間違っとる
物理のセンスないね君
784ご冗談でしょう?名無しさん:02/11/18 18:59 ID:???
>>783
受験バカはこういう事を言うようになる。
781は気にしないように。
785ご冗談でしょう?名無しさん:02/11/18 19:04 ID:???
「アポロ月面着陸はうそだった」論者の意見をことごとく論破している
サイトがあったのですが、URL等ご存知の方はいらっしゃいませんか?
786ご冗談でしょう?名無しさん:02/11/18 19:20 ID:???
>>782
あなたYahoo掲示板で叩かれまくってる人?

>>781
http://www-ccce.kek.jp/People/morita/phys-faq/4-4J.html#item23a
これと本質的に同じ問題ですね。同時刻の相対性がキーポイント
787ご冗談でしょう?名無しさん:02/11/18 19:50 ID:sPG1c7AR
電磁力で光速以上に加速は可能ですか?
たとえば、
質量は無視できるくらい小さく、ブラックホールの重力に匹敵するくらいの
巨大な正の電荷の粒子があったとします。
そこへ負の電荷の粒子が吸い寄せられていきます。
落下する負電荷の粒子は最終的に光速を越えますか?
越えないとすると、ブラックホールに落下した場合も光速以上に加速しないのでは?
それとも、電磁力と重力は根本的に性質が違うものなのですか?
788ご冗談でしょう?名無しさん:02/11/18 20:29 ID:NKuaNzju
>>786
つまり、どんなに長い車であろうとも、穴に落っこちるのですね。
いや、どんなに短い車だろうが、穴には落っこちないのかな?
789知ったか厨房:02/11/18 20:48 ID:???
>>788スピードがついているから落ちないと考えよう…
と言うのは冗談として、穴の代わりに次のような条件で考えよう。

「仮想的な、超高速で走る車があったとします。
 道には【スタート】ラインと離れたところに【ゴール】ラインが引かれています。
 【スタート】と【ゴール】がどれだけ離れていても、速度を上げてLorentz収縮させて
 しまえば【スタート】と【ゴール】を同時に通過できる。

 でも、道路上の静止系からすると、車が速度上げて短くなってしまうので
 【スタート】と【ゴール】を同時に通過することはできない。

 どこが間違っているのでしょうか!? 」

これを踏まえてもう一度>>786のリンク先をよく読んでみてください。
790知ったか工房:02/11/18 21:02 ID:???
「車の先端と後尾が同時に」のつもりで「同時」と書いたつもりだが、文章が変だ。
ハンドルも間違えたので出直してくる。
791のんちゃん ◆YJw/wwHt/g :02/11/18 21:57 ID:???
>>781
いくら大きなスピードの車の静止系から見たときでも、車の先端
が宙に浮いている瞬間がありますね。そのときに先端がぐにゃりと
曲がって落ちるのです。相対論の範囲では「剛体」は存在しえませ
ん。
792ご冗談でしょう?名無しさん:02/11/18 22:16 ID:NKuaNzju
>>786

あなたがその観測者となって車庫の屋根に座っています。竿があなたの方へ向
かってくるのが見えます。その竿は短縮して40mより少し短くなっています。
ですから、竿が車庫を通過しているとき、車庫に完全に収まる瞬間があります。
その瞬間、両方の扉を閉めます。もちろんすぐに扉を開けますが、

すぐに扉を開けない&竿は急ブレーキをかけるとどうなるの?
扉の中に収まるの?それとも収まらないの?
793ご冗談でしょう?名無しさん:02/11/18 22:27 ID:???
>>792
急ブレーキ、というのが、車庫の静止系で見て竿全体が同時に止まる、という
意味であれば、収まります。ただし竿は力学的に潰れます

同時刻は相対的なので、竿の静止系で見れば竿全体が同時ではなく、
まず竿の前方の扉が閉まり、竿の先端にブレーキの力がかかって潰れ始め、
それがだんだん後ろに伝わって行って、竿の後端までブレーキが効いて
潰れ切ったところで後ろの扉が閉まります。このときには竿は車庫の
長さより短くなっているので収まります。
794無知蒙昧:02/11/18 23:04 ID:???
諸賢の皆さんのご回答を感心して拝見しています。

すると「講釈師見てきたような嘘を言い」という
古川柳が頭に浮かんできてしまいました。
なんせ無知なもんでにわかに信じられないおもいですわ。
795ご冗談でしょう?名無しさん:02/11/18 23:32 ID:51XXkqeu
796ご冗談でしょう?名無しさん:02/11/18 23:59 ID:???
>>794
>にわかに信じられないおもいですわ。
それでいいんじゃない?
797ご冗談でしょう?名無しさん:02/11/19 00:55 ID:nvQPzlix
中学生みたいな質問ですが、教えてください。

マイクロホンで、超指向性の「ガンマイク」という細長〜いマイクの感度パターンで
「横からの音は取れずに、正面からの音だけ取れる」原理がわからないです。
位相管方式らしく 横方向からの音は、一部が正面方向から入り 管内で側面からの
音と相殺されて感度が無いとの説明を見たのですが、正面から入った音波が管内を通じて
マイクに届くときには、すでに側面から入った音はマイクに届いていて 手遅れなのでは?
と思うのです。いまいち合点がいきません。
URLでも結構ですので、何か情報がありましたら、教えてください。
宜しくお願いします。
798大鳥居☆つばめ ◆dpLvO.T.cY :02/11/19 04:09 ID:Cs5e7Cnb
東京大学の塚田捷先生にビル清掃を薦められました。
799大鳥居☆つばめ ◆dpLvO.T.cY :02/11/19 04:10 ID:Cs5e7Cnb
暴力をふるう、明らかに幾何学的に知能が私よりも低い、高卒がいました。
800大鳥居☆つばめ ◆dpLvO.T.cY :02/11/19 04:12 ID:Cs5e7Cnb
ほんと、ビル清掃には、理数的能力(芸術では無い右脳的能力ともいうが)
が私よりも低い奴しかいなかった。
801大鳥居☆つばめ ◆dpLvO.T.cY :02/11/19 04:15 ID:Cs5e7Cnb
空間的な知能が低い代わりに、決まったとおりの言語能力はあります。
802ご冗談でしょう?名無しさん:02/11/19 04:20 ID:???
おちつけつ( ゚Д゚)⊃旦 チャノメヤ
803ご冗談でしょう?名無しさん:02/11/19 04:41 ID:6njJ5A1A
(´Д`) (´Д`) (´Д`) (´Д`) (´Д`) (´Д`) (´Д`) (´Д`) (´Д`)
(´Д`) (´Д`) (´Д`) (´Д`) (´Д`) (´Д`) (´Д`) (´Д`) (´Д`)
(´Д`) (´Д`) (´Д`) (´Д`) (´Д`) (´Д`) (´Д`) (´Д`) (´Д`)
(´Д`) (´Д`) (´Д`) (´Д`) (´Д`) (´Д`) (´Д`) (´Д`) (´Д`)
(´Д`) (´Д`) (´Д`) (´Д`) (´Д`) (´Д`) (´Д`) (´Д`) (´Д`)
(´Д`) (´Д`) (´Д`) (´Д`) (´Д`) (´Д`) (´Д`) (´Д`) (´Д`)
(´Д`) (´Д`) (´Д`) (´Д`) (´Д`) (´Д`) (´Д`) (´Д`) (´Д`)
(´Д`) (´Д`) (´Д`) (´Д`) (´Д`) (´Д`) (´Д`) (´Д`) (´Д`)
(´Д`) (´Д`) (´Д`) (´Д`) (´Д`) (´Д`) (´Д`) (´Д`) (´Д`)
(´Д`) (´Д`) (´Д`) (´Д`) (´Д`) (´Д`) (´Д`) (´Д`) (´Д`)
(´Д`) (´Д`) (´Д`) (´Д`) (´Д`) (´Д`) (´Д`) (´Д`) (´Д`)
(´Д`) (´Д`) (´Д`) (´Д`) (´Д`) (´Д`) (´Д`) (´Д`) (´Д`)
(´Д`) (´Д`) (´Д`) (´Д`) (´Д`) (´Д`) (´Д`) (´Д`) (´Д`)
(´Д`) (´Д`) (´Д`) (´Д`) (´Д`) (´Д`) (´Д`) (´Д`) (´Д`)
(´Д`) (´Д`) (´Д`) (´Д`) (´Д`) (´Д`) (´Д`) (´Д`) (´Д`)
(´Д`) (´Д`) (´Д`) (´Д`) (´Д`) (´Д`) (´Д`) (´Д`) (´Д`)
(´Д`) (´Д`) (´Д`) (´Д`) (´Д`) (´Д`) (´Д`) (´Д`) (´Д`)
(´Д`) (´Д`) (´Д`) (´Д`) (´Д`) (´Д`) (´Д`) (´Д`) (´Д`)
(´Д`) (´Д`) (´Д`) (´Д`) (´Д`) (´Д`) (´Д`) (´Д`) (´Д`)
(´Д`) (´Д`) (´Д`) (´Д`) (´Д`) (´Д`) (´Д`) (´Д`) (´Д`)
(´Д`) (´Д`) (´Д`) (´Д`) (´Д`) (´Д`) (´Д`) (´Д`) (´Д`)
(´Д`) (´Д`) (´Д`) (´Д`) (´Д`) (´Д`) (´Д`) (´Д`) (´Д`)
(´Д`) (´Д`) (´Д`) (´Д`) (´Д`) (´Д`) (´Д`) (´Д`) (´Д`)
(´Д`) (´Д`) (´Д`) (´Д`) (´Д`) (´Д`) (´Д`) (´Д`) (´Д`)
(´Д`) (´Д`) (´Д`) (´Д`) (´Д`) (´Д`) (´Д`) (´Д`) (´Д`)
(´Д`) (´Д`) (´Д`) (´Д`) (´Д`) (´Д`) (´Д`) (´Д`) (´Д`)
804大鳥居☆つばめ ◆dpLvO.T.cY :02/11/19 04:56 ID:???
  ∧_∧   / ̄ ̄ ̄ ̄ ̄ ̄ ̄ ̄ ̄ ̄ ̄ ̄ ̄ ̄
 ( ・∀・) < とめんなちゅーの
 (つ旦と)   \______________
  と_)_)

(´Д`) (´Д`) (´Д`) (´Д`) (´Д`) (´Д`) (´Д`) (´Д`) (´Д`)


                   .       ∧w∧     / ̄ ̄ ̄ ̄ ̄
. ̄ ̄ ̄ ̄ ̄\    ∧w∧  .      (  ゚Д゚) ∬ < 栄養満点!
ニラが入って > ∬ ( ゚Д゚ )        ノつ  つ旦  \_____
._____/  旦⊂| ⊂|       @/  つ
              |  |        U⌒
              ⊂ノ@     彡 
              ∪
805ご冗談でしょう?名無しさん:02/11/19 08:02 ID:???
…いいかげんにしろ
806ご冗談でしょう?名無しさん:02/11/19 11:05 ID:???
久しぶりにスイッチオンした大鳥居だ。うぜぇ。
質問スレでやんなよ。せめて隔離スレでやれや。
807ご冗談でしょう?名無しさん:02/11/19 12:26 ID:???
808781です:02/11/19 12:30 ID:???
>>781で質問した者です。回答ありがとうございました。
>>786さんに紹介して頂いたリンク先の「竿と車庫」のParadox読んで
これについては納得したのですが、元の、自動車と穴の話しについてはやっぱり分かりません。

FAQの答えにある、車庫に対する静止系では長い竿が車庫に収まり得るという事は、
「車と穴」の例では、道路の静止系においてはLorentz収縮した車は、穴に落ちる事に対応します。
(簡単のために、車の先頭と後ろに、「同時」に重力がかかると、車が穴に落ちるとします。)

この事象が、車の静止系では同時ではないというのは良いとしても、
それは、車の先端と後ろに同時に重力がかかるとは限らないという主張するだけなので、
結局は、道路の静止系では収縮した車は穴に落ちるし、車の静止系では車は収縮した穴を
越えることになると思うのです。

どこか見落としてますか?
809ご冗談でしょう?名無しさん:02/11/19 12:41 ID:???
>>787
要約するとこういうことだな。
「光速の99%に加速した。さらに光速の2%加速した。光速の101%だろ?」

はっきりいって、どうやって加速したかはどうでもいい。
重力でも電磁力でも牽引光線でも超能力でも関係ない。

要点は、「0.99と0.02を足してはいけない」ということ。
相対論の世界では、速度の足し算は、温度の足し算よりナンセンス。

正しい式は「0.99+0.02」ではなく、「(0.99+0.02)/(1+0.99×0.02)」。
光速の99.04%にしかならない。
810ご冗談でしょう?名無しさん:02/11/19 12:59 ID:???
超高速で走る車を

>道路の静止系では収縮した車は穴に落ちるし

のように、穴に押し込む力があれば、

>車の静止系では車は収縮した穴を
>越えることになると思うのです。

とはならないと思いませんか?

車が穴に落ちたのであれば、道路の系にとっても車の系にとっても
誰が見ても、穴に落ちています。
そのときの落ち方を車から見ると>>791のようになります。
811ご冗談でしょう?名無しさん:02/11/19 14:41 ID:???
>>808
>(簡単のために、車の先頭と後ろに、「同時」に重力がかかると、車が穴に落ちるとします。)

この仮定がありえないな。

「同時」が観測者により異なる以上、この仮定は
「落ちるかどうかが観測者により異なる」ことを認めている。

ゆえに:

>道路の静止系では収縮した車は穴に落ちるし、
>車の静止系では車は収縮した穴を越える

という結論になるのは当然。
812ご冗談でしょう?名無しさん:02/11/19 17:03 ID:???
>>811
いかなる車と穴のサイズと速度の組み合わせをもってしても
「車の先頭と後ろに、『同時』に重力がかかること」は有り得ないてこと?
そんなアフォな!
813ご冗談でしょう?名無しさん:02/11/19 17:18 ID:???
どこにそんなこと書いてあるんだよ
814811:02/11/19 18:36 ID:???
>>812
「誰から見ても『同時』」は絶対無理であることに注意。
815811:02/11/19 18:42 ID:???
補足しておくと、811の主題は

808は言わずもがなの自明な仮定を立てたつもりかもしれないが、
それは相対性理論に激しく違反している

ということ。
816ご冗談でしょう?名無しさん:02/11/19 18:50 ID:???
811は、
>簡単のために、車の先頭と後ろに、「同時」に重力がかかると、車が穴に落ちるとします。
と言ってるだけでしょ?
これは、あくまで、「ある座標系で」車が穴に落ちる条件を簡単化しただけ。

>「誰から見ても『同時』」
なんて仮定はしてないよ。
817816:02/11/19 18:52 ID:???
816 冒頭の「811は、」は「808は」の間違い。
811さん失礼しました。
818ご冗談でしょう?名無しさん:02/11/19 18:53 ID:???
定期アゲ
819ご冗談でしょう?名無しさん:02/11/19 20:08 ID:20KVDeiY
dx^2/d^2t=(P/M(dx/dt))-g


この微分方程式を解いてください
820787:02/11/19 20:08 ID:PkI1lebi
>>809
ということは、ブラックホールに落下するときの速度も光速を越えないということですね?
821ご冗談でしょう?名無しさん:02/11/19 20:10 ID:???
>>819 お前、必死だな。宿題かい?
822ご冗談でしょう?名無しさん:02/11/19 20:12 ID:???
>>819
まるちうざい
823HAL ◆2eOaL2U.Gg :02/11/19 20:16 ID:???
>>819
高校生スレに解き方のカキコあったよ。

つーかなんの法定式だい?一様磁場中の
ストークス抵抗を受ける球の落下って感じ?
Pってなんすか。
824ご冗談でしょう?名無しさん:02/11/19 20:19 ID:???
普通は空気抵抗を受ける物体の落下を思い浮かべるだろ
825HAL ◆2eOaL2U.Gg :02/11/19 20:20 ID:???
磁場じゃなかった重力場だ。
826ご冗談でしょう?名無しさん:02/11/19 20:28 ID:20KVDeiY
すいませんマルチするつもりではなかったのですが
質問スレが二つあるとはしらず、さっき書き込めてなかったのかと想い書いてしまいました
申し訳ないです。

Pは仕事量です、一定の仕事量のアクチュエーターでジャンプするときの
状態を自分なりに調べてみようとおもって式を立ててみたのですが。
それすらあってるかどうか微妙ですが、、、
827HAL ◆2eOaL2U.Gg :02/11/19 20:37 ID:???
>>826
アクチュエーターって何ですか?
工学の問題かな。興味深い。

Pを定数と仮定しますと、
回答にあったとおり、dx/dt=vとして、


dv/dt=(P/M)v-g
dv/(v-Mg/P)=dt・(P/M)
ln(v-Mg/P) = (P/M) t + C1
v = Mg/P + C1 exp((P/M) t)

x = (Mg/P) t + C1 (M/P) exp((P/M) t) + C2

t=0で v=v0とすると、C1=v0-Mg/P
t=0で x=x0とすると、C2=x0- (v0-Mg/P)(M/P)

よって、

∴ x = (Mg/P) t + (v0-Mg/P) (M/P) exp((P/M) t) + x0- (v0-Mg/P)(M/P)

さーて、説明したんだから目的とか教えてくださいね。

Pが正だったら指数関数的に速度が増加する解か・・。

828HAL ◆2eOaL2U.Gg :02/11/19 20:42 ID:???
>>827に少し補足。

ちょいと形を整えれば、

x = x0 + (Mg/P) t + (v0-Mg/P) (M/P) ( exp((P/M) t) -1)

か。
829ご冗談でしょう?名無しさん :02/11/19 20:57 ID:loH+i3+E
いきなりで申し訳ないんですが

x=exp(-x)

ってどのような手段で解くのでしょうか?
830HAL ◆2eOaL2U.Gg :02/11/19 20:59 ID:???
>>829
数値的に解くしかないんじゃないのかな

x ≒ 0.5671432904

831ご冗談でしょう?名無しさん:02/11/19 21:00 ID:20KVDeiY
アクチュエーターはモーターとかです、言ってみれば棒を
一定の仕事率で上っていく所を調べたかったのです。
そして
ある時点での速度から棒の質量をm1としてモーターと本体の質量をm2として
h=(1/2)*m2*v^2/(m1+m2)g
としてhとPや棒の長さの関係をしらべようと思った次第です。
しかし
dv/dt=(P/Mv)-g
なんですよ、、、

E=Mgx+(1/2)Mv^2
なので

あと目的ですが、興味本位です、何かに使おうとか具体的な考えはないです。
832ご冗談でしょう?名無しさん:02/11/19 21:05 ID:???
自由落下してる棒を上るの?ワケワカラン
833ご冗談でしょう?名無しさん:02/11/19 21:20 ID:20KVDeiY
   ┏┓
   ┃┃
   ┃┃
   ┃┃
 ┏┻┻┓
 ┗┳┳┛←モーターの入った機械(m1)これが棒を上る
   ┃┃
   ┃┃
   ┃┃←置いてあるだけでくっついていない棒(m2)
 ━┻┻━━
  地面
834あのう:02/11/19 21:22 ID:j4p+444a
整流回路のコンデンサの用量を決める計算なんですが、
「電流電圧に関わらず出力回路の抵抗値で決まり2105μF/Ω」であってますか?
1Aあたり2000μFと書いてあるページを複数見つけてしまい不安です。
以下私の計算です。
トランスの2次側が最大Pボルト出力 Rオーム
50Hzでθ=100πt 秒
平均電圧を上回ってから平均電圧に戻る時まで
コンデンサが電気量を蓄え続ければいいので
電圧を抵抗値で割って電流になると仮定して、時間積分すると
最大静電気量=∫(Psinθ-P*2/π)/Rdt 範囲はsinθ=2/π同士の間
変形して最大静電気量=-[cosθ+2θ/π]P/(100πR)
[]内はθ1からθ2まで
θ2=2.45148556221525328645826547420865
θ1=0.690107091374539952004377909070851

cosθ2=-0.771177842932905401964447040055543
cosθ1=0.771177842932905401964447040055543
2*θ2/π=1.56066418057988671763667760489929
2*θ1/π=0.43933581942011328236332239510029

静電気量=0.421027324706037368655538870311*P/(100πR) クーロン
コンデンサの平衡時電圧=2*P/π

電気容量=0.421027324706037368655538870311/(200R) ファラド
835829:02/11/19 21:23 ID:???
>>HALさん
即答ありがとうございます。

数値的に解くしかないんですね。
行き詰まっていたので非常に助かりました。ありがとうございます。
836ご冗談でしょう?名無しさん:02/11/19 21:52 ID:???
丸答え厨がいるな。>>819にとってせめてもの救いは、丸答えの問題が違ってるってことだ。

vが分母に来てても積分するところまでは同じようにできる。最後は
陰関数の形になってしまってv=v(t)という形にはならないみたいだけどね
837ご冗談でしょう?名無しさん:02/11/19 22:27 ID:???
確かにdx/dtは分母でないと次元合わないな。
838ご冗談でしょう?名無しさん:02/11/19 22:28 ID:???
>>836

仕切りたがりは嫌われるよ
839ご冗談でしょう?名無しさん:02/11/20 00:12 ID:YOZT1E6A
>>836
いやいや、もっともっと仕切ってくれ
840ご冗談でしょう?名無しさん:02/11/20 00:29 ID:g69/yX0E
以前ハッブル宇宙望遠鏡が地球から最も遠い天体を
探しているといったTV番組を見ていて、
その天体は光の速さに近い速度で遠ざかっている、
と言った内容の事を言っていました。

もし仮に、地球から見てその天体と正反対の方角に
やはり、光速に近い速度で遠ざかっているとしたら、
この二つの天体が遠ざかる速度は光速を越えている
事になるのではないでしょうか?

あまりに稚拙な質問でしたらすんまそん。
841ご冗談でしょう?名無しさん:02/11/20 00:34 ID:???
何をどこから説明すればいいのか分からないけど、
速度が普通に足し算で足せるのは光の速さよりも十分小さいときだけ。
光の速さに近づくときは相対論的な別の式が必要でその式によると
合成された速度が光速を超えることはない。
842ご冗談でしょう?名無しさん:02/11/20 00:50 ID:???
>>840
地球から見たら2つの天体は両方とも光速で遠ざかる事には問題がない。
もっとも光速はありえないので、便宜的に(光速-Δ)の速さとしておこう。

で、片方の天体に行って見たら、
地球が(光速-Δ)で遠ざかっているように見え、
反対側の天体は(光速-Δ)+α(だが光速以下)で
遠ざかっているように見える。

ちょっとわかりづらいか?
843840:02/11/20 00:55 ID:g69/yX0E
>>841
レス有り難うございます。
なんとなく"地上の速度常識が通用しない"と
言う事だけは把握出来ました。
844840:02/11/20 01:05 ID:g69/yX0E
842さんもサンクスです。

あまり深く考えると脳みそがウニになりそうなんで、
今日はもう寝る事にします。
845840:02/11/20 03:29 ID:q4q1PGcy
再び疑問が沸いて目を覚ましてしまいました(W。

いわゆるウラシマ効果についてですが、
地球を出発した宇宙船が準光速で旅行して帰還した場合、
年を取っているのは地球に居た人の方だと聞きました。

しかし、両者の関係を相対的に考えてみると、
宇宙船を基準に見たら準光速で旅して来たのは
地球の方だと言う事になり、早く年を取るのは
宇宙船に乗って居た人の方、と言う事には
ならないのでしょうか?

稚拙な質問ばかりで恐縮ですが、
よろしくおながいします。
846ご冗談でしょう?名無しさん:02/11/20 03:34 ID:???
>>845
実は相対的ではない。「双子のパラドックス」でぐぐる。
847ご冗談でしょう?名無しさん:02/11/20 03:55 ID:???
電場について質問なんですが、
点電荷+aと点電荷-b間に発生する電場で、a≠bの場合
E=1/4πr^2・Q/ε
の電荷Qはどーなるんでしょうか?
848ご冗談でしょう?名無しさん:02/11/20 04:02 ID:???
>>847
題意がよくわからんが、二電荷の合成電場を出したいの?
だとしたらa=bの場合だって、E=1/4πr^2・Q/εという形では表せられないのだが。

合成電場はそれぞれの電荷が作る電場を重ね合わせればいいだけ
849847:02/11/20 04:18 ID:???
>>848
電場は
+――――――――→−
こうできるから、a=bの場合
E=1/4πr^2・a/ε
じゃないんですか?
850850:02/11/20 04:20 ID:???
850ゲット




851847:02/11/20 05:14 ID:???
勘違いしてました。
>>847は無かったことにしてくらはい
852840:02/11/20 09:47 ID:X7Ic2TxT
>>846
ぐぐりました。サンクス。
853ご冗談でしょう?名無しさん:02/11/20 11:28 ID:???
>>852
見つけたサイト貼ってみ
854ご冗談でしょう?名無しさん:02/11/20 12:27 ID:???
>>845
宇宙船が再び地球に戻ろうと方向転換するとき加速度をうけるだろ。
そこが地球のほうと違う。
それがきいてくる。
855848:02/11/20 14:03 ID:???
>>849
そのrってのはどこからの距離だ?

どこからの距離だとしても、電場が無限大になるのはr=0の一点だけだね。
でも実際は正電荷のあるところと負電荷のあるところの2か所あるね。
だからその式で表されないのは明らか
856852:02/11/20 16:02 ID:JYiDJLIv
857ご冗談でしょう?名無しさん:02/11/20 16:08 ID:???
>>856
good
858トンデモ君:02/11/20 19:00 ID:???
散々ガイシュツなんでしょうけど質問させて下さい。

←B1   A   B2→   B1、B2:矢印方向へ移動する物体
Aから時速16万kmで遠ざかる物体B1、B2があるとします。
Aから見るとB2は一時間に16万km離れていますよね?
B1とB2の相対速度はおよそ25万km近い速度で遠ざかっている(そう見える)ことになりますが
実際の空間的距離は一時間に32万km離れていることになるんですか?

もうひとつ、物体が+激しく加速+すると質量が無限大になるとありますが、宇宙空間内において
何を基準にして速度が速いということになるのでしょうか?基準点が曖昧ですよね?
その物体が持っている運動エネルギーが+激しく増加+すると質量が無限大になるということな
のでしょうか?
859ご冗談でしょう?名無しさん:02/11/20 19:19 ID:???
http://ex.2ch.net/test/read.cgi/news/1037755966/l50
                \ │ /
                 / ̄\   / ̄ ̄ ̄ ̄ ̄ ̄ ̄ ̄ ̄
               ─( ゚ ∀ ゚ )< 284 284!
                 \_/   \_________
                / │ \
                    ∩ ∧ ∧  / ̄ ̄ ̄ ̄ ̄ ̄ ̄ ̄ ̄ ̄
 ̄ ̄ ̄ ̄ ̄ ̄ ̄ ̄\∩ ∧ ∧ \( ゚∀゚)< 284 284 284!
  284〜〜〜!   >( ゚∀゚ )/ |    / \__________
________/ |    〈 |   |
              / /\_」 / /\」
               ̄     / /
                    ̄
関連リンク
http://www.tv-tokyo.co.jp/ テレ東HP
http://audition.tv-tokyo.co.jp/list.cgi 候補者一覧

みなさん、ぜひ284に一票入れてやって下さい。
http://audition.tv-tokyo.co.jp/img/candidates/large/00284.jpg

2ちゃんねらーって何一ついいことしてないよな
ここで一発284の夢をかなえてやろうよ!!
860ご冗談でしょう?名無しさん:02/11/20 19:25 ID:???
>>858
前半:
いや、時速16万kmくらいならあまり問題にならないよ(藁
時→秒だとすれば、まあだいたいその通り。ただ「実際の空間的距離」というのは
意味ないな。一秒後のB1とB2の間が、Aの立場では32万km、B1,B2の立場では25万km。
どっちが「実際」ってこたぁない。
後半:基準点は、観測者。つか、無限大にはならないと思うのだが。
861ご冗談でしょう?名無しさん:02/11/20 19:38 ID:???
>>858
「時速」は「秒速」の間違い? 以下「秒速」と解釈して進める

誰から見て、なのかをはっきりさせること。
Aから見て、ならばB1とB2は互いに毎秒32万kmずつ離れて行く、でOK。
そんなことまで相対論は禁止していない。
B1から見て、ならばB2は毎秒25万kmずつ離れて行く。
どの系から見るかで距離は変わるのだから矛盾ではない。

もうひとつ、のほう。
速度は、観測者から見て、です。

それはそれとして、速度が増すと質量が増える、といった場合の質量とは
相対論的質量のことで、エネルギーをc^2で割っただけのものです。
これを質量の名で呼ぶことは誤解を生むばかりで、ほとんど益のない
用語であるため、現在はほとんど使われていません。あなたも相対論的
質量は使わないことをお勧めします。現在質量といえば静止質量のことを
指します。これは速度によって変化しない量です。

ちなみに相対論的質量がどのような誤解を呼ぶかの格好の例が
「アインシュタインの相対性理論は間違い」スレで現在進行中です。
862858のトンデモ君:02/11/20 19:59 ID:???
>>860-861
すみません。時速じゃなくて秒速の間違いでした
つまり、観測から一秒後
●Aから見るとB1、B2の距離はAから16万km離れている
●B1からB2を見るとおよそ25万km近く離れている
というのはいいんですが、これはそう”見える”だけなのかな?と思いまして。
またトンデモ話を付け加えると、一秒後に静止させ、B1B2間をメジャーで測ると
実際は32万km離れていることになるのかなと。
一秒後、B1←16万km→A←16万km→B2  B1←16万km→←16万km→B2
でもB1に居る人はB2を見た感じ25万kmくらいしか離れてない様に見えるよー、みたいな。
B1←16万km→A←16万km→B2
 |←── 25万km ──→|   というのが座標的に納得いかなくて・・・

「質量」についてはよくわかり、納得しました。
863ご冗談でしょう?名無しさん:02/11/20 20:00 ID:???
加速度運動には基準はいらないよ。
864ご冗談でしょう?名無しさん:02/11/20 20:08 ID:???
>>862
Aから見て1秒後に同時にB1B2を止めたとしても、B1からみれば止まるのは
同時ではない。同時刻の相対性に注意。

865ご冗談でしょう?名無しさん:02/11/20 20:11 ID:???
>>862
一秒後に静止させ…つーても、その一秒後というのもA,B1,B2にとって同時では
ないからなぁ。なんとも言えん。
座標的に納得いかない、というのも含めて、どうも絶対座標・絶対時間、みたいな
イメージが頭から追い出せてないね?
866ご冗談でしょう?名無しさん:02/11/20 20:39 ID:???
>>864-865
申し訳ない。
つまり例えば、A、B1、B2がそれぞれが1秒間計ってたとすると
B1は自分の時計で一秒計って止まったつもりなのに、Aから見るとA自身の時計で
おいコラ何秒動いてんだよとか、B2が1秒計ってB1見てもお前の時計パチモンか?
みたいな感じになるんでしょうかね。
867ご冗談でしょう?名無しさん:02/11/20 20:43 ID:???
>>866
そんな感じ。それどころか、いつから一秒後だよ?というのすら統一できない
っちゅうか。
868ご冗談でしょう?名無しさん:02/11/20 20:55 ID:???
なるほど。トンデモ質問は解決しました。thx
869834:02/11/20 21:02 ID:fNbtcf+e
>>834を教えてくれる親切な人はいませんか?
870ご冗談でしょう?名無しさん:02/11/20 21:21 ID:uqZaldpo
量子井戸中でのブロッホ関数はどのように記述できるのでしょうか?
871中卒塗装工:02/11/20 22:30 ID:???
>>869>>834
簡単の為に負荷回路を抵抗 R とみなすと、
コンデンサ C による平滑回路は、RC回路とみなせます。
そして、このRC回路には一方的にしか電流は流れないような
変動する電圧源 V(t) が並列に接続されている事になります。

ここで、RC回路の時定数τはRとCの積で決まりますね。

とすると、電源からの電流が t の期間こなかったら、負荷に加わる電圧は
最大電圧の V から V e^(-t/τ) に低下するでしょう。

電流が来ないのは、RC回路の両端電圧が電圧源の電圧 V(t) より高い期間です。
これは整流回路の性質。
872中卒塗装工:02/11/20 22:30 ID:???
供給される電流がどれだけの期間止まるかは、
交流電源の周波数だけでなく、全波整流や半波整流などの整流方式にも依存します。

最終的に平滑コンデンサの容量は、
どれだけの電圧低下までを負荷が許容するのかを考慮して選べば良いわけです。
また、その容量は負荷回路側の条件としてその負荷抵抗 R にのみ依存します。

大きな容量を選べばリプルも小さいなどメリットも大きいが、
コストと場所をとってしまう... そこらへんはトレードオフですね。
873ご冗談でしょう?名無しさん:02/11/20 22:46 ID:???
>>870
問題設定は具体的にどんなポテンシャルか。
ブロッホの定理を知っているか。どんなものか。

以上2点の補足求む。
874大鳥居☆つばめ ◆dpLvO.T.cY :02/11/21 03:18 ID:Op9mHzul
>>870
ブロッホψ(a:ポテンシャルの周期、L:井戸の幅)
ψ=sin(2πkx)f(x)
f(x+a)=f(x)
k=n/2L
875美保子:02/11/21 08:33 ID:???
 私は広島大学で3成分(NaCl−H2O−CO2)系ににおいてそれぞれ3つの成分のモル分率が変化するときの三角ダイアグラムを書くと言う研究を行っております。
しかし私はこれまで遊んでばかりでやっとここ1ヶ月熱力学についての勉強を始めたところなんです。教授にも卒業が危ないって言われちゃって。それで唐突なんですけどお願いがあります。
熱力学に詳しい方相談に乗って頂けませんか?


876美保子:02/11/21 08:35 ID:???
 613のみほこなんですけどいまの所状態方程式からNaClsolution中の各モル
分率に対するCO2のhenry's law constant は分かるんです。これからある温度における液相と気相の割合を求める式ってできるもんなんでしょうか?
 
877美保子:02/11/21 08:50 ID:???
 じつわ私875と876の書き込みするの2回目なんです。前回の書き込みしてから考えることがたくさんあって軽い鬱になっちゃって最近まで研究中断してたんです。
レス頂いた617さん619さん622さんほんとにごめんなさい。状況がかなりせっぱくしているので直にメール交換して頂ける方いませんか?[email protected]まで連絡ください。
勝手なお願いで申し訳ありません。ちなみに617さんと619さんと622さんの質問にお答えしますと圧力と温度は変数で二成分系のへいこうについては理解しています。
878k:02/11/21 09:25 ID:???
拉致家族が野中・亀井・中山・土井を名指しで・・
http://ex.2ch.net/test/read.cgi/news/1037837844/l50
879ご冗談でしょう?名無しさん:02/11/21 11:55 ID:???
>>875-878

この辺で聞いてみたらどうです?熱力学詳しい人がよってくるんじゃない


■熱力学研究のあたらしい潮流を議論するスレッド■
http://science.2ch.net/test/read.cgi/sci/1019285691/

ここらで一発、ゼミスレッド「熱力学」
http://science.2ch.net/test/read.cgi/sci/997392944/

880ご冗談でしょう?名無しさん:02/11/21 13:50 ID:DFpPui13
十次元時空って言いますけど、空間が三、時間が一、力が四つでは
足しても八にしかなりません。十次元というのはどのようなものなんでしょう?
MKS同様に考えてはいけないのでしょうか?
881ご冗談でしょう?名無しさん:02/11/21 13:56 ID:???
>>880
ごめん。全然意味わからん…かなり誤解してるよ。
たぶん気にしないほうがいいと思うよ>10次元
もっと基礎なことから勉強するほうがいいと思う。
882ご冗談でしょう?名無しさん:02/11/21 15:00 ID:???
なぜ力を次元に足すんだ?
MKSって何の関係があるんだ?
883ご冗談でしょう?名無しさん:02/11/21 15:10 ID:tUtM8biO
>>880
なんか俺も十次元なんて話をどっかで聞いたなと思ったんで思い出そうとしてみた。
恐らく何年か前のニュートンに載ってた話題だと思う。詳しいことは全然知らないけど。
884ご冗談でしょう?名無しさん:02/11/21 15:12 ID:???
  /ヽ、へ     ,ヘ、/ヽ
 | l ,ゝ〃  ̄ ` ヽー┐ヽヽ
 | lく i     ,、 ヾ/ l | ちょっと聞きたいのですが〜〜
 .| l ヾi/レ/!リノノ )))ヽ l |   
 .| l  |ld' ○ ○〈リ|  l |  1万光年先にあるお星様の光は〜〜
 | l  |゙ 、 _lフ / | l l | 1万年前のモノですよね〜〜
 | l l /``==ヽ   | l l |  
  |l l├| [まお]H   |,、l l | なら、1万光年先のお星様から〜〜
 ノ'リ ∪________∪   リ l/レソ  地球を見ると〜〜
     | `T' |          1万年前の風景が見れるのですか〜〜?
     |- | -|
     (二)二)
885ご冗談でしょう?名無しさん:02/11/21 15:38 ID:???
>>883
まあ、ひも理論の関連記事っしょ。
>>884
細かいこと考えなければ、見えると言っていいんじゃないかなー。
886ご冗談でしょう?名無しさん:02/11/21 16:25 ID:BxLYwkkD
>>884
「今」の数光年離れた場所という考え方に意味がないはず。
今ここにいる人が五年前に五光年離れたところで十年前のここを見たとは言えるけど。
今現在のこの場所から枝を伸ばすように考察していかないと。
光速の制限を無視して位置をジャンプさせちゃいけないわけだ。
887ご冗談でしょう?名無しさん:02/11/21 16:53 ID:6zJLf/hx
ある文献で自由電子の理想ガスの状態方程式が以下のようになってました

P=Z*ρ*T/mp

Zは実効的な電荷で、mp(g)は陽子質量です
しかし単位が、P(Mbar)、ρ(g/cm3)、T(eV)とばらばら

この式ってあってるんですか?
単位が上の通りなので定数が必要だと思うんですけど
888両端:02/11/21 17:28 ID:hGdgkhCI
可視光線ってどんなの?
光ってなにでできているの?
889ご冗談でしょう?名無しさん:02/11/21 20:38 ID:fVj0N+kD
剛体の運動エネルギー保存則について教えていただきたいのですが。

回転をせずに直進する場合は、剛体を質点の集まりと考えて、それぞれの質点の速度は同じ方向を向いているので質点はぶつからずにいるので、運動エネルギー保存則がなりたちそうであると理解できるのですが、
回転をしながら直進する場合は剛体を質点の集まりと考えて、それぞれの質点の速度を考えると、それぞれの質点がぶつかりあって、運動エネルギーの散逸がおこりそうな気がするのですが。
私の考えのどの辺りが誤っているのかどなたか教えてください。
890三日考えた(ρ_;):02/11/21 21:32 ID:nr93eRXa
質量mの物体がケプラー運動している。
このとき質量mの物体の角運動量は・・・

☆地球の質量をM、動径r mの速度をvとする。

どうか教えて下さい。
891ご冗談でしょう?名無しさん:02/11/21 21:54 ID:???
>>889
剛体の定義は、質点間の距離が変わらないこと。
「それぞれの質点がぶつかりあって」というところがおかしい。
892ご冗談でしょう?名無しさん:02/11/21 22:20 ID:???
ありがとうございました。
質点系の特殊な形であって、絶対に質点同士はぶつからないわけですね
893苦学生:02/11/21 22:35 ID:9y1fAD/W
電気に詳しい人お願いします。
一つの軸を中心として同心円形の磁束を生じ、その半径をrとすると、
磁束密度の大きさが
           r>aの時 B=k/r
r<aの時 B=rk/(a二乗)
である。この磁束を生じる電流を求めよ。
という問題ですが、アンペアの周回積分とdiv rotA=0を使って説く方法を
教えてもらえませんか?
解はr>aの解j=0で、r<aの解jx=0,jy=0,j=2k/(μa二乗)となるのですが。
894苦学生:02/11/21 22:41 ID:9y1fAD/W
アンペアの周回積分は∫Bds=μjですよね。
使い方がワカラナイッス!
895ご冗談でしょう?名無しさん:02/11/21 22:57 ID:???
とりあえずアンペールの法則(アンペアの周回積分)にB=rot Aを
代入してストークスの定理使って書き換えてぐらん。どうなる?
896苦学生:02/11/21 22:59 ID:9y1fAD/W
やってみます!
897ご冗談でしょう?名無しさん:02/11/22 05:17 ID:Wcm6J3MD
>>875
ちょっと高度な話なので、
http://groups.google.co.jp/groups?hl=ja&lr=&ie=UTF-8&oe=UTF-8&group=sci.physics
で聞くことをお勧めします。
898ご冗談でしょう?名無しさん:02/11/22 05:27 ID:???
899大鳥居☆つばめ ◆dpLvO.T.cY :02/11/22 05:36 ID:NZQM7uRZ
>>875
3種類の物質が化学変化と相変化を起こして平衡になっている時、
各物質のモル分率の分配の条件
化学反応についての平衡条件
相変化についての平衡条件
が必要。
900ご冗談でしょう?名無しさん:02/11/22 06:19 ID:???
901ご冗談でしょう?名無しさん:02/11/22 11:48 ID:???
>>888
可視光とは、あなたが見ている光です。
可視光でない光としては、赤外線や紫外線があります。
光は、電磁場の波動、もしくは、光子で出来ています。
「もしくは」の厳密な意味を知りたければ、量子論を学んでください。
902ご冗談でしょう?名無しさん:02/11/22 12:11 ID:???
>>901
> 「もしくは」の厳密な意味を知りたければ、量子論を学んでください。

広辞苑の方がよいと思うが。
金庫に関する質問はここでしてもいいのでしょうか?
904ご冗談でしょう?名無しさん:02/11/22 13:28 ID:30XkTr6D
SPring-8は学部生、院生だと旅費は出ないんですか?
905ご冗談でしょう?名無しさん:02/11/22 13:29 ID:J3pidWHU
>>904
ぷっ
906ご冗談でしょう?名無しさん:02/11/22 14:02 ID:???
>>905
ぺっ
907ご冗談でしょう?名無しさん:02/11/22 17:33 ID:???
>>906
ぽっ
908ご冗談でしょう?名無しさん:02/11/22 19:06 ID:Wcm6J3MD
>>901
> 可視光とは、あなたが見ている光です。

なんだこの変な説明は、、、

光の中でも人間が見ることのできる周波数範囲のものを
可視光と呼びます。つまり、見ることのできる光。
909☆受験生☆:02/11/22 21:37 ID:2twpF2NT
光子について、エネルギーを光速で割ったら運動量の形になりますよね。
このエネルギー>運動量って関係は光子だけに成り立つ関係なんでしょーか?!
普遍的に成り立つんでしょーか?
910ご冗談でしょう?名無しさん:02/11/22 22:28 ID:???
>>909
距離を時間で割ったら速度の形になりますよね。
この距離>速度って関係は...?!

単位が違うものの大きさを比べることはできません
911909:02/11/23 00:23 ID:LVpbLEQd
あ゛っ
そっか!!式の形だけに捉われてました
ありがとーございました☆
912ご冗談でしょう?名無しさん:02/11/23 01:29 ID:uQ6qjdFB
音速超える瞬間にどーんって聞こえると聞いたのですが、なぜでしょうか???
913ご冗談でしょう?名無しさん:02/11/23 01:35 ID:???
>>912
衝撃波でぐぐる
914ご冗談でしょう?名無しさん:02/11/23 01:38 ID:???
自分の出す音波の波長の間隔が
つまっていって飛行機が音速に達したとき
かたまり状態になっている絵をよく見るね。
915ご冗談でしょう?名無しさん:02/11/23 10:07 ID:???
普通の剛体の定義は、各質点間の距離が不変、だけど、
この距離がローレンツの距離だったら、どうなるんだろ。
なんかの物理的なモデルになってるのかな?
力学の教科書を久しぶりに見たが、どこを変えればいいのかよく分からん。

(r×相対論的なp)を固有時間で微分=r×相対論的なF
に従うのか?
×は、4次元空間の外積で、3次元とは相当性質が違うような気がする。
916912:02/11/23 10:12 ID:uQ6qjdFB
>913
衝撃波というのを調べると
波源の移動速度が波の移動速度を越えた時に起きるとありますが、
越えた後はどうなるのでしょうか????

あと音の強さって音の大きさってことでOKですよね。

教えて君ですいません。
917ご冗談でしょう?名無しさん:02/11/23 10:29 ID:???
>>916
波源からあなたまで波が伝わっている間じゅう
波がどんどん大きくなってしまうという
特異点が存在するのかな?
918ご冗談でしょう?名無しさん:02/11/23 12:55 ID:???
>>915
どうもならん
919ご冗談でしょう?名無しさん:02/11/23 14:38 ID:???
>>917
普通は「1秒前に波源から出た波」「2秒前に波源から出た波」
「3秒前に波源から出た波」「4秒前に波源から出た波」は干渉しないが、
波源が音速を超えると、干渉しちゃう点が出てくる。

で、全部が増幅するような干渉の仕方をする地点ができちゃう。
920HAL ◆2eOaL2U.Gg :02/11/23 16:14 ID:???
衝撃波ってのは、波動が不連続になる現象だよ。
爆発したとかにもおきます。

じゃあ、なぜ、突然、ドカーンと聞こえるかは、物理量が不連続に
変化するため、突然、音が大きくなったように聞こえるため。
典型的にはγ+1/γ-1倍(γ=5/3)にドカーンと振幅が大きくなる。
音の大きさは振幅の二乗に比例するから・・・あとは、計算
やってくださいませ。
921ご冗談でしょう?名無しさん:02/11/23 16:48 ID:???
キッテル上巻(7版)242ページの式38から39への変形がうまくいきません。
どなたかヒントください!公式使うのかなぁ・・・
922ご冗談でしょう?名無しさん:02/11/23 17:19 ID:???
>>921
キッテルの何?
923921:02/11/23 17:58 ID:???
固体物理学入門です。
924ご冗談でしょう?名無しさん:02/11/23 19:04 ID:???
>>920
なるほど。理解しました。

ジェット機の衝撃波でガラス窓が壊れるのと、
大きなスピーカーでボリュームMAXでガラス窓が壊れるのでは、
微妙に違うんだ。
925ご冗談でしょう?名無しさん:02/11/23 19:52 ID:???
鏡を14枚もってきて、日光を同じ場所に反射させたら、温度が70度に
なりました。鏡1枚の時と比べるために、もう1枚鏡を探させられました。
なんで、14のうちの1枚を使って、13枚:1枚で比べたらだめなんですか?
926ぶちゅり:02/11/23 23:42 ID:3u6X0ioo
水素スペクトルの波長を求める実験で望遠鏡の回転角を2θとすると回折角ってθですかん?
927ご冗談でしょう?名無しさん:02/11/23 23:47 ID:???
>>925
先生も鏡を持ちたかったから。察してやれ。
928ご冗談でしょう?名無しさん:02/11/24 00:32 ID:???
>>921
今、手元にキッテルないのだが、どんな式?
929ご冗談でしょう?名無しさん:02/11/24 02:47 ID:???
物理の実験ので両対数グラフを使って図を作ってみたんですが、ものすごく微妙に曲線になってしましました。
理想結果としては、一直線でそのグラフの傾きを求めて実験成功!って終わりなんですけど、無理やり直線にしたほうがいいんでしょうか?
誰かお願いします。
930ご冗談でしょう?名無しさん:02/11/24 03:12 ID:rGD3Qqe9
原子とかのことを学校で習いましたが
思い出しても、なんか頭の中でうまくまとまりません。

1、原子は原子核と電子からできているとしたら、原子の見た目の大きさは
  原子核と同じなんですよね?電子はその周りを回っているだけですし。

2、で、原子核は陽子と中性子でできていて、それらはクォーク(=素粒子)で
  できているらしいのですが、
  つまり、染色体は陽子と中性子というクォークに囲まれているのでしょうか?

3、染色体は写真で見たら、XとYの形になってましたが、
  DNA(とたんぱく質)が螺旋状になっているものが染色体だとすると、
  螺旋状のDNAがXとかYの形になっているって事ですか?

931ご冗談でしょう?名無しさん:02/11/24 03:53 ID:???
>>930
1、普通原子の大きさとは
原子核の周りを回ってる電子の広がりを考慮して考えます。
「電子が回っている」というのは実際には比喩表現で、
実際にはそういう古典的な描像では表現不可能な状態にあります。

2、何が言いたいのかよく分かりませんが、
真空中に染色体を置かない限りは
クォークに囲まれてるといえばそうなのでしょう。
だからと言って、何も特別な状態だとは思いませんが。

それから、陽子と中性子はクォークの名前ではありません。
陽子はアップクォーク2個、ダウンクォーク1個から、
中性子はアップクォーク1個、ダウンクォーク2個からできています。

3、DNAはそれ自体が螺旋状の構造をしており、
それが折りたたまれてそういう形になっています。
932ご冗談でしょう?名無しさん:02/11/24 04:14 ID:rGD3Qqe9
レスありがとうございます!
アップクォークとかダウンクォークいう単語は初めて知りました。

今本を読んでいて疑問に思ったのですが、
宇宙全体の質量の見積もりに影響するダークマターがニュートリノではないかと
考えられているそうですが、もしそれが確定したとして、
その見積もりを達成するのにニュートリノだけで十分なんですか?
十分でないなら、ダークマターはニュートリノ以外にもあるって事になりますよね。

また、ニュートリノはなんでもすり抜けるそうですが、
今現在も我々の体をすり抜けているくらいニュートリノは沢山あるのですか?
933ご冗談でしょう?名無しさん:02/11/24 05:35 ID:???
>>929
なんで直線からそれるのかを考察すればいいレポートが書ける
成績はどうでもいいなら無理やり直線にしたらいいかと
934ご冗談でしょう?名無しさん:02/11/24 05:46 ID:???
>>932
ブラックホールもダークマターだけど
ニュートリノは観測しにくいから総量は謎

太陽からもたくさんニュートリノは出てるから
地球をすり抜けてるニュートリノはたくさんある
935ご冗談でしょう?名無しさん:02/11/24 10:18 ID:EWEPzK4c
半径1cm長さ1mのアルミニウム棒を一度ねじったときの、偶力モーメント
と弾性エネルギーを計算せよ。
この問題ですけど剛性率と回転角度がわからないんですけど
どうやってもとめたらいいですか
936ご冗談でしょう?名無しさん:02/11/24 10:31 ID:mRPorHA2
一度←角度
937935:02/11/24 10:44 ID:???
>936
うっかりしてました。恥ずかしい。
教科書にアルミニウムの剛性率が2.61*10^10と
載っているのですがこれ使っていいんですかね?
この条件だけで剛性率はもとめられる物なのでしょうか?
938929:02/11/24 11:53 ID:???
>>933
了解しました。いいレポートを目指すことにしますよ。
朝一でありがとうございました。
939921:02/11/24 13:44 ID:1iAdKH1F
>>928
半導体の伝導バンドにおける電子の濃度は
  ∞                        ∞
n=∫D(E)f(E)dE={1/(2π^2)}(2m/h^2)^(2/3)exp(μ/kT)*∫(E-Ec)^(1/2)exp(-E/kT)dE
 Ec                         Ec

=2(mkT/2πh^2)^(3/2){exp(μ-Ec)/kT}

ここで、hはh/2πです。(ほんとはエイチバーとかきたかった)
mは電子の有効質量、μはフェルミエネルギー、Ecは伝導バンドの下端でのエネルギーです。
D(E)は状態密度です。
上の式から下の式への変形の仕方がわかりません。

よろしくおねがいします。
940921:02/11/24 13:48 ID:1iAdKH1F
二つ目の積分の積分区間の値の位置がずれてました。
もちろん、二つ目の積分記号の上と下にあります。
941925:02/11/24 14:14 ID:???
>>927 ありがとうございました。謎が氷解しました。
942ご冗談でしょう?名無しさん:02/11/24 14:16 ID:???
>>932
ニュートリノだけでダークマターを説明するのはどう考えても無理っぽい。
943787:02/11/24 15:51 ID:i+QjTC/T
>>820で質問しましたが回答が無かったのでもう一度質問させてください
ブラックホールに落下する物質の速度は光速を越えない、でいいですか?
944ご冗談でしょう?名無しさん:02/11/24 18:26 ID:???
>>943あまり深入りすると当局に捕まりますよ
945ご冗談でしょう?名無しさん:02/11/24 19:04 ID:???
>>943
今の理論の範囲内ではそうです。
946知ったか工房:02/11/24 20:00 ID:???
そうすると、ブラックホールに落ちていく宇宙船から信号を送れば
ブラックホールの外でその信号を受け取れることになるな…
947ご冗談でしょう?名無しさん:02/11/24 22:21 ID:R+r37kvK
図がないとわかりにくいのですが・・・

『なめらかな水平面上に、質量mの物体Aと質量M(M>m)の物体Bがあり、Bにばね定数kの
軽いばねが取り付けてある。A、Bおよびばねは一直線上にある,
静止しているBに向かってAを速さvでばねに衝突させた。衝突時には力学的エネルギーは
失われないものとする。 

衝突後、Aがばねと接触している間に、AとBの速度が等しくなるときがある。
このとき、A、Bは速さ(ア)で動き、ばねはもっとも(イ)いる。 』

(ア)がV=mv/M+m だということは分かったのですが、
(イ)の答えがなぜ「縮んで」なのか分かりません・・・

どなたか教えて下さいませ。

■     〜〜〜□
 ̄ ̄ ̄ ̄ ̄ ̄ ̄ ̄ ̄

下手くそですが図はこんな感じです。
(■が物体A、〜〜〜がばね、□が物体B)
948ご冗談でしょう?名無しさん:02/11/24 22:24 ID:???
(イ)に入るのは「縮んで」か「伸びて」だろ。「伸びて」の場合一体どんな
状況を想定してるんだよ。
949ご冗談でしょう?名無しさん:02/11/24 22:28 ID:R+r37kvK
いや、まぁ消去法で考えればそりゃそうなんですが。

解答によると
「AとBの相対速度が0となるときであるから、
AとBがもっとも近づくどきである、つまりばねは最も縮んでいる」

という説明なんですよ。これがよくわからない。。。
950ご冗談でしょう?名無しさん:02/11/24 22:41 ID:???
いいか、落ち着いて考えろ
間にばねがある以上、Aの速度の方が大きくなることはないだろ。
951ご冗談でしょう?名無しさん:02/11/24 23:06 ID:???
直感から明らかだが真面目に計算しようとすればA ,Bの速度をv_A v_B
,ばねの縮みをxとでもするだろ。
力学的エネルギー保存から mv^2/2=mv_A^2/2+Mv_B^2/2+kx^2/2
運動量保存から        mv=mv_A+Mv_B
このときx^2が最大になるのは(ア)の速度のときである、って示せばいいんだ。
微分か平方完成すれば簡単に示せるよ。
952947:02/11/25 00:15 ID:???
うーん。なんとなく分かりました。
ありがとうございました。
953ご冗談でしょう?名無しさん:02/11/25 00:26 ID:zdaaa+Ik
あの、くだらないことなんですけどふと思ったんで聞いてみます
スキーで使うゴーグルのような黒い眼鏡のことなんですが
あれは太陽の日差しを避けるためにきっと偏光現象を利用していると思うんですが
実際はどうなんでしょうか?
954ご冗談でしょう?名無しさん:02/11/25 00:28 ID:???
>>953
釣りですか?
955953:02/11/25 00:31 ID:???
いえ、違います。
太陽の光を抑えるためにサングラスとは違う効果があると思うんですけど。
もしかしてとんでもないこといってますか?
956ご冗談でしょう?名無しさん:02/11/25 00:32 ID:???
ゴーグル 偏光 で検索したら山ほど出てきますがな
957ご冗談でしょう?名無しさん:02/11/25 00:33 ID:???
太陽の日差しというか、雪に反射した日差し
反射光は偏光なので、偏光レンズを通せば反射光は見えず、
雪面がちゃんと見える
と、もっぱらの噂です
958ご冗談でしょう?名無しさん:02/11/25 00:33 ID:???
>>954
釣りにも使えるでしょう。
959ご冗談でしょう?名無しさん:02/11/25 00:35 ID:???
>>957
水面での反射ならだいぶと偏光してるけど、
雪面での反射ってなんで偏光が入るの?

微細な氷によるランダムな反射に思えるので。
960953:02/11/25 00:37 ID:???
そうですね。簡単なことなのにありがとうございました。
961ご冗談でしょう?名無しさん:02/11/25 00:50 ID:???
>>959
太陽が一つなら、目に入ってくる光はだいたい同じ角度で反射
してるんじゃないかな。
962ご冗談でしょう?名無しさん:02/11/25 00:55 ID:???
>>961
反射角度じゃなくて、偏光の向き〜氷の結晶の反射面の向き
だと思うのだが。
963ご冗談でしょう?名無しさん:02/11/25 01:08 ID:???
>>962
偏光面は確かに反射面によって決まるが、光源が太陽の一点だけと仮定すると、人間の
方に向かってくる光が反射してきた面は、全てほぼ同じ向きだろう。と思って961の
ように言ったのだが、確かにそれは全ての雪面上に対して成立する事ではないな。

偏光ゴーグルは、雪面上の特定の位置付近(光が反射してきた面の向きが比較的近い)
の反射にしか効果は無いということだろうか?
964ご冗談でしょう?名無しさん:02/11/25 02:14 ID:yWIOSqve
965ご冗談でしょう?名無しさん:02/11/25 02:16 ID:???
>>964
966ご冗談でしょう?名無しさん:02/11/25 02:51 ID:f+g7BRxy
ヒートポンプの仕組みがいまいち分からないのですが
成績係数が1を超えるのは熱力学の法則に反しないのでしょうか
COP=ヒートポンプにより利用可能となった熱(kW)
          /ヒートポンプに投入された動力(kW) 
ですよね。
ここでいうヒートポンプに投入されたエネルギーというのは
圧縮機関に投入されたエネルギーであって、実際に利用するエネルギーの
熱源は周囲の環境から集める(暖房時)という解釈でよろしいのでしょうか?
あとエアコンなどの説明書にある“能力”とは成績係数の
ことを表しているのでしょうか?      
967ご冗談でしょう?名無しさん:02/11/25 07:14 ID:???
庭から仏像が次々と出てきます。もう150体を超えました。
どうすればよいのでしょう?

ちなみにオレは神です。
968ご冗談でしょう?名無しさん:02/11/25 08:58 ID:dUEUzsJN
比電荷の測定実験で、
真空管内を電子が運動する際、内部に封入してあるへリウムガスと衝突して緑色の光を出す。
これによって電子の軌道を観察できるわけだが、
その軌道がある程度の幅を持っていること、そして、
その最も外側の軌道の半径を測った方が良いのは何故か。
誰か教えてくださいっ!!


969レザード ◆9dzDKWLpMo :02/11/25 09:33 ID:???
キックボードで走ってる時にハンドルを百八十度回転させるとどうなりますか?
970ご冗談でしょう?名無しさん:02/11/25 09:35 ID:???
>>969
実験してみればよいのでは?
971ご冗談でしょう?名無しさん:02/11/25 10:30 ID:???
っていうか、次スレ立ってないのですが。
972ご冗談でしょう?名無しさん:02/11/25 16:48 ID:StA5I2d0
TiBNとTiNとK10のデバイの特性温度もしくはデバイ振動数を教えてください。
純金属の場合は調べられるのですが、合金の場合は資料が見つかりません。
どうかよろしくお願いします。
973ご冗談でしょう?名無しさん:02/11/25 18:12 ID:???
>>968
磁場かかってる?それ
だとしたら外側の軌道と内側の軌道をとった電子で違うのは何?
974ご冗談でしょう?名無しさん:02/11/25 19:36 ID:???
>>921
遅スレで申し訳ない。

要するに、∫[0,∞]E^0.5exp(-βE)dEの計算ですね。([,]は積分範囲。)
x=E^0.5と置いて、I(2)=∫[0,∞]x^2exp(-βx^2)dxと変形すると(係数略)
I(2)=-∂I(0)/∂β  (ここでI(0)=∫[0,∞]exp(-βx^2)dx)

結局、I(0)を計算してβ(=kT)で微分すればよろしい。
I(0)の計算は分かりますよね?
975ご冗談でしょう?名無しさん:02/11/25 19:37 ID:???
すみません。
スペクトルって、それぞれ明るさが違いますよね。
その違いはどうして起きるんですか?
976975:02/11/25 19:47 ID:60qRhsU0
高校の教科書・参考書は調べたんですが、載ってません...
検索してもよくわかりません。
よろしくおねがいします。
977   :02/11/25 19:57 ID:???
スペクトルマン
978ご冗談でしょう?名無しさん:02/11/25 20:44 ID:???
>>975
工房には難しいと思うよ。
ところで、遷移のときのf値って何ですか?
979ご冗談でしょう?名無しさん:02/11/25 20:46 ID:???
印籠や成仏ってここ来ないな。
他人に教えられないんだろうなw
980964:02/11/25 20:48 ID:hgDjegW5
騙してごめんなさい、今度こそちゃんとたてたyo
■ちょっとした疑問はここに書いてね16■
http://science.2ch.net/test/read.cgi/sci/1038224562/
981ご冗談でしょう?名無しさん:02/11/25 21:10 ID:???
>>975
何のスペクトルの話かわからないのに答えられる奴はいない
982ご冗談でしょう?名無しさん:02/11/25 22:20 ID:???
>>981
プランク分布するから
983ご冗談でしょう?名無しさん:02/11/25 23:46 ID:???
高校物理でも扱ってますよ>975
ただし、TBではなくUの分野。
984ご冗談でしょう?名無しさん:02/11/26 00:01 ID:e0eDEBy0
>>975
>>981にもあるけど、ハロゲンランプみたいな連続光の事を言っているのか、
ナトリウムランプのようなスペクトル線のことを言っているのか、あるいは
もっと違うものの事なのか、はっきりして欲しい。
985921:02/11/26 00:50 ID:???
>>974
ありがとうございました。解けました。
やっぱりキッテルは難しい・・・。
というか、実際に扱ってる内容はそれほど難しくもないんでしょうけど、独特な表現が多くて分かりづらく感じます。
986ご冗談でしょう?名無しさん:02/11/26 01:51 ID:???
>>985
おいおい、キッテルはどれも名著ぞろいだぞ
987ご冗談でしょう?名無しさん:02/11/26 03:45 ID:???
キッテルは今日も張り切ってる
988975:02/11/26 11:25 ID:W+uqegH+
テストが1時間しかなかったのでこんな時間にきました。

>>981 >>984
すみません。ナトリウムランプです。線スペクトルです。
この実験をしてD線だけ異常に強いので、何でかな...と思ったのですが
先生も答えられませんでした。
「スペクトル」「強度」でグーグル検索してますが、内容が難しいです。

>>982
プランク分布、ですか。ありがとうございます。
なるほど、温度が高いほど短波長の光が出る確率が高くなるんですね。

>>983
まだUに入ってないです。IBの波動の実験です。
989ご冗談でしょう?名無しさん:02/11/26 11:35 ID:???
>>987
おいおい、キッテルはどれも張り切ってるぞ
990 :02/11/26 14:06 ID:Fz8Js/L0
僕が後ろで見ていたら、彼は、
12巡目くらいに中のトイツができました。彼は
3巡目に中を捨てていたのに、なぜか、彼は中のトイツを
大事に持っていました。タンピンを狙えそうな手で、
とてもアンパイ確保には思えませんでした。
ふと自分のカワに目をやった彼は、「あーー」と叫び、
次に「固体物理学だーーー」と言いました。
なぜ、これが固体物理学なのか、教えて下さい。
991ご冗談でしょう?名無しさん:02/11/26 14:14 ID:???
>>990
ここはキッテルで思い付いたことを書くスレじゃありませんよ。
992:02/11/26 14:46 ID:tteklzd6
http://www.emzshop.com/stirring/
資料請求してみてくれ!
2ちゃんの力を見せてやれ!



993ご冗談でしょう?名無しさん:02/11/26 16:10 ID:???
>>992
請求ははがきに切手ルを貼ればいいのか?
994ご冗談でしょう?名無しさん:02/11/26 17:26 ID:???
>>988
線スペクトルの話ならプランク分布じゃないよ
995ご冗談でしょう?名無しさん:02/11/26 18:20 ID:???
>>993
ちょっとそこのアナタ!
私の話を聞ッテル?!
996ご冗談でしょう?名無しさん:02/11/26 19:44 ID:???
1000Get
997ご冗談でしょう?名無しさん:02/11/26 21:12 ID:???
1001Get
998ご冗談でしょう?名無しさん:02/11/26 21:26 ID:???
あと2
9991000はくれてやる:02/11/26 21:26 ID:???
スリーナインget!!!!!
お言葉に甘えて1000初ゲット
10011001
このスレッドは1000を超えました。
もう書けないので、新しいスレッドを立ててくださいです。。。